Álgebra Nível 3

Fazer download em pdf ou txt
Fazer download em pdf ou txt
Você está na página 1de 130

Polos Olmpicos de Treinamento

Aula

Curso de lgebra - Nvel 3


Prof. Antonio Caminha

Desigualdades 1
Nesta aula, aprenderemos e exercitaremos a desigualdade entre as medias aritmetica e
geometrica e a desigualdade de Cauchy, bem como alguns corol
arios seus. Para saber mais
sobre o conte
udo desta aula, sugerimos as secoes 7.1 a 7.3 de [1], 2.3 e 2.4 de [3] e a secao
5.5 de [4].
A observac
ao b
asica para o estudo sistematico de desigualdades e o fato do quadrado
de todo n
umero real ser n
ao negativo, sendo igual a zero se e s
o se o n
umero em questao for
2
tambem igual a zero. Portanto, para x, y R temos (|x| |y|) 0, ocorrendo a igualdade
se e s
o se |x| = |y|. Desenvolvendo a express
ao entre parenteses, conclumos que

x2 + y 2
|xy|,
(1)
2
ocorrendo a igualdade se e s
o se |x|
umeros reais positivos
= |y|. Assim, partindo de dois n

a e b e fazendo x = a 0 e y = b 0, segue da desigualdade acima que


a+b
ab,
(2)
2

o se a = b.
ocorrendo a igualdade se e s
o se a = b, i.e., se e s
Exemplo 1. Para x, y, z reais positivos, temos
x2 + y 2 + z 2 xy + xz + yz,

(3)

ocorrendo a igualdade se, e somente se, x = y = z.


Prova. Para obter a desigualdade do enunciado, basta somar membro a membro as desigualdades parciais (obtidas a partir de (2))
x2 + y 2
x2 + z 2
y2 + z2
xy,
xz,
yz.
2
2
2
Se x = y = z, ent
ao a desigualdade do enunciado e claramente uma igualdade. Reci2
2
> xy,
procamente, se ao menos uma das desigualdades acima for estrita, digamos x +y
2
entao, ap
os somarmos as mesmas membro a membro, obteremos
x2 + y 2 + z 2 > xy + xz + yz.


POT 2012 - Algebra
- Nvel 3 - Aula 01 - Prof. Antonio Caminha

A desigualdade (2) e um caso particular da desigualdade entre as medias aritmetica


e geometrica. A fim de enunciar e provar tal generalizacao precisamos, inicialmente, da
seguinte
Defini
c
ao 2. Para n > 1 n
umeros reais positivos a1 , a2 , . . . , an , definimos sua:
(a) M
edia aritm
etica como o n
umero
(b) M
edia geom
etrica como o n
umero

a1 +a2 ++an
.
n

a1 a2 . . . an .

No contexto da definic
ao acima, o que fizemos em (2) foi mostrar que a media aritmetica
de dois reais positivos e sempre maior ou igual que sua media geometrica, ocorrendo a
igualdade somente se os dois n
umeros forem iguais. Estabelecemos o caso geral no resultado
a seguir, sendo (4) conhecida como a desigualdade entre as m
edias.
Teorema 3. Dados n > 1 reais positivos a1 , a2 , . . . , an , temos

a1 + a2 + + an
n a1 a2 an ,
n

(4)

ocorrendo a igualdade se e s
o se a1 = a2 = = an .
Para entender a din
amica da prova do teorema acima, analisemos separadamente os
casos n = 3 e n = 4, comecando
comc+do caso
n = 4. Para tanto, dados reais positivos
a+b
a, b, c, d, ja sabemos que 2 ab e 2 cd. Da,
a+b+c+d
=
4
Mostramos, acima, que
lugar de d, obtemos

a+b
2

+
2

a+b+c+d
4

a+b+c+
4

abc

c+d
2

ab +
2

cd

4
ab cd = abcd.

abcd. Escrevendo tal desigualdade com

q
4

abc no

4
3
3
abc abc = d3 d = d = abc.

3 abc.
Segue, da, a desigualdade a + b + c + 3 abc 4 3 abc, ou, o que e o mesmo, a+b+c
3
Conforme veremos a seguir, a prova da vers
ao geral da desigualdade entre as medias e
uma adaptac
ao dos argumentos utilizados para os dois casos acima.
Prova do Teorema 3. Inicialmente, provemos por inducao que a desigualdade desejada
e verdadeira sempre que n for uma potencia de 2, ocorrendo a igualdade se e s
o se a1 =
a2 = = an . Para tanto, temos de verificar o caso inicial n = 2 (o que ja foi feito ao longo
da discussao que estabeleceu (2)), formular a hip
otese de inducao (para n = 2j , digamos)
e executar o passo de induc
ao (deduzir o caso n = 2j+1 a partir do caso n = 2j ). Mas
desde que 2j+1 = 2 2j , basta supormos que a desigualdade seja verdadeira para quaisquer
k reais positivos, com igualdade se e s
o se os k n
umeros forem todos iguais, e deduzir a
partir da que ela tambem ser
a verdadeira para quaisquer 2k reais positivos, com igualdade


POT 2012 - Algebra
- Nvel 3 - Aula 01 - Prof. Antonio Caminha

novamente se e s
o se todos os n
umeros forem iguais. Para estabelecer esse fato, considere
os 2k reais positivos a1 , a2 , . . . , a2k . Entao:

2k
k
k
X
X
X

1 1
1
1
1

k
a1 . . . ak + k ak+1 . . . a2k
aj =
aj +
ak+j
2k
2 k
k
2
j=1
j=1
j=1
q

a1 . . . ak k ak+1 . . . a2k = 2k a1 . . . ak ak+1 . . . a2k .


Para haver igualdade, devemos ter igualdade em todas as passagens. Entao, deve ser

ak+1 + + a2k
a1 + + a k

= k a1 . . . ak ,
= k ak+1 . . . a2k
k
k
e

a1 . . . ak +

ak+1 . . . a2k

q
k

a1 . . . ak k ak+1 . . . a2k .

2
Para as duas primeiras igualdades, devemos ter por hip
otese que a1 = = ak e ak+1 =

= a2k . Por fim, a u


ltima igualdade ocorre se e s
o se k a1 . . . ak = k ak+1 . . . a2k , e esta
condicao, juntamente com as duas anteriores, implica que devemos ter a1 = = ak =
tambem evidente que, se os n
ak+1 = = a2k . E
umeros forem todos iguais, entao a
igualdade ocorre (verifique!). Logo, por inducao temos (4) verdadeira, com a condicao para
a igualdade dada no enunciado, sempre que n for uma potencia de 2.
Provemos agora, por induc
ao forte, que a desigualdade e verdadeira em geral, ocorrendo
a igualdade se e s
o se os n
umeros forem todos iguais. Para tanto, seja n > 1 natural e
a1 , a2 , . . . , an reais positivos dados. Tome k N tal que 2k > n. Aplicando a desigualdade
entre as medias aos n n
umeros a1 , a2 , . . . , an , juntamente com 2k n copias do n
umero

k
k
n
umeros), obtemos
a = a1 a2 . . . an (totalizando n + (2 n) = 2 n
q
p
p
a1 + + an + a + + a
2k
2k
2k
2k n =
n a2k n =

a
.
.
.
a

a
a
a2k = a.
1
n
2k
A partir da, obtemos a1 + a2 + + an + (2k n)a 2k a ou, ainda,

a1 + a2 + + an
a = n a1 a2 . . . an .
n
Para haver igualdade, segue da primeira parte que a1 = a2 = = an = a = = a.
Em particular, todos os n
umeros a1 , a2 , . . . , an devem ser iguais. Finalmente, e facil ver
que se esses n
umeros forem todos iguais, entao haver
a igualdade.
Exemplo 4. Para n > 1 reais positivos a1 , a2 , . . . , an , temos


1
1
1
+
+ +
n2 ,
(a1 + a2 + + an )
a1 a2
an
ocorrendo a igualdade se e s
o se a1 = a2 = = an .

(5)


POT 2012 - Algebra
- Nvel 3 - Aula 01 - Prof. Antonio Caminha

Prova. Aplicando duas vezes a desigualdade entre as medias, temos





 r

1
1
1
1
1
n 1
n
+
+ +

(n a1 a2 an ) n
= n2 .
(a1 + a2 + + an )
a1 a2
an
a1 a2
an

Para haver a igualdade, devemos ter a1 + a2 + + an = n n a1 a2 an , donde a1 = a2 =


= an . Reciprocamente, e imediato verificar que se todos os n
umeros forem iguais, entao
teremos igualdade em (5).

Exemplo 5 (Asia-Pac
fico). Se a, b e c s
ao reais positivos, prove que





a
a+b+c
c
b 
.
1+
2 1+
1+
1+
3
b
c
a
abc

Prova. Desenvolvendo o primeiro membro, obtemos





a
a+c b+c a+b
b 
c
1+
=2+
1+
+
+
,
1+
b
c
a
b
a
c
donde basta mostrarmos que

2(a + b + c)
a+c b+c a+b

+
+

.
3
b
a
c
abc
Denotando por S o primeiro membro da express
ao acima, segue da desigualdade entre
as medias e de (5) que


1 1 1
+ +
3
S = (a + b + c)
a b
c




1 1 1
1 1 1
1
2
(a + b + c)
+ +
+ +
+ (a + b + c)
3
=
3
a b
c
3
a b
c


1
2
3

+ 93

(a + b + c) 3
3
3
abc
2(a + b + c)

=
.
3
abc

Voltando a (1), suponha dados n


umeros reais a1 , a2 , a3 e b1 , b2 , b3 , tais que a21 +a22 +a23 =
1 e b21 + b22 + b23 = 1. Temos
a21 + b21 |a1 b1 |, a22 + b22 |a2 b2 |, a23 + b23 |a3 b3 |,

(6)

ocorrendo a igualdade se e s
o se |a1 | = |b1 |, |a2 | = |b2 |, |a3 | = |b3 |. Somando membro a
membro as desigualdades acima, obtemos
(a21 + a22 + a23 ) + (b21 + b22 + b23 ) = (a21 + b21 ) + (a22 + b22 ) + (a23 + b23 )
2(|a1 b1 | + |a2 b2 | + |a3 b3 |)

2|a1 b1 + a2 b2 + a3 b3 |,
4


POT 2012 - Algebra
- Nvel 3 - Aula 01 - Prof. Antonio Caminha

onde, na u
ltima desigualdade, aplicamos a desigualdade triangular para tres n
umeros (cf.
Problema 1).
Portanto, provamos acima que, se a21 + a22 + a23 = 1 e b21 + b22 + b23 = 1, entao
|a1 b1 + a2 b2 + a3 b3 | 1.

(7)

A igualdade ocorre se e s
o se tivermos igualdade tanto nas desigualdades em (6) quanto
na desigualdade triangular utilizada, i.e., se e s
o se |a1 | = |b1 |, |a2 | = |b2 | e |a3 | = |b3 | e,
alem disso, a1 b1 , a2 b2 , a3 b3 0 ou a1 b1 , a2 b2 , a3 b3 0. Mas e imediato verificar que tais
condicoes s
ao equivalentes a a1 = b1 , a2 = b2 e a3 = b3 .
Considere, agora, n
umeros reais a1 , a2 , a3 e b1 , b2 , b3 quaisquer, exceto pelo fato de que
pelo menos um
dos
n
u
meros
a1 , a2 , ap
umeros b1 , b2 , b3 s
ao n
ao nulos.
3 e pelo menos um dos n
p
2
2
2
2
2
2
Fazendo c = a1 + a2 + a3 e d = b1 + b2 + b3 , temos c, d > 0; portanto, se xi = aci e
a2 +a2 +a2

yi = bdi , para 1 i 3, temos x21 +x22 +x23 = 1 c22 3 = 1 e, analogamente, y12 +y22 +y32 = 1.
Segue, pois, de (7) que
|x1 y1 + x2 y2 + x3 y3 | 1,

com igualdade se e s
o se xi = yi para 1 i 3.
Substituindo as definic
oes de xi e yi na desigualdade acima, conclumos ser ela equivalente `a desigualdade
q
q
|a1 b1 + a2 b2 + a3 b3 | cd = a21 + a22 + a23 b21 + b22 + b23 .

Ademais, h
a igualdade se e s
o se ai = dc bi para 1 i 3.
A discuss
ao acima estabeleceu, para n = 3, a desigualdade do teorema a seguir, conhecida como a desigualdade de Cauchy.
Teorema 6 (Cauchy). Sejam n > 1 inteiro e a1 , a2 , . . . , an , b1 , b2 , . . . , bn n
umeros reais
dados. Ent
ao
v

v
uX
X
uX
u n
n
u n 2
2


t

a
b
aj t
bj ,
(8)
j j


j=1
j=1
j=1

ocorrendo a igualdade se e s
o se os ai e os bi forem respectivamente proporcionais, i.e., se
e s
o se existir um real n
ao nulo tal que a1 = b1 , a2 = b2 , an = bn .
Prova. Se todos os ai ou todos os bi forem iguais a zero, nada h
a a fazer. Sen
ao, a fim de
estabelecer (8), basta seguir os passos do caso particular n = 3 discutido acima, tomando
o cuidado de, no momento oportuno, utilizar o caso geral da desigualdade triangular.
Os dois exemplos a seguir ilustram a utilizacao da desigualdade de Cauchy.
Exemplo 7 (Romenia). Sejam x1 , x2 , . . ., xn+1 reais positivos tais que x1 + x2 + + xn =
xn+1 . Prove que
p
p
p
x1 (xn+1 x1 ) + + xn (xn+1 xn ) xn+1 (xn+1 x1 ) + + xn+1 (xn+1 xn ).
5


POT 2012 - Algebra
- Nvel 3 - Aula 01 - Prof. Antonio Caminha

Prova. Para 1 j n seja yj = xn+1 xj . Pela desigualdade de Cauchy, temos

x1 y 1 + + xn y n
x1 + + xn y 1 + + y n
p

=
xn+1 (xn+1 x1 ) + + (xn+1 xn ).
Exemplo 8. Dados n
umeros reais a1 , . . . , an e b1 , . . . , bn , temos
v
v
v
uX
u n
uX
u n 2 uX
u n
2
t (aj + bj ) t
aj + t
b2j ,
j=1

j=1

(9)

j=1

ocorrendo a igualdade se e s
o se a1 , . . . , an e b1 , . . . , bn forem positivamente proporcionais,
i.e., se e s
o se existir um real positivo , tal que ai = bi para 1 i n.

Prova. Facamos a prova para n = 3, sendo o caso geral inteiramente an


alogo. Uma vez
que ambos os membros de (9) s
ao reais n
ao negativos, basta mostrar que o quadrado do
primeiro membro e menor ou igual que o quadrado do segundo membro, i.e., que
2
q
q
2
2
2
2
2
2
2
2
2
a1 + a2 + a3 + b1 + b2 + b3 .
(a1 + b1 ) + (a2 + b2 ) + (a3 + b3 )
Desenvolvendo todos os quadrados (ai +bi )2 , segue que o quadrado do primeiro membro
e igual a
(a21 + 2a1 b1 + b21 ) + (a22 + 2a2 b2 + b22 ) + (a23 + 2a3 b3 + b23 ).
Analogamente, o quadrado do segundo membro e igual a
q
q
2
2
2
2
2
2
(a1 + a2 + a3 ) + 2 a1 + a2 + a3 b21 + b22 + b23 + (b21 + b22 + b23 ).

Mas, como em ambas as express


oes temos a parcela (a21 + a22 + a23 ) + (b21 + b22 + b23 ), a
desigualdade do enunciado e equivalente a
q
q
2(a1 b1 + a2 b2 + a3 b3 ) 2 a21 + a22 + a23 b21 + b22 + b23 ,

a qual e, precisamente, a desigualdade de Cauchy.


A deducao das condic
oes para a igualdade fica a cargo do leitor.

Problemas
1. * Dados n
umeros reais n
ao nulos x1 , x2 , . . . xn , prove a desigualdade triangular:
|x1 + x2 + + xn | |x1 | + |x2 | + + |xn |,
ocorrendo a igualdade se e s
o se x1 , x2 , . . . , xn tiverem um mesmo sinal.
6


POT 2012 - Algebra
- Nvel 3 - Aula 01 - Prof. Antonio Caminha

2. (OBM). Sejam a, b, c reais positivos dados. Prove que


p
(a + b)(a + c) 2 abc(a + b + c).

3. Dispomos de uma folha de cartolina de 2m por 3m e queremos construir com a mesma


uma caixa aberta com o maior volume possvel. Quais devem ser as dimensoes da
caixa? Justifique sua resposta.
4. (Estados Unidos). Prove que, para todos a, b, c reais positivos, tem-se
1
1
1
1
+
+

.
a3 + b3 + abc b3 + c3 + abc c3 + a3 + abc
abc
Para os dois problemas a seguir precisamos de um pouco de geometria Euclidiana
plana. Mais precisamente (cf. Figura 1), sendo a = BC, b = AC e c = AB os
comprimentos dos lados de um tri
angulo ABC, existem x, y, z > 0 tais que a = y + z,
b = x + z e c = x + y: basta tomar x, y e z como sendo iguais aos comprimentos
dos segmentos determinados sobre os lados de ABC pelos pontos de tangencia com
os mesmos do crculo inscrito em ABC (para uma prova de tais afirmacoes, veja o
Captulo 3 de [2]). No contexto de desigualdades envolvendo os lados a, b e c de
um tri
angulo, a substituic
ao dos mesmos respectivamente por y + z, x + z e x + y e
conhecida como a transforma
c
ao de Ravi.
y B y
x

I
x

Figura 1: a transformacao de Ravi.

5. (IMO). Se a, b, c s
ao os comprimentos dos lados de um tri
angulo, prove que
abc (a + b c)(b + c a)(c + a b).
6. Sejam a, b, c os comprimentos dos lados de um tri
angulo. Prove que
b
c
a
+
+
3.
b+ca c+ab a+bc
7. (Baltic Way). Sejam a, b, c, d reais positivos dados. Prove que
a+c b+d c+a d+b
+
+
+
4.
a+b b+c c+d d+a
7


POT 2012 - Algebra
- Nvel 3 - Aula 01 - Prof. Antonio Caminha

8. (Uniao Sovietica). Sejam a, b, c reais positivos. Prove que


(ab + ac + bc)2 3abc(a + b + c).
9. Sejam a, b e c reais positivos dados. Prove que 9(a3 + b3 + c3 ) (a + b + c)3 .
10. Dados a, b, c reais positivos, prove que
a4 (1 + b4 ) + b4 (1 + c4 ) + c4 (1 + a4 ) 6a2 b2 c2 ,
com igualdade se e s
o se |a| = |b| = |c| = 1.
11. Sejam a1 , a2 , . . . , an reais positivos. Prove que
a1 a2 a3
an1 an
+
+
+ +
+
n.
a2 a3 a4
an
a1
12. O proposito deste problema e apresentar uma segunda demonstracao da desigualdade
(5), a qual n
ao faz uso da desigualdade (4). Para tanto, faca os dois itens a seguir:
(a) Mostre que
(a1 + a2 + + an )

1
1
1
+
+ +
a1 a2
an

=n+


aj
+
.
aj
ai

X  ai
i<j

(b) Aplique a desigualdade entre as medias para cada uma das parcelas
somat
orio acima e obtenha (5).

ai
aj

aj
ai

do

13. (Romenia). Sejam n > 1 inteiro e 0 < a1 < a2 < < an reais dados. Prove que
12
22
n2
n
n1
n2
1
+
+ +

+
+
+ +
.
a1 a2
an
a1 a2 a1 a3 a2
an an1
Sob que condic
oes a igualdade ocorre?
14. (China). Para a, b e c reais positivos, prove que
s 



a+b
a + ab + 3 abc
a+b+c
3
a
.
3
2
3
15. Dados reais positivos a1 , a2 , . . . , an , definimos sua media quadr
atica como o n
umero
real
r
a21 + a22 + + a2n
.
n
Prove a desigualdade entre as medias quadr
atica e aritmetica:
r
a1 + a2 + + an
a21 + a22 + + a2n

,
(10)
n
n
com igualdade se e s
o se a1 = a2 = = an .
8


POT 2012 - Algebra
- Nvel 3 - Aula 01 - Prof. Antonio Caminha

16. Sejam a1 , a2 , a3 , a4 reais positivos. Prove que


X

1i<j<k4

a2i + a2j + a2k


a1 + a2 + a3 + a4 ,
ai + aj + ak

ocorrendo a igualdade se e s
o se a1 = a2 = a3 = a4 .
17. (Leningrado). Dados reais positivos a, b, c e d, prove que
1 1 4 16
64
+ + +

.
a b
c
d
a+b+c+d
18. (Uniao Sovietica). Se x, y, z > 0, prove que

x2
y2

y2
z2

z2
x2

y
x

z
y

+ xz .

19. (Torneio das Cidades). Sejam a1 , a2 , . . . , an reais positivos dados. Prove que





a21
a22
a2n
1+
1+
... 1 +
(1 + a1 )(1 + a2 ) (1 + an ).
a2
a3
a1
20. (IMO). Sejam a, b e c reais positivos tais que abc = 1. Prove que
1
1
3
1
+ 3
+ 3
.
+ c) b (a + c) c (a + b)
2

a3 (b

Bibliografia
1. A. Caminha. T
opicos de Matem
atica Elementar, Volume 1: N
umeros Reais. Sociedade Brasileira de Matematica, Rio de Janeiro, 2012.
2. A. Caminha. T
opicos de Matem
atica Elementar, Volume 2: Geometria Euclidiana
Plana. Sociedade Brasileira de Matematica, Rio de Janeiro, 2012.
3. E. Lozansky e C. Rousseau. Winning Solutions. Springer-Verlag, Nova Iorque, 1996.
4. P. Zeitz. The Art and Craft of Problem Solving. John Wiley & Sons, Nova Iorque,
1999.


POT 2012 - Algebra
- Nvel 3 - Aula 01 - Prof. Antonio Caminha

Dicas e Soluc
oes
1. Faca induc
ao sobre n 2. Para o caso inicial, eleve ambos os membros da desigualdade em quest
ao ao quadrado e utilize (1), juntamente com o fato de que |a|2 = a2 ,
para todo a R.
2. Desenvolva o primeiro membro e, em seguida, aplique a desigualdade (2) adequadamente.
3. Sendo x o comprimento do lado do quadrado que deve ser recortado de cada canto
da folha, ficaremos com uma caixa de dimensoes 2 2x, 3 2x e x. Escolha n
umeros
reais positivos a, b e c tais que a(2 2x) + b(3 2x) + cx independa de x e a(2 2x) =
b(32x) = cx; em seguida aplique a desigualdade entre as medias a fim de maximizar
o volume da caixa.
4. Mostre inicialmente que a3 + b3 (a + b)ab; em seguida, deduza a partir da que
c
1
alogas para as outras duas parcelas
a3 +b3 +abc abc(a+b+c) , obtendo desigualdades an
do primeiro membro.
5. Aplique a transformac
ao de Ravi e, em seguida, utilize (2) tres vezes.
6. Aplique a transformac
ao de Ravi para escrever o primeiro membro como


z
x z y x
1 y
+ + + + +
;
2 x x y
y z
z
em seguida, utilize a desigualdade entre as medias.
7. Agrupe adequadamente as quatro parcelas em pares e utilize duas vezes a desigualdade (5), para n = 2.
8. Inicialmente, mostre que e suficiente provar que (ab)2 + (bc)2 + (ca)2 abc(a + b + c);
para o que falta, faca x = ab, y = bc, z = ca e aplique a desigualdade (3).
9. Inicialmente, mostre a identidade algebrica
(a + b + c)3 = a3 + b3 + c3 + 3(a + b)(a + c)(b + c);
em seguida, ap
os efetuar as simplificacoes obvias, mostre que
8(a3 + b3 + c3 ) (a + b)3 + (a + c)3 + (b + c)3
e utilize a desigualdade entre as medias para tres n
umeros.
10. Aplique a desigualdade entre as medias.
11. Aplique a desigualdade entre as medias.

10


POT 2012 - Algebra
- Nvel 3 - Aula 01 - Prof. Antonio Caminha

13. Faca a0 = 0 e aplique a desigualdade (5) para obter


1
j2
1
+ +
.
aj aj1
a1 a0
aj
Em seguida, some membro a membro as desigualdades acima para 1 j n e
agrupe os termos iguais para obter a desigualdade procurada. Por fim, conclua que
h
a igualdade se e s
o se a sequencia (ak )k1 for uma PA.
14. Substituindo a, b e c na desigualdade desejada respectivamente por 6x6 , 6y 6 e 6z 6 ,
mostre que basta provarmos a desigualdade
7x12 + 12x6 y 6 + 7y 6 z 6 + 9y 12 + 9x6 z 6
2x3 y 9 + 6x9 y 3 + 6x2 y 8 z 2 + 12x5 y 5 z 2 + 6x4 y 4 z 4 + 6xy 7 z 4 + 6x8 y 2 z 2 .
Para tanto, escreva a express
ao do primeiro membro como a soma de sete outras
express
oes tais que, aplicando a desigualdade entre as medias a cada uma delas,
obtenhamos as sete parcelas do segundo membro; por exemplo,
2x6 z 6 + 2x6 z 6 + 2x12 + 2y 12 + 2y 12 + 2x6 y 6 12x5 y 5 z 2 .
15. Aplique a desigualdade de Cauchy com b1 = b2 = = bn = 1.
16. Aplique a desigualdade do problema anterior ao numerador de cada parcela do somat
orio acima.
17. Multiplique ambos os membros por a + b + c + d e use a desigualdade de Cauchy;
alternativamente, tente aplicar a desigualdade (5).
18. Use a desigualdade de Cauchy.
19. Faca a prova por induc
ao sobre n 2. Para o passo de inducao, aplique a desigualdade
de Cauchy.
20. Faca x =
obter

1
a,

y =

1
b

ez =

1
c

e, em seguida, aplique a desigualdade de Cauchy para

((y + z) + (x + z) + (x + y))

x2
y2
z2
+
+
y+z x+z x+y

Por fim, aplique a desigualdade entre as medias.

11

(x + y + z)2 .

Polos Olmpicos de Treinamento


Aula

Curso de lgebra - Nvel 3


Prof. Antonio Caminha

Desigualdades 2
Esta aula e devotada ao estudo de outras desigualdades elementares importantes. Para
saber mais sobre o material aqui discutido, remetemos o leitor ao Captulo 2 de [1], `a Secao
7.4 de [2], ao Captulo 7 de [3] ou, por fim, `a Secao 2.4 de [4].
A primeira desigualdade que apresentamos remonta os irm
aos Bernoulli (Jacob e Johann
Bernoulli, matem
aticos sucos do seculo XVIII), sendo conhecida como a desigualdade
de Bernoulli. Apesar de sua aparente simplicidade, veremos que ela se revela bastante
u
til em aplicac
oes.
Proposi
c
ao 1 (Bernoulli). Dados n natural e x > 1 real, temos (1 + x)n 1 + nx,
ocorrendo a igualdade para n > 1 se e s
o se x = 0.
Prova. Facamos induc
ao sobre n, sendo o caso n = 1 imediato. Suponha, por hip
otese de
inducao, que (1 + x)k 1 + kx; como 1 + x > 0, temos
(1 + x)k+1 = (1 + x)(1 + x)k (1 + x)(1 + kx) = 1 + (k + 1)x + kx2 1 + (k + 1)x,
ocorrendo a igualdade se e s
o se (1 + x)k = 1 + kx e kx2 = 0, i.e., se e s
o se x = 0.
Exemplo 2. Dados n natural e a e b reais positivos, mostre que



a n
b n
1+
+ 1+
2n+1 ,
b
a
ocorrendo a igualdade se e s
o se a = b.
Prova. Dividindo ambos os membros da desigualdade do enunciado por 2n , vemos que
basta provar que




1
a n
b n
1
+ 1 +
2.
1 +
2 2b
2 2a
a
b
Como 21 + 2b
> 1 e 21 + 2a
> 1, aplicando a desigualdade de Bernoulli a cada parcela
do primeiro membro acima e somando os resultados, obtemos






a
1
a n
b n
b
1
+ 1 +
2+n
+
1 .
1 +
2 2b
2 2a
2b 2a


POT 2012 - Algebra
- Nvel 3 - Aula 02 - Prof. Antonio Caminha

Basta, agora, aplicar a desigualdade entre as medias para obter


r
b
a
a b
+
12

1 = 0,
2b 2a
2b 2a
com igualdade se e s
o se

a
2b

b
2a ,

i.e., se e s
o se a = b.

A pr
oxima desigualdade que apresentamos e conhecida na literatura como a desigualdade de Chebyshev, assim nomeada ap
os Pafnuty Chebyshev, matem
atico russo do
seculo XIX.
Teorema 3 (Chebyshev). Se a1 , a2 , . . . , an e b1 , b2 , . . . , bn s
ao n
umeros reais tais que
a 1 a 2 a n e b1 b2 b n ,
ent
ao

1X
ai
n
i=1

1X
bi
n
i=1

1X
ai bi ,
n
i=1

ocorrendo a igualdade se e s
o se a1 = a2 = = an ou b1 = b2 = = bn .
Prova. Temos de mostrar que
n

n
X
i=1

a i bi

n
X

ai

i=1

n
X

bi

i=1

0,

para o que basta observar que a express


ao do primeiro membro e igual a
n
X

(ai aj )(bi bj ).

i,j=1

Mas, como os ai s e bi s s
ao igualmente ordenados, conclumos que a express
ao acima e,
realmente, n
ao negativa.
Note agora que, se a1 = a2 = = an ou b1 = b2 = = bn , entao haver
a igualdade
na desigualdade de Chebyshev. Reciprocamente, suponha que temos igualdade em tal
desigualdade. Como (ai aj )(bi bj ) 0 para todos os ndices i, j, para haver igualdade
devemos ter (ai aj )(bi bj ) = 0 para todos i, j = 1, . . . , n. Se existir 1 k n tal que
bk < bk+1 , ent
ao b1 bk < bk+1 bn e a condicao (ai ak+1 )(bi bk+1 ) = 0
para todo i garante que ai = ak+1 para i k. Portanto, temos a1 = a2 = = ak = ak+1 .
Por outro lado, a partir de (ai ak )(bi bk ) = 0 para i > k, conclumos que ak+1 = = an .
Logo, todos os ai s devem ser iguais.
Exemplo 4. Se k e um natural e a1 , a2 , . . . , an s
ao reais positivos, entao


ak1 + ak2 + + akn
a1 + a2 + + an k
,

n
n
com igualdade se e s
o se todos os ai s forem iguais.
2

(1)


POT 2012 - Algebra
- Nvel 3 - Aula 02 - Prof. Antonio Caminha

Prova. Facamos induc


ao sobre k 1, sendo (1) trivialmente verdadeira para k = 1 e
todos os a1 , a2 , . . . , an reais positivos. Seja agora l > 1 um natural tal que (1) valha para
k = l 1 e todos a1 , a2 , . . . , an reais positivos. Dados reais positivos a1 , a2 , . . . , an , como
ambos os membros da desigualdade que queremos provar s
ao invariantes por permutacoes
dos ndices 1, 2, . . . , n podemos supor, sem perda de generalidade, que a1 a2 an .
Da, temos al1
al1
al1
n , e segue da desigualdade de Chebyshev que
1
2
!
!
n
n
n
1 X l1
1X
1X l
.
ai
ai
ai
n
n
n
i=1

i=1

i=1

Por outro lado, a hip


otese de induc
ao fornece
n

1 X l1
ai
n

1X
ai
n

i=1

i=1

!l1

e combinando essas duas desigualdades obtemos


n

1X
ai
n

1X l
ai
n
i=1

i=1

!l

conforme desejado. Por fim, a condic


ao de igualdade e obvia a partir condicao de igualdade
na desigualdade de Chebyshev.
Exemplo 5 (Pol
onia). Sejam a1 , a2 , . . . , an reais positivos com soma s. Prove que
a1
a2
an
n
+
+ +

.
s a1 s a2
s an
n1
Prova. Suponhamos, sem perda de generalidade, que a1 a2 an . Entao s a1
1
1
1
s a2 s an . Como s ai > 0 para todo i, segue que sa
.
sa
sa
n
1
2
Portanto, pela desigualdade de Chebyshev temos
! n
!
!

n
n
n 
n
X 1
X
X
ai
1 X
s X 1
1
ai
=

=
.
ai
s ai
s ai
n
s ai
n
s ai
i=1

i=1

i=1

i=1

i=1

Mas, pelo Exemplo 4 da Aula 1, temos


n
X
i=1

!1
n
X
1
n2
2
(s ai )
= n2 (ns s)1 =
n
.
s ai
(n 1)s
i=1

Por fim, basta combinar as duas desigualdades acima.


A segunda igualdade que apresentamos e conhecida como a desigualdade do rearranjo. Para o enunciado da mesma, recorde que uma permuta
c
ao de uma sequencia
(a1 , a2 , . . . , an ), e uma sequencia (x1 , x2 , . . . , xn ) tal que xi = a(i) , para alguma bijecao
: {1, 2, . . . , n} {1, 2, . . . , n}.
3


POT 2012 - Algebra
- Nvel 3 - Aula 02 - Prof. Antonio Caminha

Proposi
c
ao 6. Sejam a1 < a2 < < an reais positivos dados. Se (x1 , x2 , . . . , xn ) e uma
permutaca
o qualquer de (a1 , a2 , . . . , an ), ent
ao
n1
X
i=1

ai ani

n1
X
i=1

ai x i

n1
X

a2i ,

i=1

ocorrendo a igualdade na primeira (resp. segunda) desigualdade acima se e s


o se xi = ani
(resp. xi = ai ) para 1 i n.
Prova. Mostremos como maximizar a soma a1 x1 + a2 x2 + + an xn , sendo o raciocnio
para minimiza-la totalmente an
alogo.
Como o n
umero de permutac
oes (x1 , x2 , . . . , xn ) dos ai s e finito, h
a pelo menos uma
delas que maximiza a soma a1 x1 +a2 x2 + +an xn . Se (b1 , b2 , . . . , bn ) e uma tal permutacao,
queremos mostrar que bi = ai para 1 i n, e para tanto basta mostrarmos que deve ser
b1 < b2 < < bn . Suponha o contr
ario, i.e., que existam ndices i < j tais que bi > bj .
Defina a permutac
ao (b1 , b2 , . . . , bn ) dos ai s pondo

bk , se k 6= i, j

bk =
b , se k = j
.
i
bj , se k = i
Entao

n
X
i=1

ai bi

n
X
i=1

ai bi = (ai bi + aj bj ) (ai bi + aj bj )
= (ai bj + aj bi ) (ai bi + aj bj )
= (ai aj )(bj bi ) > 0.

Isso e o mesmo que


a1 b1 + a2 b2 + + an bn > a1 b1 + a2 b2 + + an bn ,
o que por sua vez contraria o fato de que a permutacao (b1 , b2 , . . . , bn ) dos ai s maximiza a
soma a1 x1 + a2 x2 + + an xn m
aximo. Logo, b1 < b2 < < bn .
Exemplo 7. Dados reais positivos a, b e c, mostre que a3 + b3 + c3 a2 b + b2 c + c2 a.
Prova. Suponha, sem perda de generalidade, que a b c. Uma aplicacao direta da
desigualdade do rearranjo nos d
a
a3 + b3 + c3 = a2 a + b2 b + c2 c a2 b + b2 c + c2 a.

Conforme mostrado pelo pr


oximo exemplo, a ideia da prova da desigualdade do rearranjo e, por vezes, t
ao u
til quanto a desigualdade em si.
4


POT 2012 - Algebra
- Nvel 3 - Aula 02 - Prof. Antonio Caminha

Exemplo 8 (Eslovenia). Dados 2n reais positivos a1 , a2 , . . . , a2n , como devemos arranj


a-los
em pares de modo que a soma dos n produtos dos n
umeros de cada par seja m
axima?
Solu
c
ao. Suponha, sem perda de generalidade, que a1 a2 a2n e arranje tais
n
umeros em n pares (b1 , c1 ), (b2 , c2 ), . . . , (bn , cn ), com b1 b2 bn e bj cj , para
todo j. Queremos maximizar
S = b1 c1 + b2 c2 + + bn cn .
Para isso, vamos mostrar que c1 c2 cn . De fato, se a sequencia (ci ) n
ao for n
ao
decrescente, existir
ao ndices i > j tais que ci cj . Neste caso, trocamos as posicoes de ci
e cj em S, ap
os o que a nova soma ser
a
S = S bi ci bj cj + bi cj + bj ci = S + (bi bj )(cj ci ) S.
Logo, a soma ser
a m
axima quando c1 c2 cn .
Finalmente, observe que, para todos i < j, temos bi ci cj . Suponha que, para
algum par i < j, tivessemos bj ci . Neste caso, trocamos ci por bj , de modo que a nova
soma seja
S = S bi ci bj cj + bi bj + ci cj = S + (bi cj )(bj ci ) S.
Portanto, devemos ter, para todos i < j, bi ci bj cj . Em geral, teremos
b1 c1 b2 c2 bn cn .
Logo, a soma m
axima e a1 a2 + a3 a4 + + a2n1 a2n .
A u
ltima desigualdade que discutiremos aqui e devida ao matem
atico noruegues do
seculo XIX Niels Henrik Abel, sendo conhecida como a desigualdade de Abel.
Teorema 9 (Abel). Sejam n > 1 natural e a1 , a2 , . . . , an , b1 , b2 , . . . , bn n
umeros reais
dados, com a1 a2 an 0. Se M e m denotam respectivamente os elementos
m
aximo e mnimo do conjunto de somas {b1 , b1 + b2 , . . . , b1 + b2 + . . . + bn }, ent
ao
ma1 a1 b1 + a2 b2 + + an bn M a1 .
Prova. Provemos a desigualdade da direita, sendo a prova da desigualdade da esquerda
totalmente an
aloga.
Faca s0 = 0 e si = b1 + + bi , para 1 i n. Entao
n
X

a i bi =

i=1

n
X

i=1
n1
X

ai (si si1 ) =

n
X
i=1

ai si

n1
X
i=0

(ai ai+1 )si + an sn

i=1
n1
X
i=1

M (ai ai+1 ) + M an = M a1 .

ai+1 si


POT 2012 - Algebra
- Nvel 3 - Aula 02 - Prof. Antonio Caminha

Para referencia futura, observamos que, nas notacoes da prova do teorema acima, a
igualdade
n1
n
X
X
(ai ai+1 )si + an sn
(2)
ai bi =
i=1

i=1

e conhecida como a identidade de Abel, sendo quase tao u


til quanto a desigualdade de
Abel em si.
Exemplo 10 (Romenia - adaptado). Sejam n > 1 inteiro e x1 , . . . , xn , y1 , . . . , yn reais
positivos tais que x1 y1 < x2 y2 < < xn yn e, para 1 k n, x1 + + xk y1 + + yk .
Prove que
1
1
1
1
1
1
+
+ +

+
+ + .
x1 x2
xn
y1 y2
yn
Prova. Inicialmente, observe que
n
n
n
X
X
xi y i
1 X 1

=
.
yi
xi
xi y i
i=1

i=1

(3)

i=1

Por outro lado, a condic


ao x1 + + xk y1 + + yk para 1 k n pode ser escrita
como
(x1 y1 ) + + (xk yk ) 0

para 1 k n. Assim, fazendo ai = xi1yi e bi = xi yi para 1 i n, temos


i
= ai bi para 1 i n.
0 < a1 < a2 < < an , b1 + + bi 0 e xxi y
i yi
Aplicando a desigualdade de Abel a (3), obtemos entao
n
n
X
1 X 1

an min{b1 + + bi ; 1 i n} 0.
yi
xi
i=1

i=1

Problemas
1. Para n N, prove que


1
1+
n

n

<

1
1+
n+1

2. (Estados Unidos). Para m, n naturais, seja a =

n+1

mm+1 +nn+1
mm +nn .

am + an m m + n n .

Prove que


POT 2012 - Algebra
- Nvel 3 - Aula 02 - Prof. Antonio Caminha

3. (Cro
acia). Encontre todas as solucoes reais positivas do sistema de equacoes

x1 + x2 + + x1994 = 1994
.
x41 + x42 + + x41994 = x31 + x32 + + x31994
4. Sejam a1 , a2 , a3 , a4 reais positivos. Prove que
X

1i<j<k4

a3i + a3j + a3k


a21 + a22 + a23 + a24 ,
ai + aj + ak

ocorrendo a igualdade se e s
o se a1 = a2 = a3 = a4 .
5. Sejam n > 1 inteiro e a1 , a2 , . . . , an , b1 , b2 , . . . , bn reais dados, com a1 a2 an
e b1 b2 bn . Se 1 2 n s
ao reais positivos com soma igual a 1,
prove que
!
! n
n
n
X
X
X
i ai bi
i bi
i ai
i=1

i=1

i=1

e de condic
oes necessarias e suficientes para a igualdade. A que caso particular
corresponde a desigualdade de Chebyshev?

6. Sejam a, b, c reais positivos. Prove que


a8 + b8 + c8
1 1 1
+ +
.
a b
c
a3 b3 c3
7. Sejam a, b, c, d reais n
ao negativos tais que ab + bc + cd + da = 1. Prove que
a3
b3
c3
d3
1
+
+
+
.
b+c+d a+c+d a+b+d a+b+c
3
8. Para a, b, c reais positivos e n N, prove que
bn
cn
an1 + bn1 + cn1
an
+
+

.
b+c a+c a+b
2
9. Sejam x, y e z reais positivos tais que xyz = 1. Prove que
x3
y3
z3
3
+
+
.
(1 + y)(1 + z) (1 + x)(1 + y) (1 + x)(1 + y)
4
10. Sejam n > 1 inteiro e x1 , x2 , . . . , xn reais positivos dados, com soma igual a 1. Prove
que
r
n
n
X
X

n
1
x
i

xi .

1
1 xi
n 1 i=1
i=1
11. Dados reais positivos a, b e c, mostre que
7

a+b+c
abc

1
a2

1
b2

1
.
c2


POT 2012 - Algebra
- Nvel 3 - Aula 02 - Prof. Antonio Caminha

12. (IMO). Seja (ak )k1 uma sequencia de inteiros positivos dois a dois distintos. Prove
que, para todo n N, temos
n
n
X
ak X 1
.

k2
k
k=1

k=1

13. (Torneio das Cidades). Sejam a1 , a2 , . . . , an reais positivos dados. Prove que


a2
1+ 1
a2



a2
1+ 2
a3


 Y
n
a2n
(1 + ak ).
1 +

a1
k=1

14. Sejam ai , bi n
umeros reais tais que a1 a2 a1 > 0 e b1 a1 , b1 b2 a1 a2 ,
. . . , b1 b2 bn a1 a2 an . Mostre que
b1 + b2 + + bn a1 + a2 + + an .

Bibliografia
1. T. Andreescu e R. Gelca. Mathematical Olympiad Challenges. Birkhauser, Boston,
2009.
2. A. Caminha. T
opicos de Matem
atica Elementar, Volume 1: N
umeros Reais. Sociedade Brasileira de Matematica, Rio de Janeiro, 2012.
3. A. Engel. Problem Solving Strategies. Springer-Verlag, Nova Iorque, 1998.
4. E. Lozansky e C. Rousseau. Winning Solutions. Springer-Verlag, Nova Iorque, 1996.


POT 2012 - Algebra
- Nvel 3 - Aula 02 - Prof. Antonio Caminha

Dicas e Soluc
oes
1. Comece obsevando que

1+
1

n+1
1
n+1
n
+ n1

1+

1
n+1



1
(n + 1)2

n

Em seguida, aplique a desigualdade de Bernoulli.


m
2. Escreva am = mm 1 + am
e, em seguida, aplique a desigualdade de Bernoulli.
m
Faca o mesmo com an e, por fim, some os resultados.
3. Escreva x41 + x42 + + x41994 = x31 x1 + x32 x2 + + x31994 x1994 e, em seguida,
aplique a desigualdade de Chebychev para obter
1
(x3 + x32 + + x31994 )(x1 + x2 + + x1994 ).
1994 1
Por fim, use as equac
oes do sistema.
x41 + x42 + + x41994

4. Aplique a desigualdade de Chebyshev a cada parcela do somat


orio.
5. Adapte a prova da desigualdade de Chebyshev ao caso em questao.
6. Aplique a desigualdade de Chebychev.
7. Aplique a desigualdade de Chebychev para concluir que a express
ao do primeiro
membro e maior ou igual que


1 3
1
1
1
1
(a + b3 + c3 + d3 )
+
+
+
.
4
b+c+d a+c+d a+b+d a+b+c
Em seguida, aplique o resultado do Exemplo 4, juntamente com o resultado do Exemplo 4 da Aula 1, para concluir que a u
ltima express
ao acima e maior ou igual que

3
a+b+c+d
42
(a + b + c + d)2

=
.
4
3(a + b + c + d)
12
Por fim, observe que a condic
ao do enunciado equivale a (a + c)(b + d) = 1 e aplique
a desigualdade entre as medias.
8. Adapte a sugestao dada ao problema anterior.
9. Supondo, sem perda de generalidade, x y z, use a desigualdade de Chebychev
para concluir que a express
ao do enunciado e maior ou igual que

 3
(x + 1) + (y + 1) + (z + 1)
x + y3 + z3
.

3
(x + 1)(y + 1)(z + 1)
Em seguida, aplique a desigualdade de Chebychev ao primeiro fator e a desigualdade
entre as medias ao segundo fator para concluir que a u
ltima express
ao acima e maior
1
3t3
ou igual que (t+1)2 , onde t = 3 (x+ y + z). Por fim, use a desigualdade entre as medias
para concluir que t 1 e, em seguida, prove que t s
9

3t3
(t+1)2

3s3
.
(s+1)2


POT 2012 - Algebra
- Nvel 3 - Aula 02 - Prof. Antonio Caminha

10. Para obter a primeira desigualdade, aplique a desigualdade de Chebychev ao primeiro


somat
orio, seguida do resultado do Exemplo 4 da Aula 1. Para a segunda desigualdade, utilize a desigualdade entre as medias aritmetica e quadr
atica (cf. Problema
15 da Aula 1).
11. Suponha, sem perda de generalidade, que a b c. Basta aplicar a desigualdade do
rearranjo, observando que a desigualdade a ser provada e equivalente a
a2 bc + ab2 c + abc2 (ab)2 + (bc)2 + (ca)2 .
12. Aplique a desigualdade do rearranjo.
13. Faca a prova por induc
ao sobre n > 1 inteiro. Para o passo de inducao, basta provar
que

 


a2n+1
a2n
a2n
1+
1+
(1 + an+1 )
1+
an+1
a1
a1
ou, equivalentemente, que
a2n+1

1
a2
1
+ a2n
an+1 + n .
a1
an+1
a1

Por fim, observe que tal desigualdade e uma aplicacao imediata da desigualdade do
rearranjo.
14. Faca i = bi /ai , para 1 j n, e conclua que
b1 + + bn a1 + + an a1 (1 1) + a2 (2 1) + + an (n 1) 0.
Em seguida, aplique a desigualdade de Abel para concluir que
n
X
j=1

aj (j 1) a1 min {1 1, 1 + 2 2, . . . , 1 + + n n}.

Por fim, use a condic


ao do enunciado e a desigualdade entre as medias para mostrar
que 1 + + k k.

10

Polos Olmpicos de Treinamento


Curso de lgebra - Nvel 3
Prof. Antonio Caminha

Aula

Fun
c
oes Definidas Implicitamente 1
Esta e a primeira de duas aulas devotadas ao estudo de funcoes definidas implicitamente.
Para saber mais sobre o material aqui discutido, remetemos o leitor ao Captulo 1 de [2]
ou ao Captulo 11 de [3].
Uma func
ao pode ser definida implicitamente por um conjunto de propriedades. Por
exemplo, sendo g(x) = x + 1 e h(x) = x 1, a funcao f : R R dada por f (x) = x2 e tal
que
f (g(x)) = g(x)2 e f (h(x)) = h(x)2 ,
ou seja, ela e tal que
f (x + 1) = (x + 1)2 = x2 + 2x + 1 e f (x + 1) = (x 1)2 = x2 2x + 1.
Da, temos que a func
ao acima satisfaz, para todo x R, a relacao
f (x + 1) f (x 1) = 4x.
Podemos tentar reverter os passos acima, perguntando agora quais s
ao as funcoes f : R R
tais que
f (x + 1) f (x 1) = 4x, x R.
(1)

claro que a func


E
ao f (x) = x2 n
ao e a u
nica, pois, como e facil verificar, para qualquer
constante real c a func
ao fc (x) = x2 + c tambem satisfaz (1).
Como uma func
ao f : R R satisfazendo (1) n
ao est
a dada por seus valores, e sim por
uma relacao que deve satisfazer, dizemos que a funcao est
a definida implicitamente. Note
que, a partir de (1), podemos descobrir outras relacoes que a funcao satisfaz. Por exemplo,
se g : R R e dada por g(x) = x2 , temos
f (g(x) + 1) f (g(x) 1) = 4g(x),
ou ainda
f (x2 + 1) f (x2 1) = 4x2 , x R.

(2)

Assim, qualquer func


ao que satisfizer (1) tambem satisfar
a (2). Entretanto, a relacao (2)
pode n
ao ser muito u
til para ajudar a determinar as funcoes f que satisfazem (1). S
oa


POT 2012 - Algebra
- Nvel 3 - Aula 03 - Prof. Antonio Caminha

experiencia dir
a que relac
oes obtidas a partir de uma relacao inicialmente dada ser
ao u
teis
nesse sentido.
Em geral, um problema interessante e o de encontrar todas as funcoes definidas implicitamente por um certo conjunto de relacoes dadas. Uma vez que n
ao h
a uma teoria geral
a esse respeito, no que segue, veremos alguns exemplos que ilustram um certo n
umero de
tecnicas u
teis no trato de func
oes definidas implicitamente.
Exemplo 1 (Canada). Ache todas as funcoes f : N N, crescentes e tais que f (2) = 2 e
f (mn) = f (m)f (n), para todos m, n N.
Solu
c
ao. De 1 f (1) < f (2) = 2 obtemos f (1) = 1. Agora f (4) = f (2)f (2) = 4 e
f (8) = f (4)f (2) = 8. Suponha pois, por hip
otese de inducao, que f (2k ) = 2k para um
certo natural k. Ent
ao
f (2k+1 ) = f (2k )f (2) = 2k 2 = 2k+1 ,
e segue que f (2n ) = 2n para todo inteiro n
ao negativo n. Portanto, fixado n natural, segue
de f ser crescente que
2n = f (2n ) < f (2n + 1) < < f (2n+1 1) < f (2n+1 ) = 2n+1 .
Mas, uma vez que f (2n + 1), f (2n + 2), . . ., f (2n+1 1) s
ao naturais, a u
nica possibilidade
e termos
f (2n + 1) = 2n + 1, f (2n + 2) = 2n + 2, . . . , f (2n+1 1) = 2n+1 1.
Finalmente, como esse raciocnio e valido para todo n natural, segue que f (m) = m para
todo m natural.
Exemplo 2 (Olimp. Iberoamericana). Se D = R {1, 0, 1}, encontre todas as funcoes
f : D R tais que, para todo x D, tenhamos


1x
= 64x.
f (x)2 f
1+x


Solu
c
ao. Note antes de tudo que, como x 6= 0, temos f (x)2 f 1x
1+x 6= 0 para todo x D.

Em particular, f (x) 6= 0 para todo x D. Seja agora g(x) = 1x


cao
1+x para x D. A defini
de D garante facilmente que g(D) D, de modo que podemos compor f com g. Assim,
para todo x D temos


1 g(x)
2
= 64g(x).
(3)
f (g(x)) f
1 + g(x)
Substituindo a express
ao de g na relac
ao acima, chegamos a
!




1 1x
1x
1x
1+x
f
f
,
= 64
1+x
1+x
1 + 1x
1+x
2


POT 2012 - Algebra
- Nvel 3 - Aula 03 - Prof. Antonio Caminha

ou ainda
f

1x
1+x

2

f (x) = 64

1x
1+x

para todo x D. Elevando ao quadrado ambos os membros da relacao do enunciado e


dividindo o resultado pela relac
ao acima, vem que


3
2 1x
,
f (x) = 64x
1+x
r 

e da f (x) = 4 3 x2 1x
1+x .

Ate este ponto, mostramos apenas que, se f existir, deve ser dada por essa express
ao.
Temos, pois, de verificar que f , assim definida, realmente satisfaz a relacao do enunciado
para todo x D. Mas tal verificac
ao e imediata e ser
a deixada a cargo do leitor.

Ainda em relac
ao ao exemplo anterior, com um pouco mais de pr
atica poderamos
prescindir de definir a func
ao g para em seguida comp
o-la com f a fim de obter (3). Ao
inves disso, poderamos apenas ter dito
Substituindo x por

1x
1+x

na relaca
o do enunciado, obtemos . . . ,

tendo em mente que essa substituica


o e meramente uma composicao de funcoes. Doravante,
sempre que n
ao houver perigo de confus
ao, adotaremos essa simplificacao de linguagem, a
qual ja aparece no exemplo a seguir. Ao le-lo, tente identificar as composicoes que foram
mascaradas por substituic
oes.
Exemplo 3. Encontre todas as func
oes f : R R tais que f (1) = 1 e, para todos x, y R,
tenhamos
(a) f (x + y) = f (x) + f (y).
(b) f (xy) = f (x)f (y).
Solu
c
ao. Seja f uma func
ao satisfazendo as condicoes do enunciado. Fazendo x = y = 0
em (a), obtemos
f (0) = f (0 + 0) = f (0) + f (0) = 2f (0),
donde segue que f (0) = 0. Fazendo y = x em (a), obtemos
f (2x) = f (x + x) = f (x) + f (x) = 2f (x)
para todo x R. Fazendo agora y = 2x em (a), segue que
f (3x) = f (x + 2x) = f (x) + f (2x) = f (x) + 2f (x) = 3f (x)
para todo x R. Repetindo o argumento acima conclumos, por inducao sobre n N, que
f (nx) = nf (x), n N, x R
3

(4)


POT 2012 - Algebra
- Nvel 3 - Aula 03 - Prof. Antonio Caminha

Em particular, fazendo x = 1 em (4), obtemos f (n) = n para todo n N. Fazendo agora


x = n1 em (4), segue que

 

1
1
= nf
,
1 = f (1) = f n
n
n

1
em (4), com m N, fornece
donde f n1 = n1 . Finalmente, x = m


 
n
1
n
1
1
f
= f n
= .
= nf
= n
m
m
m
m
m
Vamos ver o que ocorre com os racionais negativos. Para isso, facamos y = x no item
(a), obtendo
0 = f (0) = f (x + (x)) = f (x) + f (x),
ou ainda
f (x) = f (x), x R.

(5)

Em particular, sendo x < 0 racional, segue de (5) e do fato de ser x um racional positivo
que f (x) = f (x) = (x) = x, e portanto f (x) = x para todo x Q.
Como f (x) = x para todo x Q, desconfiamos que a funcao identidade seja a u
nica
satisfazendo as condic
oes do enunciado. Para confirmar tal suposicao, voltemos nossa
atencao `a condic
ao do item (b). Inicialmente, mostremos que se para um certo x R
tivermos f (x) = 0, ent
ao x = 0. De fato, caso fosse x 6= 0, fazendo y = x1 em (b) teramos
 


1
1
0 = f (x)f
=f x
= f (1) = 1,
x
x
o que e uma contradic
ao. Agora, fazendo y = x 6= 0 em (b), obtemos
f (x2 ) = f (x x) = f (x) f (x) = f (x)2 > 0;

(6)

portanto, se x, y R, com x < y, e a 6= 0 for tal que y x = a2 , entao, aplicando


sucessivamente (a), (5) e (6), obtemos
f (y) f (x) = f (y) + f (x) = f (y x) = f (a2 ) = f (a)2 > 0,
de sorte que f e crescente. Suponha, por fim, que existe a R tal que f (a) < a e tome (cf.
Problema 1.5.2 de [1]) um racional r tal que f (a) < r < a; o car
ater crescente de f fornece
r = f (r) < f (a),
o que e uma contradic
ao. Analogamente, n
ao podemos ter f (a) > a, e a u
nica possibilidade
e f (a) = a. Mas como a R foi escolhido arbitrariamente, devemos ter f (x) = x para
todo x R.
Para o pr
oximo exemplo precisamos da seguinte


POT 2012 - Algebra
- Nvel 3 - Aula 03 - Prof. Antonio Caminha

Defini
c
ao 4. Se X e um conjunto n
ao vazio e f : X X e uma funca
o dada, um elemento
x0 X e dito um ponto fixo de f se f (x0 ) = x0 .
Se I R e um intervalo, uma funcao decrescente f : I I admite no m
aximo um
ponto fixo. De fato, se x1 , x2 I fossem pontos fixos de f , com x1 < x2 , seguiria de f ser
decrescente que
x1 = f (x1 ) > f (x2 ) = x2 ,
uma contradic
ao `
a hip
otese x1 < x2 .
Exemplo 5 (Argentina). Seja f : R R uma funca
o decrescente e tal que f (x + f (x)) =
x + f (x) para todo real x. Prove que f (f (x)) = x para todo real x.
Prova. As hip
oteses sobre f garantem que x + f (x) e ponto fixo de f para todo x R.
Por outro lado, o car
ater decrescente de f garante, de acordo com a discussao anterior, a
existencia de no m
aximo um ponto fixo para f , de sorte que deve existir a R tal que
x+f (x) = a para todo x R, o que e o mesmo que f (x) = ax para todo x R. Portanto
f (f (x)) = f (a x) = a (a x) = x,
para todo x R.

Problemas
1. Encontre todas as func
oes f : Q Q tais que f
x, y Q.

x+y 
2

f (x)+f (y)
,
2

para todos

2. (Austria).
Encontre todas as func
oes f : Z\{0} Q tais que, para todos x, y Z\{0}
para os quais x + y seja m
ultiplo de 3, tenhamos


x+y
f (x) + f (y)
f
.
=
3
2
3. (Vietna). Ache todas as func
oes f : R R tais que
1
1
1
f (xy) + f (xz) f (x)f (yz) ,
2
2
4
para todos x, y, z R.
4. (Espanha). Encontre todas as funcoes crescentes f : N N tais que, para todo
n N, tenhamos f (n + f (n)) = 2f (n).
5. Encontre todas as func
oes f : R Z tais que:
(a) f (x + a) = f (x) + a, para todo x R e todo a Z.
5


POT 2012 - Algebra
- Nvel 3 - Aula 03 - Prof. Antonio Caminha

(b) f (f (x)) = 0 para x [0, 1).

6. (Austria-Pol
onia). Prove que n
ao existe funcao f : Z Z tal que, para todos x, y Z,
tenhamos f (x + f (y)) = f (x) y.
7. (Romenia). Ache todas as func
oes f : Z Z tais que f (0) = 1 e
f (f (k)) + f (k) = 2k + 3
para todo k Z.
8. (Romenia). Sejam k > 1 um inteiro mpar e A = {x1 , x2 , . . . , xk } um conjunto de k
n
umeros reais. Obtenha todas as funcoes injetivas f : A A tais que
|f (x1 ) x1 | = |f (x2 ) x2 | = = |f (xk ) xk |.
9. Ache todas as func
oes f : Q Q+ tais que f (x + y) = f (x)f (y) para todos x, y Q.
10. Ache todas as func
oes f : [0, 1] [0, 1] tais que f (0) = 0, f (1) = 1 e
f (x + y) + f (x y) = 2f (x)
para todos os x, y tais que x y, x + y [0, 1].

Bibliografia
1. A. Caminha. T
opicos de Matem
atica Elementar, Volume 1: N
umeros Reais. Sociedade Brasileira de Matematica, Rio de Janeiro, 2012.
2. A. Caminha. T
opicos de Matem
atica Elementar, Volume 3: Introduca
o a
` An
alise.
Sociedade Brasileira de Matematica, Rio de Janeiro, 2012.
3. A. Engel. Problem Solving Strategies. Springer-Verlag, Nova Iorque, 1998.


POT 2012 - Algebra
- Nvel 3 - Aula 03 - Prof. Antonio Caminha

Dicas e Soluc
oes
1. Fazendo g(x) = f (x) f (0) na relacao do enunciado, conclua que basta considerar

o caso em que f (0) = 0. Sob tal suposicao, faca y = 0 para obter f x2 = f (x)
2 para
todo x Q. Em seguida, use a relacao do enunciado para concluir que f (x + y) =
f (x) + f (y) para todos x, y Q, de sorte que f (x) = f (1)x para todo x Q.
2. Use a relac
ao do enunciado para provar por inducao que f (n) = f (1) para todo n N;
em seguida, considere o caso n < 0.
3. Faca x = y = z = 0 para obter f (0) = 12 ; em seguida, obtenha f (1) = 12 de maneira
an
aloga. Faca y = z = 1 para concluir que f (x) 21 para todo x R. Por fim, utilize
uma substituic
ao an
aloga para concluir que f (x) 21 para todo x R.
4. Em cada um dos intervalos [n, n + f (n)] e [f (n), 2f (n)] h
a f (n) + 1 naturais. Use,
pois, o car
ater crescente de f , juntamente com f (n + f (n)) = 2f (n), para concluir
que f (n + k) = f (n) + k, para todos 1 k f (n). Tome agora k, n naturais, com
n > k. Use que f (n) n para mostrar que f (n) > k e da, pelo que fizemos acima,
que f (n) = n 1 + f (1).
5. Faca x = a = 0 em (a) para concluir que f (0) = 0. Em seguida, fazendo x = 0
em (a), mostre que f (a) = a para todo a Z. Tomando agora x [0, 1), use que
f (x) Z e f (f (x)) = 0 para concluir que f (x) = 0. Por fim, para x R qualquer,
troque x por {x} e faca a = x em (a) para concluir que f (x) = x.
6. Comece calculando f (x + f (y + f (0))) de duas maneiras distintas para concluir que
f (0) = 0. Em seguida, deduza a partir da que f (f (y)) = y para todo y Z, de sorte
que f e bijetiva. A partir dessa u
ltima relacao, calcule f (f (f (x))) de duas maneiras
distintas para concluir que f e mpar. Troque x por f (x) na relacao do enunciado
para concluir que f (f (x) + f (y)) = f (f (x + y)), e da que f (x) + f (y) = f (x + y)
para todos x, y Z. Por fim, use inducao para obter f (x) = f (1)x para todo x Z,
chegando assim `
a contradic
ao 1 = f (f (1)) = f (1)f (1) 0.
7. Faca k = 0 para concluir que f (1) = 2; em seguida, use inducao para mostrar que
f (n) = n + 1 para todo n N. Use agora a relacao do enunciado para mostrar que
se f (1) = a, ent
ao n
ao pode ser a < 0 nem a > 0. Por fim, faca uma nova inducao
para concluir que f (n) = n + 1 para todo n N.
8. Inicialmente, podemos supor que f n
ao tem pontos fixos. Escreva agora A = B C,
onde B e C s
ao conjuntos disjuntos e tais que f (x) > x para todo x B e f (x) < x
para todo x C. Se |B| = l e a e o valor comum de |f (xj ) xj |, conclua que
P
0 = kj=1 (f (xj ) xj ) = (2l k)a e, a partir da, que a = 0, o que e uma contradicao.

9. Faca x = y = 0 para obter f (0) = 1. Em seguida, use inducao para provar que
f (nx) = f (x)n para todos x Q, n N; mostre a partir da que f (x) = f (1)x para
todo x Q+ . Fazendo agora y = x, mostre que f (x) = f (x)1 ; conclua entao
que f (x) = f (1)x para todo x Q. Por fim, use o fato de o contradomnio de f ser
o conjunto dos racionais positivos para concluir que f (1) = 1.
7


POT 2012 - Algebra
- Nvel 3 - Aula 03 - Prof. Antonio Caminha


10. Fazendo x + y = a e x y = b, mostre que f (a) + f (b) = 2f a+b
2 , para todos
a, b [0, 1]. Fazendo x = y, mostre que f (2x) = 2f (x) para todo x [0, 1], e conclua
que f (a) + f (b) = f (a + b), para todos a, b [0, 1]. Mostre a partir da que f (x) = x,
para todo x [0, 1] Q. Para o que falta, imite a sugestao para a passagem de Q a
R, no Exemplo 3.

Polos Olmpicos de Treinamento


Aula

Curso de lgebra - Nvel 3


Prof. Antonio Caminha

Fun
c
oes Definidas Implicitamente 2
Nesta segunda aula sobre func
oes definidas implicitamente, examinamos e propomos
alguns exemplos mais elaborados. Para muitos mais, remetemos o leitor a [3] ou ao Captulo
11 de [4].
Comecamos observando que, por vezes, o que se pede n
ao e obter todas as funcoes definidas implicitamente por certas relac
oes, mas somente deduzir que tais funcoes satisfazem
certas propriedades ou que, ao contr
ario, n
ao existem. Vejamos alguns exemplos.
Exemplo 1. Sejam c e reais positivos dados e Q um quadrado no plano, tambem dado.
Prove que n
ao existe func
ao sobrejetiva f : [0, 1] Q tal que, para todos 0 x, y 1,
tenhamos
|f (x) f (y)| c|x y|+1/2 .
Prova. Suponha que exista uma tal funcao. Sejam k > 1 inteiro e xj = kj , para 0 j k.
Dado um real x [xj , xj+1 ], temos
|f (x) f (xj )| c|x xj |+1/2

c
k+1/2

Portanto, sendo Cj o crculo do plano com centro f (xj ) e raio

c
,
k +1/2

segue que

x [xj , xj+1 ] f (x) Cj .


Entao
Q = f ([0, 1]) C0 C1 Ck1 ,
o que implica (A(F ) denota a
area da figura F )
k1
 X

k1
A(Cj )
1 = A(Q) = A j=0
Cj
j=0

= k

k+1/2

2

c2
= 2 .
k

Mas, como > 0 e k > 1 e arbitrario, chegamos a uma contradicao, uma vez que a relacao
k2 c2 n
ao e verdadeira para k suficientemente grande.


POT 2012 - Algebra
- Nvel 3 - Aula 04 - Prof. Antonio Caminha

Exemplo 2 (Franca). Seja f : N N uma bijecao. Prove que existem naturais a < b < c,
tais que f (a) + f (c) = 2f (b)
Prova. Se g = f 1 , o problema equivale a provarmos a existencia de tres inteiros positivos
x < y < z, em P A e tais que g(x) < g(y) < g(z). Faca x = 1 e seja = g(1). Defina
A = {g1 (1), g 1 (2), . . . , g1 ( 1), g1 ()}.
Se t N \ A, ent
ao g(t) > = g(1) = g(x). Portanto, basta provarmos que existem dois
inteiros y < z em N \ A que formam uma P A com 1 e s
ao tais que g(y) < g(z).
Para o que falta, fixado t N \ A, considere os inteiros
t, 2t 1, 4t 3, 8t 7, . . . , 2k t (2k 1), . . . .
Quaisquer dois elementos consecutivos dessa sequencia de n
umeros formam uma P A com
1, de modo que, se existir k tal que
g(2k t (2k 1)) < g(2k+1 t (2k+1 1)),
nada mais haver
a a fazer. Mas, se este n
ao fosse o caso, a injetividade de g nos daria
g(t) > g(2t 1) > g(4t 3) > g(8t 7) > > g(1),
o que seria um absurdo.
Voltemo-nos, agora, a exemplos mais difceis onde se pede, efetivamente, obter todas
as funcoes que satisfazem certos conjuntos de condicoes.
Exemplo 3 (Ira). Obtenha todas as funcoes f : R R tais que, para todos os x, y reais,
tenhamos
f (f (x + y)) = f (x + y) + f (x)f (y) xy.
Solu
c
ao. Seja f uma func
ao satisfazendo as condicoes do enunciado. Fazendo y = 0,
obtemos
f (f (x)) = f (x) + f (0) f (x), x R.
(1)
Seja Im(f ) a imagem de f e c = f (0). Segue de (1) que
f (z) = (c + 1)z, z Im(f ).
que

(2)

Agora, como f (x + y) Im(f ), para todos x, y R, segue de (2) e da relacao original


(c + 1)f (x + y) = f (x + y) + f (x)f (y) xy

ou, ainda,
c f (x + y) = f (x)f (y) xy, x R.

(3)

Suponha que c 6= 0. Ent


ao 0
/ Im(f ). De fato, caso fosse 0 Im(f ) teramos de (2)
que f (0) = 0, uma contradic
ao. Mas a, pondo x = c, y = c em (3), obteramos
c2 = c f (0) = f (c)f (c) + c2 ,
2


POT 2012 - Algebra
- Nvel 3 - Aula 04 - Prof. Antonio Caminha

donde f (c)f (c) = 0 e, portanto, f (c) = 0 ou f (c) = 0; isto, por sua vez, contradiria a
suposicao de que 0
/ Im(f ). Logo, c = 0.
Segue, pois, de (3), que
f (x)f (y) = xy, x R.
(4)
Como a func
ao identicamente nula n
ao satisfaz as condicoes do enunciado, existe z0
Im(f ) tal que z0 6= 0. Ent
ao, temos a partir de (2) que f (z0 ) = z0 . Assim, fazendo y = z0
em (4), obtemos f (x) z0 = xz0 , para todo x R, donde f (x) = x para todo x R.
Por fim, e imediato verificar que a funcao identidade satisfaz a condicao do enunciado.
Antes de apresentar o pr
oximo exemplo, precisamos enunciar um resultado que se revela
importante tambem em outros contextos. Para tanto, recordamos o leitor de que uma
fun
c
ao polinomial e uma func
ao f : R R da forma
f (x) = an xn + an1 xn1 + + a1 x + a0 ,

(5)

para todo x R, onde n e um inteiro n


ao negativo dado e a0 , . . . , an s
ao n
umeros reais
tambem dados.
Teorema 4. Seja f : R R uma funca
o polinomial. Se a, b e d s
ao n
umeros reais tais
que a < b e (f (a) d)(f (b) d) < 0, ent
ao existe a < c < b tal que f (c) = d.
O teorema acima e conhecido em Matematica como o teorema do valor intermedi
ario. (Abreviamos TVI.) Intuitivamente, sua validade decorre do fato de que, o
gr
afico de uma func
ao polinomial n
ao tem saltos; portanto, se ele passar pelos pontos
(a, f (a)) e (b, f (b)), e d pertencer ao intervalo de extremos f (a) e f (b) (que e o que garante
a condicao (f (a) d)(f (b) d) < 0), entao tal gr
afico deve intersectar a reta y = d ou, o
que e o mesmo, deve existir a < c < b tal que f (c) = d.
O TVI, de fato, pode ser aplicado a uma classe de funcoes bem mais ampla que aquela
das funcoes polinomiais, qual seja, aquela das funcoes contnuas; intuitivamente, o leitor
pode pensar numa func
ao contnua f : I R (I R um intervalo), como uma funcao
cujo gr
afico n
ao admite interrupc
oes. Para uma discuss
ao mais adequada desse conceito,
juntamente com uma prova do TVI para tal classe de funcoes, remetemos o leitor `a Secao
4.2 de [1]. Tambem, uma prova do TVI para funcoes polinomiais, baseada na validade do

teorema fundamental da Algebra,


pode ser encontrada na Secao 5.1 de [2].
Precisamos, ainda, da seguinte consequencia do TVI.
Corol
ario 5. Se f : R R e uma funca
o polinomial como em (5), com n mpar, ent
ao a
imagem de f e todo o conjunto dos n
umeros reais.
Prova. Dado d R, faca g(x) = f (x) d. Entao g : R R tambem e polinomial, e basta
garantirmos a existencia de c R tal que g(c) = 0.
O argumento do par
agrafo anterior reduz nosso problema a mostrar a existencia de
c R tal que f (c) = 0. Para tanto, suponhamos, sem perda de generalidade, que an > 0.


POT 2012 - Algebra
- Nvel 3 - Aula 04 - Prof. Antonio Caminha

Entao, para x 6= 0, repetidas aplicac


oes da desigualdade triangular fornecem
f (x)
xn

an1
a1
a0
+ + n1 + n
x
x
ax
a1
a0
n1
+ + n1 + n
an
x
a x
x
n1
a1 a0
an
n1 n
x
x
x
|an1 |
|a1 |
|a0 |
= an
n1 n .
|x|
|x|
|x|
= an +

Portanto, se |x| 1, ent


ao |x| |x|2 |x|n e, da,
n1

1 X
f (x)
|aj |,
an
n
x
|x|
j=0

Pn1
o qual, por sua vez, e positivo para |x| > a1n j=0
|aj |.
Em resumo, se

n1
1 X

A > max 1,
|aj | ,
an

j=0

mpar, segue que f (A) < 0 < f (A) e o TVI


entao fx(x)
n > 0 para x = A. Mas, como n
garante a existencia de c [A, A] tal que f (c) = 0.
Podemos, por fim, discutir o exemplo a seguir.
Exemplo 6 (Bielorr
ussia). Encontre todas as funcoes f, g, h : R R tais que, para todos
x, y R, tenhamos
f (x + y 3 ) + g(x3 + y) = h(xy).
Solu
c
ao. Fazendo x = y = 0, obtemos f (0) + g(0) = h(0). Assim, definindo f1 (x) =
f (x) f (0), g1 (x) = g(x) g(0) e h1 (x) = h(x) h(0), temos
f1 (x + y 3 ) + g1 (x3 + y) = h1 (xy),
com f1 (0) = g1 (0) = h1 (0) = 0. Podemos, portanto, comecar supondo que f (0) = g(0) =
h(0) = 0.
Fazendo y = x3 na relac
ao do enunciado, obtemos g(x x9 ) = h(x4 ) para todo
3
x. Fazendo x = y , obtemos f (y 9 + y) = h(y 4 ), para todo y R; da, segue que
f (x x9 ) = g(x x9 ). Mas a imagem da funcao polinomial x 7 x x9 e o conjunto dos
n
umeros reais, de modo que f (x) = g(x) para todo x R. A relacao do enunciado se
reduz, entao, a
f (x + y 3 ) + f (x3 + y) = h(xy).
(6)
Fazendo sucessivamente y = x3 e x = y 3 em (6), e levando em conta que f (0) = 0,
obtemos, respectivamente,
f (x x9 ) = h(x4 ) e f (y 9 + y) = h(y 4 ),
4


POT 2012 - Algebra
- Nvel 3 - Aula 04 - Prof. Antonio Caminha

de modo que f (x x9 ) = f (x9 + x). Usando de novo o fato de que a imagem da funcao
polinomial x 7 x x9 e R, segue que f e uma funcao par, i.e., tal que que f (x) = f (x),
para todo x R.
Fazendo, agora, y = 0 em (6), obtemos f (x) + f (x3 ) = 0, donde f (x) = f (x3 ).
Voltando a (6), essa relac
ao nos d
a
f (x3 + y) f ((x + y 3 )3 ) = h(xy).
Mas f par implica, ent
ao, que
f (x3 + y) f ((x + y 3 )3 ) = h(xy).
Segue, da, que, se a R for tal que existam reais x, y satisfazendo o sistema de equacoes

xy = a
,
x3 + y = (x + y 3 )3
entao h(a) = 0. Provemos que esse e o caso para todo a 0.
Se a = 0, ja temos h(a) = 0. Se a 6= 0 entao a segunda equacao do sistema acima
equivale a y 9 + 3y 6 x + 3x2 y 3 + y + 2x3 = 0. Escrevendo x = ya , segue que basta verificarmos
a existencia de um real y tal que
y 12 + 3ay 8 + (3a2 + 1)y 4 + 2a3 = 0.
Para tanto, seja p(y) = y 12 + 3ay 8 + (3a2 + 1)y 4 + 2a3 . Se a < 0, entao p(0) = 2a3 < 0 e
p(a) > 0, de modo que o TVI garante que p tem ao menos uma raiz real.
Para terminarmos, veja que f (x x9 ) = h(x4 ), de modo que f (x x9 ) = 0 para todo
x R. Usando novamente a sobrejetividade de f , segue que f 0. Da, (6) nos d
a h 0.
Entao, as func
oes f, g, h que satisfazem as condicoes do enunciado s
ao as funcoes constantes f f (0), g g(0), h h(0), tais que f (0) + g(0) = h(0).

Problemas
1. (IMO). Seja G um conjunto n
ao vazio de funcoes afins, possuindo as seguintes propriedades:
(a) Se f, g G, ent
ao f g G.

(b) Se f G, ent
ao f 1 G.

(c) Para toda f G, existe xf R tal que f (xf ) = xf .

Prove que existe um real x0 tal que f (x0 ) = x0 , para toda f G.


POT 2012 - Algebra
- Nvel 3 - Aula 04 - Prof. Antonio Caminha

2. (IMO). Seja f : R [0, 1] uma funcao tal que, para um certo a R, tenhamos
f (x + a) =
para todo x R.

1 p
+ f (x) f (x)2 ,
2

(a) Prove que f e peri


odica, i.e., que existe um real p > 0 tal que f (x + p) = f (x),
para todo x R.

(b) Para a = 1, de um exemplo de uma funcao n


ao constante f : R [0, 1] que
satisfaca a relac
ao do enunciado.
3. (Lituania). Seja f : Z Z uma funcao tal que f (m2 + f (n)) = f (m)2 + n para todos
m, n inteiros.
(a) Prove que f (0) = 0 e f (1) = 1.
(b) Ache todas essas func
oes.
4. (OIM). Encontre todas as func
oes f : N N satisfazendo, para todos x, y N, as
duas condic
oes a seguir:
(a) x > y f (x) > f (y).

(b) f (yf (x)) = x2 f (xy)

5. (Bulgaria). Sejam n > 1 inteiro e a1 , a2 , . . . , an reais positivos dados. Prove que a


equacao

1 + a1 x + 1 + a2 x + + 1 + an x = nx
possui exatamente uma soluc
ao real positiva
6. (Romenia adaptado). Existe uma funcao polinomial f : R R tal que
f (x) Q f (x + 1)
/ Q?
7. (IMO). Ache todas as func
oes f : [0, +) R satisfazendo as duas condicoes a
seguir:
(a) f (xf (y))f (y) = f (x + y), para todos x, y [0, +).

(b) f (2) = 0 e f (x) 6= 0 para 0 x < 2.

8. (IMO). Seja S = {x R; x > 1}. Obtenha todas as funcoes f : S S que


satisfacam as duas condic
oes a seguir:
(a) f (x + f (y) + xf (y)) = y + f (x) + yf (x)), para todos x, y S.
(b)

f (x)
x

e crescente em cada um dos intervalos (1, 0) e (0, +).

9. (Polonia). Encontre todas as funcoes f : Q+ Q+ satisfazendo, para todo racional


positivo x, as seguintes condic
oes:
6


POT 2012 - Algebra
- Nvel 3 - Aula 04 - Prof. Antonio Caminha

(a) f (x + 1) = f (x) + 1.
(b) f (x3 ) = f (x)3 .
10. (IMO). Decida se existe uma funcao f : N N satisfazendo as condicoes a seguir:
(a) f (1) = 2.
(b) f (n) < f (n + 1) para todo n N.

(c) f (f (n)) = f (n) + n para todo n N.

Bibliografia
1. A. Caminha. T
opicos de Matem
atica Elementar, Volume 3: Introduca
o a
` An
alise.
Sociedade Brasileira de Matematica, Rio de Janeiro, 2012.
2. A. Caminha. T
opicos de Matem
atica Elementar, Volume 6: Introduca
o a
` An
alise.
Sociedade Brasileira de Matematica, Rio de Janeiro, 2012.
3. D. Djukic, V. Jankovic, I. Matic e N. Petrovic. The IMO Compendium: a Collection
of Problems Suggested for The International Mathematical Olympiads from 19592004. Springer-Verlag, Nova Iorque, 2006.
4. A. Engel. Problem Solving Strategies. Springer-Verlag, Nova Iorque, 1998.


POT 2012 - Algebra
- Nvel 3 - Aula 04 - Prof. Antonio Caminha

Dicas e Soluc
oes
1. Inicialmente, mostre que basta provar que duas funcoes quaisquer em G comutam
em relac
ao `
a operac
ao de composicao de funcoes. Para tanto, use os itens (a) e (b),
juntamente com o fato de que a u
nica funcao h G da forma h(x) = x + a, para
algum a R, e h = IdR .
q
2. Para o item (a), fazendo g(x) = f (x) + 21 , mostre que g(x + a) = 14 g(x)2 e, da,

1
2
que g(x + 2a) = g(x), para todo x R. Quanto a (b), faca h(x) = 4g(x)
 1 2 e
x
2
e uma
conclua que h(x + 1) = h(x); em seguida, mostre que h(x) = sen
2 2
possibilidade.

3. Para o item (a), comece fazendo m = 0 na relacao do enunciado para obter f (f (n)) =
f (0)2 + n, concluindo, assim, que f e bijetiva. Em seguida, tome k Z tal que
f (k) = 0 e seja l = f (0); ent
ao temos l = f (0) = f (f (k)) = f (0)2 + k = l2 + k,
ao passo que, fazendo m = k e n = 0 na relacao do enunciado, temos k2 + l = k.
Logo, k = l = 0. Quanto a (b), faca m = 1 e n = 0 na relacao do enunciado para
obter f (1) = 1; em seguida, deduza que f (f (n) + 1) = n + 1 e, a partir da, que
f (n) = n para todo inteiro n
ao negativo n. Por fim, estenda o argumento aos inteiros
negativos, mostrando que f (n) = n para todo n Z.
4. Fazendo x = y = 1, conclua que f (1) = 1. Em seguida, mostre que f (f (xy)) =
x2 y 2 f (xy) = f (f (x)f (y)), obtendo, entao, que f (xy) = f (x)f (y), para todos x, y N;
conclua a partir da, que f (x2 ) = f (x)2 para todo x N. Por fim, se f (x) < x2 para
um certo x N, escreva f (x)3 = f (x3 ) > f (xf (x)) = x2 f (x)2 para obter f (x) > x2 ,
uma contradic
ao; analogamente, mostre que n
ao pode ser f (x) > x2 , de sorte que
2
f (x) = x para todo x N.
5. Divida ambos os membros da equacao por x e, em seguida, aplique o TVI.
6. Comece observando que a func
ao g : R R, dada por g(x) = f (x + 1) f (x) e
contnua e, pela condic
ao do enunciado, transforma todo n
umero real num irracional.
Agora, use o TVI, juntamente com o fato de que todo intervalo n
ao degenerado
contem n
umeros irracionais, para concluir que g e constante; entao, use isto para
mostrar que f (x + 2) f (x) e sempre irracional. Por fim, mostre que existe x0 R
tal que f (x0 ) Q e, da, que f (x0 + 2) f (x0 ) Q.
7. Troque x por x 2 e y por 2 em (a) para concluir que f (x) = 0 para x 2. Faca,
agora, x = y = 0 em (a) para obter, a partir de (b), que f (0) = 1. Por fim, para
2
; por outro lado,
0 < x < 2, troque y por 2 x em (a) para concluir que f (x) 2x


2
2
mostre que f x + f (x) = f (2)f (x) = 0 e conclua que f (x) 2x para cada um de
tais x.
8. Observe inicialmente que, a partir de (b), f tem no m
aximo um ponto fixo em cada um
dos intervalos (1, 0), (0, +). Se existir um ponto fixo x0 de f tal que 1 < x0 < 0,
faca x = y = x0 em (a) para concluir que x20 + 2x0 tambem e ponto fixo de f , e da
8


POT 2012 - Algebra
- Nvel 3 - Aula 04 - Prof. Antonio Caminha

que x20 + 2x0 = x0 , o que e uma contradicao. Argumente analogamente para concluir
que f n
ao tem pontos fixos em (0, +). Por fim, fazendo x = y em (a), conclua que
x
para todo x S.
f (x) = x+1
9. Observe que, por (a), temos f (x + k) = f (x) + k, para todo k N. Tome, agora,
2
3 n3
m, n N e use que (n +m)
N para calcular
3
n
f

m3 (n3 + m)3 n3
+
n3
n3

de duas maneiras distintas, obtendo a equacao


(a + n2 )3 = a3 + (n6 + 3n3 m + 3m2 ),
onde a = f

m
n

. Conclua, a partir da, que f (x) = x para todo x Q+ .

10. Considere a sequ


encia de Fibonacci (Fn )n1 , dada por F1 = 1, F2 = 1 e Fk+2 =
Fk+1 + Fk , para todo inteiro k 1. Comece mostrando, por inducao, o teorema
de Zeckendorff: todo n
umero natural pode ser unicamente escrito como soma de
n
umeros de Fibonacci de ndices maiores que 1 e n
ao consecutivos. Voltando ao
problema em quest
ao, observe que, se uma tal f existir, entao f (Fk ) = Fk+1 para todo
k N. Agora, utilize o teorema de Zeckendorff para mostrar que uma possibilidade
para f e definirmos
f (Fi1 + Fi2 + + Fij ) = Fi1 +1 + Fi2 +1 + + Fij +1 ,
para todos 1 i1 < i2 < < ij naturais.

Polos Olmpicos de Treinamento


Curso de lgebra - Nvel 3

Aula

Prof. Ccero Thiago / Prof. Marcelo Mendes

N
umeros Complexos
Defini
c
ao 1
O conjunto dos n
umeros complexos, representado por C, consiste de todos os pares ordenados (a, b) com a, b R. Definiremos que (a, b) + (c, d) = (a + c, b + d) e (a, b) (c, d) =
(ac bd, ad + bc). Chamaremos de unidade imagin
aria o complexo i = (0, 1). Usando a
2
operacao de multiplicac
ao verificamos que ii = i = (0, 1)(0, 1) = (0 0 1 1, 0 1 + 1 0) =
(1, 0). Vamos fazer um abuso de linguagem matem
atica e definir que (1, 0) = 1, sendo
assim definiremos os n
umeros complexos como as express
oes da forma z = a + bi, onde a e
2
b s
ao n
umeros reais e i = 1, ou seja,


C = z = a + bi| a, b R, i2 = 1.

O n
umero a e chamado de parte real de z e o n
umero b e chamado de parte imagin
aria
de z. Com isso, fica facil perceber que se um n
umero complexo que tem parte imagin
aria
igual a zero ser
a um n
umero real.

Dois n
umeros complexos s
ao iguais se, e somente se, eles possuem a mesma parte real e a
mesma parte imagin
aria, isto e,
a + bi = c + di a = c e b = d.
A soma e o produto de dois n
umeros complexos z = a + bi e w = c + di s
ao definidos assim:
z + w = (a + c) + (b + d)i e z.w = (ac bd) + (ad + bc)i.
a chamado de conjugado do n
umero complexo z = a+bi.
O n
umero complexo z = abi ser
facil perceber que z e um n
E
umero real se, e somente se, z = z. Usaremos o conjugado do
1
n
umero complexo z = a + bi para poder representar o complexo :
z
1
1
1
a bi
a bi
a
b
z
=
=
.
= 2
= 2 = 2
+ 2
i.
2
2
z
a + bi
a + bi a bi
a +b
|z|
a +b
a + b2
z1
1
= z1 . , onde z1 e z2 s
ao n
umeros complexos.
z2
z2
Vejamos abaixo algumas propriedades as quais deixaremos as provas com o leitor:
A partir do resultado acima faremos


POT 2012 - Algebra
- Nvel 3 - Aula 5 - Prof. Marcelo Mendes/ Prof. Ccero
Thiago
1.
2.
3.
4.

z=z
z+w =z+w
z.w
 z = z.w
z
= .
w
w

O valor absoluto, ou m
odulo de um n
umero complexo z = a + bi e definido por:
p

|z| = a2 + b2 = z.z.

Vejamos agora algumas propriedades do m


odulo de um n
umero complexo:
1. |z| = | z| = |z|
2. z.z = |z|2
3. |z1 .z2 | = |z1 |.|z2 |
1
1
, z 6= 0
4. =
z
|z|

z1 |z1 |
5. =
, z2 6= 0
z2
|z2 |
6. |z1 + z2 | |z1 | + |z2 |
Prova:
Observe que |z1 +z2 |2 = (z1 + z2 ) (z1 + z2 ) = (z1 + z2 ) (z1 + z2 ) = |z1 |2 +z1 z2 +z1 z2 +|z2 |2 .
Por outro lado, z1 z2 = z1 z2 = z1 z2 , assim:
z1 z2 + z1 z2 = 2 Re (z1 z2 ) 2|z1 z2 | = 2|z1 | |z2 |,
portanto, |z1 + z2 |2 (|z1 | + |z2 |)2 . Finalmente, |z1 + z2 | |z1 | + |z2 |.
7. |z1 | |z2 | |z1 z2 | |z1 | + |z2 |.
A demonstrac
ao das outras propriedades fica a cargo do leitor.
Exerccios Resolvidos
1. Resolva a equac
ao z 3 = 18 + 26i, onde z = x + yi e x, y s
ao n
umeros inteiros.
Solucao:
(x+yi)3 = (x+yi)2 (x+yi) = (x2 y 2 +2xyi)(x+yi) = (x3 3xy 2 )+(3x2 y y 3 ) = 18+26i.
Usando a definic
ao de igualdade de n
umeros complexos, obtemos:

x3 3xy 2 = 18
3x2 y y 3 = 26
Fazendo y = tx na igualdade 18(3x2 y y 3 ) = 26(x3 3xy 2 ), observamos que x 6= 0 e y 6= 0
implica 18(3tt3 ) = 26(13t2 ). A u
ltima relacao e equivalente a (3t1)(3t2 12t13) = 0.
1
Au
nica soluc
ao racional da equac
ao e t = , entao,
3
x = 3, y = 1 e z = 3 + i.
2. Prove a identidade
|z1 + z2 |2 + |z1 z2 |2 = 2(|z1 |2 + |z2 |2 )
2


POT 2012 - Algebra
- Nvel 3 - Aula 5 - Prof. Marcelo Mendes/ Prof. Ccero
Thiago

para todos os complexos z1 e z2 .


Solucao:
Usando z.z = |z|2 , temos que
|z1 + z2 |2 + |z1 z2 |2 = (z1 + z2 )(z1 + z2 ) + (z1 z2 )(z1 z2 )
= |z1 |2 + z1 .z2 + z2 .z1 + |z2 |2 + |z1 |2 z1 .z2 z2 .z1 + |z2 |2
2(|z1 |2 + |z2 |2 ).

2
3. (Cro
acia) No conjunto dos n
umeros complexos resolva a equacao x2 a2 4ax1 = 0,
em que a e um n
umero real.
Solucao:
2
x2 a2 4ax 1 = 0
2
x2 + a2 4a2 x2 4ax 1 = 0
2
x2 + a2 (2ax + 1)2 = 0


x2 + a2 2ax 1 x2 + a2 + 2ax + 1 = 0
h
ih
i
(x a)2 1 (x + a)2 + 1 = 0

(x a 1) (x a + 1) (x + a i) (x + a + i) = 0.
x = a + 1, x = a 1, x = a + i e x = a i.
4. (AIME) Sejam w1 , w2 , . . . , wn n
umeros complexos. Uma reta L no plano complexo
e chamada de reta media para os pontos w1 , w2 , . . . , wn se L contem pontos (n
umeros
complexos) z1 , z2 , . . . , zn tais que
n
X
k=1

(zk wk ) = 0.

Para os n
umeros w1 = 32 + 170i, w2 = 7 + 64i, w3 = 9 + 200i, w4 = 1 + 27i e
w5 = 14 + 43i existe uma u
nica reta media que intersecta o eixo y no ponto (0, 3). Determine o coeficiente angular desta reta media.
Solu
c
ao:
Seja y = mx + b uma reta media para os complexos wk = uk + ivk , onde uk e vk , e
k = 1, 2, 3, . . . , n. Assuma que os n
umeros complexos zk = xk + iyk , onde xk e yk s
ao
n
umeros reais escolhidos sobre a reta y = mx + b, assim
n
X
k=1

Entao

xk =

(zk wk ) = 0.

uk ,
3

yk =

vk ,


POT 2012 - Algebra
- Nvel 3 - Aula 5 - Prof. Marcelo Mendes/ Prof. Ccero
Thiago

com yk = mxk + b, 1 k n. Consequentemente,


X 
X
X
uk m + nb.
yk =
(mxk + b) = m
xk + nb =
P
P
Nesse caso, n = 5, b = 3,
uk = 3 e
vk = 504. Segue que 504 = 3m + 15 m = 163.
X

vk =

5. Se a,b e n s
ao n
umeros inteiros e positivos, prove que existem inteiros x e y tais que
n
a2 + b2 = x2 + y 2 .
Solu
c
ao:
n
n
Seja z = a + bi. Ent
ao, a2 + b2 = |z|2 = |z|2n = (|z|n )2 . Mas, z n = x + iy, com x e
y inteiros (pois a e b s
ao inteiros). Portanto, (|z|n )2 = |x + iy|2 = x2 + y 2 .
6. Seja f : C C uma func
ao tal que f (z)f (iz) = z 2 para qualquer z C. Prove que
f (z) + f (z) = 0 para qualquer z C.
Solu
c
ao:
Substitua z por iz na igualdade f (z)f (iz) = z 2 , entao f (iz)f (z) = z 2 . Somando as
duas igualdades temos que f (iz) (f (z) + f (z)) = 0 entao f (iz) = 0 ou f (z) + f (z) = 0.
Da igualdade f (z)f (iz) = z 2 deduzimos que f (z) = 0 se, e somente se, z = 0. Se z 6= 0,
entao f (iz) 6= 0 e, com isso, f (z) + f (z) = 0 e, se z = 0, entao f (z) + f (z) = 2f (0) = 0.
Portanto, f (z) + f (z) = 0 para qualquer
z C. Um exemplo de funcao que satisfaz

!
2
2
f (z)f (iz) = z 2 e f (z) =
z.
+i
2
2
7. Se x e um n
umero real, prove que todos os n
umeros complexos de m
odulo 1 podem ser
escritos na forma
x+i
.
xi
Solu
c
ao:
a
+i

a + bi
2
, e use o fato que
Para comecar seja (a + bi) = (a bi) (a + bi)
= a2 + b2 ab
a bi
i
b
a
a2 + b2 = 1. Faca z = (a + bi)2 e
= t R. Assim, |z| = | (a + bi)2 | = a2 + b2 = 1 e
b
t+i
z=
, t R.
ti
Defini
c
ao 2
Todo n
umero complexo z = a + bi pode ser escrito na forma trigonom
etrica
z = r(cos + i sin ),
onde r = |z| e o
angulo (em radianos) que a reta, que liga a origem ao ponto z, forma
com o eixo positivo real. O
angulo e chamado de argumento de z.


POT 2012 - Algebra
- Nvel 3 - Aula 5 - Prof. Marcelo Mendes/ Prof. Ccero
Thiago
Im
z = a + bi = r(cos + i sin )

Re

O conjugado do complexo z = a + bi, tem a forma trigonometrica


z = r(cos() + i sin()) = r(cos i sin ),
com isso,
r(cos i sin )
1
z
1
= 2 =
= (cos i sin ).
z
|z|
r2
r

Teorema 1
Seja n um inteiro, r e n
umeros reais, entao

[r(cos + i sin )]n = r n (cos n + i sin n) .


Prova:
Vamos provar que a igualdade e valida para n N e, em seguida, provemos para n Z.
Para isso usaremos o princpio da inducao finita.
Se n = 0 z 0 = 1 e r 0 (cos 0 + i sin 0) = 1. Vamos admitir a validade da formula para
n = k 1:
z k1 = r k1 [cos (k 1) + i sin (k 1) ]
e agora provemos a validade da igualdade para n = k:
z k = z k1 z = r k1 [cos (k 1) + i sin (k 1) ] r (cos + i sin ) =
r k (cos k + i sin k)
Fica como exerccio provar que a igualdade e valida para n negativo. Para isso, use que
1
z n = n , n N.
z
Teorema 2
Seja n um inteiro positivo e z um n
umero complexo. Existem n razes n - esimas de z, que
s
ao assim definidas





1
+ 2k
+ 2k
n
+ i sin
.
i = r
cos
n
n
5


POT 2012 - Algebra
- Nvel 3 - Aula 5 - Prof. Marcelo Mendes/ Prof. Ccero
Thiago

para k = 0, 1, 2, . . . , n 1.
Prova:
Para mostrar este resultado, inicialmente e z nas respectivas formas trigonometricas, ou
seja,
= s(cos + i sin ) e z = r(cos + i sin ).
com s 0 e r 0. Com isso,
z = r(cos + i sin ) = n = sn (cos + i sin )n = sn (cos n + i sin n),
resultando,
1

s = r n e n = + 2k, para algum inteiro k.


Com isso, teremos n razes n - esimas de z.
Exerccios Resolvidos
01. Calcule

!100
1
3
+i
.
2
2

Solu
c
ao:

v
u 2
!2
u
1
3
3
1
1
t
= 1, com isso cos = , sin =
, ent
ao |z| =
+
Seja z = + i

2
2
2
2
2

3
.
2


2
2
Portanto, a forma trigonometrica ser
a, z = 1. cos
+ i sin
,
3
3





200
2
2
1
3
200
100
100
+ i sin
+ i sin
.
= 1. cos
= +i
entao z
=1
cos
3
3
3
3
2
2
02. As seis soluc
oes de z 6 = 64 s
ao escritas na forma a + bi, onde a e b s
ao n
umeros reais.
Qual e o produto das soluc
oes com a > 0?
Solu
c
ao:
O teorema 2 implica que as seis razes sextas de
64 = 64(cos + i sin )
s
ao
zk = 64

1
6

cos

+ 2k
6

+ i sin

+ 2k
6



para k = 0, 1, 2, 3, 4 e 5.

Verificando, conclumos que apenas z0 = 3 + i e z5 = 3 i, tem a parte real positiva,


6


POT 2012 - Algebra
- Nvel 3 - Aula 5 - Prof. Marcelo Mendes/ Prof. Ccero
Thiago

entao z0 z5 = 4.
3. Prove que cos

4
6 1
2
+ cos
+ cos
+ = 0.
7
7
7
2

Solu
c
ao:

z+z
Se z = cos + i sin e z = cos i sin . Entao, cos =
. Assim, seja z =
2
2
2
cos
+ i sin
, ent
ao z 7 = 1. Portanto,
7
7






1
1
1
1
1
1
1
2
3
z+
+
z + 2 +
z + 3 + = 0.
2
z
2
z
2
z
2
Multiplicando tudo por 2z 3 e organizando as parcelas, temos:
z6 + z5 + z4 + z3 + z2 + z + 1 = 0
ou seja,
z7 1
= 0.
z1

4. (OPM) As razes quintas do n


umero 1 s
ao as solucoes da equacao x5 1 = 0. Uma
2
2
dessas razes e = cos
+ i sin .
5
5
a) De as demais soluc
oes da equac
ao em funcao de .
b) Calcule a soma dos cubos das 5 razes.
c) Calcule a soma das decimas potencias das 5 razes.
d) Generalize, se possvel, os resultados anteriores para as n razes n - esimas de 1.
Solu
c
ao:
2k
2k
+ i sin
, k = 1, 2, 3, 4, 5 e s
ao, respectiva5
5
mente, iguais a , 2 , 3 , 4 e 5 = 1.
b) S = 3 + 6 + 9 + 12 + 15 = 3 + + 4 + 2 + 1 = 0 pois a soma das cinco razes e o
coeficiente de x4 na equac
ao dada.
10
20
30
40
c) + + + + 50 = 1 + 1 + 1 + 1 + 1 = 5.
d) A soma das potencias k das n razes n - esimas da unidade e igual a 0 se k n
ao divide
n e e igual a n, se k divide n.
Demonstra
c
ao:
Tomando - se uma raiz primitiva z da unidade podemos escrever todas as razes n esimas como potencias de z: z 1 , z 2 , z 3 , . . . , z n . Logo as potencias de grau k ser
ao:
z k , z 2k , z 3k , . . . , z nk .
Supondo que k n
ao divide n, z k 6= 1. Nesse caso a soma das potencias k das n razes ser
a
dada por:

z k z kn 1
.
Sk =
zk 1
a) As solucoes da equac
ao s
ao: cos


POT 2012 - Algebra
- Nvel 3 - Aula 5 - Prof. Marcelo Mendes/ Prof. Ccero
Thiago

Como z n = 1, z kn = 1 e Sk = 0.
Se k e m
ultiplo de n, as potencias de z: z k , z 2k , z 3k ,. . ., z nk ser
ao todas iguais a 1. Logo a
soma ser
a Sk = 1 + 1 + . . . + 1 = n.
5. Seja f (x) = cos x + i sin x.
(a) Prove que f (0) = 1 e f (x)f (y) = f (x + y) para quaisquer x e y reais.
(b) Suponha que a func
ao g satisfaz g(0) = 1 e g(x)g(y) = g(x + y) para quaisquer x e y
reais. Prove que essa func
ao satisfaz:
(i) g(x) 6= 0 para todo x.
g(x)
para quaisquer x e y reais.
(ii) g(x y) =
g(y)
(iii) [g(x)]n = g(nx) para quaisquer x e y reais e n inteiro positivo.
1
(iv) g(x) =
.
g(x)
(c) De exemplo de uma func
ao g tal que g(0) = 1 e que satisfaz g(x)g(y) = g(x + y) para
quaisquer x e y reais.
Solu
c
ao:
(a) Temos que f (0) = cos 0 + i sin 0 = 1 + 0i = 1. Alem disso,
f (x)f (y) = (cos x + i sin x) (cos y + i sin y)
= cos x cos y sin x sin y + i (sin x cos y + cos x sin y)
= cos (x + y) + i sin (x + y)
f (x + y)
(b) Fazendo y = x em g(x)g(y) = g(x + y) temos que g(x)g(x) = g(0) = 1 para todo x.
Portanto, n
ao podemos ter g(x) = 0 para algum x, assim g(x)g(x) 6= 0.
g(x + y)
Como g(x)g(y) = g(x + y), dividindo ambos os lados por g(y) temos que g(x) =
.
g(y)
g(z)
, o que a prova (ii). A
Fazendo x + y =, temos que x = z y, ou seja, g(z y) =
g(y)
n
identidade [g(x)] = g(nx) e uma consequencia imediata de g(x)g(y) = g(x + y). Vamos
usar inducao para provar. Veja que [g(x)]1 = g(x) e se [g(x)]k = g(kx) para algum inteiro
positivo k, ent
ao
[g(x)]k+1 = g(x) [g(x)]k = g(x)g(kx) = g(x + kx) = g((k + 1)x).
Isto mostra que se [g(x)]n = g(nx) ocorre para n = k, entao ocorre tambem para n = k + 1.
g(x)
1
Alem disso, se g(0) = 1 e
= g(x y), fazendo x = 0 temos que
= g(y).
g(y)
g(y)
(c) Um exemplo de func
ao que satisfaz todas as propriedades e g(x) = 2y , pois g(0) = 20 = 1
e
g(x)g(y) = 2x 2y = 2x+y = g(x + y).
8


POT 2012 - Algebra
- Nvel 3 - Aula 5 - Prof. Marcelo Mendes/ Prof. Ccero
Thiago

Isto nos motiva a fazer a seguinte definicao (n


ao ser
a exibida a prova desta afirmacao neste
material):
Para todo x real, temos que
eix = cos x + i sin x.
Fazendo x = , na igualdade acima, encontramos uma maravilhosa igualdade
ei + 1 = 0.
Nas aulas 8 e 9 estudaremos v
arias aplica
co
es das razes da unidade. Deixaremos
a abordagem geom
etrica dos n
umeros complexos para a aula 21 de Geometria.
Exerccios propostos
z1 + z2
e um n
umero real.
1 + z1 z2







3
1
1

2. Seja z C tal que z + 3 2. Prove que z + 2.


z
z

1. Prove que se |z1 | = |z2 | = 1 e z1 z2 6= 1, entao

3. Ache todos os n
umeros complexos z tais que

|z| = 1 e |z 2 + z 2 | = 1.
4. Seja zk = 3k + 2k i, k = 0, 1, 2, . . .. Calcule

zk .

k=0

5. Seja zk = ak +ibk , k = 1, 2, onde ak , bk R. Se z1 = 3z2 e z1 = (1+2i)4 . Ache a2 e b2 .


6. Seja z = x + yi. Se |z 6| = 5 e |z| = 5, ache todos os possveis valores de x e y.
7. Ache todos os n
umeros complexos z tais que
(3z + 1)(4z + 1)(6z + 1)(12z + 1) = 2.
8. Resolva o sistema de equac
oes:
x+

3x y
=3
x2 + y 2

x + 3y
= 0.
x2 + y 2
9


POT 2012 - Algebra
- Nvel 3 - Aula 5 - Prof. Marcelo Mendes/ Prof. Ccero
Thiago

9. (OCM)
umeros complexos tais que |a| < 1 e az 6= 1. Mostre que se
Sejam a e z n

za


ao |z| < 1.
1 az < 1 ent

10. (OCM) Uma lista de n


umeros complexos distintos z1 , z2 , . . . , zn e um ciclo de comprimento n para uma func
ao f : C C se z2 = f (z1 ), z3 = f (z2 ), . . . , zn = f (zn1 )
e z1 = f (zn ). Seja f (z) = z 2 + 2003 e z1 , z2 , . . . , z2003 um ciclo de comprimento
2003. Calcule
2003
Y
(f (zi ) + zi ),
i=1

onde o smbolo

indica o produto.

11. Sejam z1 , z2 C n
umeros complexos tais que |z1 +z2 | =
|z1 z2 |.

3 e |z1 | = |z2 | = 1. Calcule

12. (Putnam) Prove que se 11z 10 + 10iz 9 + 10iz 11 = 0, entao |z| = 1.


13. Seja z um n
umero complexo tal que z C R e
1 + z + z2
R.
1 z + z2
Prove que |z| = 1.
14. (Espanha) Resolva o sistema se equacoes no conjunto dos n
umeros complexos:
|z1 | = |z2 | = |z3 | = 1,
z1 + z2 + z3 = 1,
z1 z2 z3 = 1.
15. Ache todos os n
umeros complexos z tais que |z| = 1 e


z z
+ = 1.
z z

16. Ache todos os pares ordenados (a, b) de n


umeros reais tais que (a + bi)2002 = a bi.
17. Prove que
cos

3
5
7
9
1
+ cos
+ cos
+ cos
+ cos
= .
11
11
11
11
11
2
10


POT 2012 - Algebra
- Nvel 3 - Aula 5 - Prof. Marcelo Mendes/ Prof. Ccero
Thiago

18. Sejam a, b, c n
umeros reais tais que
cos a + cos b + cos c = sin a + sin b + sin c = 0.
Prove que
cos 2a + cos 2b + cos 2c = sin 2a + sin 2b + sin 2c = 0.
19. Prove que
cos
20. Prove que

3
5
1
+ cos
+ cos
= .
7
7
7
2

4
8
7
2
+ sin
+ sin
=
.
sin
7
7
7
2

21. Sejam a, b, c n
umeros reais tais que
cos a + cos b + cos c = sin a + sin b + sin c = 0.
Prove que
1
(cos 3a + cos 3b + cos 3c),
3
1
sin (a + b + c) = (sin 3a + sin 3b + sin 3c).
3

cos (a + b + c) =

1
22. (Cro
acia) Determine os valores mnimo e m
aximo, caso existam, da express
ao |z |,
z
em que z e um n
umero complexo tal que |z| = 2.
23. (OCM) Determinar todos os subconjuntos S dos n
umeros complexos que satisfazem
aos seguintes requisitos:
1. Se x, y S, ent
ao xy S.
2. S possui 2002 elementos.

24. (Mandelbrot) O n
umero complexo

!8
3+i 3
+
2

!8
3i 3
e um n
umero inteiro.
2

Qual?
25. (Mandelbrot) Se 1997 = 1 e 6= 1, entao determine o valor de
1
1
1
+
+ ... +
.
2
1+ 1+
1 + 1997
11


POT 2012 - Algebra
- Nvel 3 - Aula 5 - Prof. Marcelo Mendes/ Prof. Ccero
Thiago

26. (Romenia) Resolva em C, o conjunto dos n


umeros complexos, as seguintes equacoes:
a) |z a| + |z b| = b a.
b) |z| + |z 1| + |z 2| + |z 3| = 4.
z+i
para qualquer complexo z 6= i, e seja zn = F (zn1 ) para
zi
1
todo inteiro positivo n. Dado que z0 =
+ i e z2002 = a + bi, em que a e b s
ao
137
n
umeros reais, determine a + b.

27. (AIME) Seja F (z) =

28. (AIME) Para quantos inteiros positivos n menores ou iguais a 1000


(sin t + i cos t)n = sin nt + i cos nt
para todo real t?
29. (AIME) Uma func
ao f e definida no conjunto dos n
umeros complexos por f (z) =
(a + bi) z em que a e b s
ao n
umeros positivos. Esta funcao tem a propriedade de que
a imagem de cada ponto no plano complexo e equidistante do ponto e da origem.
m
ao inteiros positivos primos entre si.
Dado que |a + bi| = 8 e b2 = , em que m e n s
n
Determine m + n.
3
30. (Cro
acia) Determine
todos os inteiros a, b e c que satisfazem a igualdade (a + bi)
aria.
107i = c, onde i = 1 e a unidade imagin

31. (Cro
acia) Calcule

6
1 + cos + i sin
7
7

14

Sugest
oes / Solu
co
es
1. Use que z R z = z.
1
2. Faca a = |z + |, desenvolva
z

1
z+
z

3

e use a desigualdade triangular.

3. Use que z z = |z|2 .


4. Use progress
oes geometricas.
7. Perceba que
8 (3z + 1) 6 (4z + 1) 4 (6z + 1) 2 (12z + 1) = 768,
(24z + 8) (24z + 6) (24z + 4) (24z + 2) = 768.
12


POT 2012 - Algebra
- Nvel 3 - Aula 5 - Prof. Marcelo Mendes/ Prof. Ccero
Thiago

Faca u = 24z + 5 e w = u2 assim,


(u + 3) (u + 1) (u 1) (u 3) = 768,


u2 1 u2 9 = 768,
w2 10w 759 = 0,

(w 33) (w + 23) = 0.

33 5
23i 5
Portanto, z =
ez=
.
24
24
8. Multiplicando a segunda equac
ao por i e adicionando `a primeira equacao, temos:
x + yi =

(3x y) (x + 3y) i
=3
x2 + y 2

x + yi +

3 (x yi) i (x yi)
2
= 3.
x2 + y 2
x + y2

Seja z = x + yi. Ent


ao,

x yi
1
= 2
.
z
x + y2

Au
ltima equac
ao e equivalente a
z+

3i
=3
z

3 (1 + 2i)
3 + 4i
=
,
2
2
assim, (x, y) = (2, 1) ou (x, y) 
= (1, 1) 

za 2
za
za
9. Use o fato que |
| =
.
1 az
1 az
1 az
z=

2
10. Usando a lei de formac
ao da func
ao conseguimos a seguinte relacao zi2 zi1
= zi+1 zi .

11. Use que z R z = z.


13. Faca z 9 =

11 10iz
e z = a + bi.
11z + 10i

14. Use que z R z = z.

16. Seja z = a + bi, z = a bi, e |z| = a2 + b2. Temos que z 2002 = z. Note que
|z|2002 = |z 2002 | = |z| = |z|, segue que |z| |z|2001 1 = 0. Entao |z| = 0 ou |z| = 1. Se
|z| = 0 teremos apenas uma soluc
ao z = 0. No caso em que |z| = 1, n
os temos z 2002 = z,
2
2003
2003
= 1 tem 2003 solucoes distintas.
que e equivalente a z
= z z = |z| = 1. A equacao z

13


POT 2012 - Algebra
- Nvel 3 - Aula 5 - Prof. Marcelo Mendes/ Prof. Ccero
Thiago

Nos problema 18 e 20 use que se z = cos +isin e z = cos isin . Entao, cos =

z+z
.
2

Nos problemas 19, 21 e 22 use que se z = cos + i sin e z = cos i sin . Entao,
zz
z+z
e sin =
.
cos =
2
2i
22. Faca z = a + bi. Assim,
4 = |z|2 = a2 + b2 .
Portanto,

1
1
a bi 2
|z | = |a + bi
| = |a + bi 2
|
z
a + bi
a + b2
a bi 2
3
5
|a + bi
| = | a + bi|2
4
4
4

9 2 25 2
9
9
25
a + b =
4 b2 + b2 = + b2 .
16
16
16
16
4
9
1
Assim, o menor valor de |z |2 e que e obtido quando b = 0. Entao, o menor valor de
z
4
1
3
2
|z | e . Observe que a = 4 a = 2. Por outro lado, se a2 + b2 = 4, o m
aximo de
z
2
9
1
b2 e 4. Entao, o m
aximo valor de |z |2 e + 4 e e obtido quando b2 = 4 b = 2.
z
4
1
5
Entao, o maior valor de |z | e para b = 2. Observe que nesse caso a2 = 0 a = 0.
z
2
!
!
3+i 3
3i 3
facil ver que C + D = 3 e CD = 3, entao
24. Faca C =
eD=
. E
2
2
2
C 8 + D 8 = C 4 + D 4 2 (CD)4 . Faca o mesmo com C 4 + D 4 .
25. Calcule

1
1
+
, k = 1, 2, . . . , 998.
k
1+
1 + 1997k

26. a) A interpretac
ao geometrica da dist
ancia no plano complexo e a desigualdade triangular mostram que z e um ponto sobre o segmento [a, b].
b) Usando os argumentos do item (a) temos:
|z| + |z 3| |z z + 3| = 3, com igualdade acontecendo se, e somente se, z e um
n
umero real e 0 z 3.
|z 1| + |z 2| |z 1 z + 2| = 1, com igualdade acontecendo se, e somente se, z e
real e 1 z 2. Adicionando as desigualdades obtemos |z| + |z 1| + |z 2| + |z 3| = 4
se, e somente se, z e um n
umero real e 1 z 2.
z+i
+i
1+i z+1
z+i
z + i + iz + i
=

= i
. Agora, e facil ver que
=
27. F (F (z)) = z i
z+i
z + i iz i
1i z1
zi
i
zi
F (F (F (z))) = z, o que mostra que zn = zn3 , n 3. Em particular, z2002 = z20026673 =

14


POT 2012 - Algebra
- Nvel 3 - Aula 5 - Prof. Marcelo Mendes/ Prof. Ccero
Thiago
1
+ 2i
137
= 1 + 274i. Portanto, a + b = 275.
z1 =
1
137
28. Note que
h



in
(sin t + i cos t)n = sin nt + i cos nt = cos
t + i sin
t
=
2
2




t + i n sin
t =
cos n
2
2

 n

 n
nt + i sin
nt .
cos
2
2
e que




sin nt + i cos nt = cos
nt + i sin
nt .
2
2
 n



 n



Tal condicao e equivalente a cos
nt = cos
nt e sin
nt = sin
nt ,
2
2
2
2
n

entao
nt + nt = 2k o que implica que n = 4k + 1. Como 1 n 1000 entao
2
2
0 k 249, ou seja, existem 250 valores de n que satisfazem a igualdade.
29. Como (a + bi) z e equidistante de z e 0, entao | (a + bi) z z| = | (a + bi) z|. Assim,
1
|a 1 + bi| = |a + bi|, ou seja, (a 1)2 + b2 = a2 + b2 . Portanto, a = . Sabemos que
2
255
|a + bi| = 8, ent
ao b =
. Finalmente, m + n = 259.
4

30. (a + bi)3 107i = c a3 3ab2 + 3a2 b b3 107 i = c. Comparando as partes
3
2
2
3
reais e imagin
arias temos
 que a 3ab = c e 3a b b 107 = 0. A segunda equacao e
2
2
equivalente a 3a b b b = 107. Mas 107 e um n
umero primo e a e b inteiros positivos,
segue que:
3a2 b b2 = 107, b = 1
ou
3a2 b b2 = 1, b = 107.
No primeiro caso teremos a = 6 enquanto que no segundo caso n
ao encontraremos um valor
inteiro para a. Portanto, c = 198.

= 2 cos2
e sin
= sin = 2 sin
cos . Entao,
7
14
7
7
14
14


6


cos
.
= 2 cos2
+ i sin
1 + cos + i sin
7
7
14
14
14

31. Temos que 1 + cos

Finalmente,


6
1 + cos + i sin
7
7

14


14
cos
= 2 cos2
+ i sin
14
14
14
15


POT 2012 - Algebra
- Nvel 3 - Aula 5 - Prof. Marcelo Mendes/ Prof. Ccero
Thiago

Bibliografia


14 

14
= 214 cos
cos
+ i sin
14
14
14


14

= 214 cos
(cos + i sin )
14

14
= 214 cos
14

1. T
opicos de Matematica Elementar, vol. 6.
Polin
omios.
Antonio Caminha Muniz Neto
SBM
2. Complex Numbers from A to Z.
Titu Andreescu e Dorin Andrica
Birkhauser
3. 101 Problems in Algebra: From the training of the USA IMO team.
Titu Andreescu
AMT publishing
4. Olimpadas Cearenses de Matematica, Ensino Medio, 1981 - 2005
Emanuel Carneiro, Francisco Ant
onio M. de Paiva e Onofre Campos
5. The USSR Olympiad Problem Book
Selected Problems and Theorems of Elementary Mathematics
D.O. Shklarsky, N.N. Chentzov e I.M. Yaglom
6. First Steps for Math Olympians
Using the American Mathematics Competitions
J. Douglas Faires
7. Problem - Solving Through Problems
Loren C. Larson
Springer
8. Olimpada de Matematica de 1977 a 1997
Questoes e Soluc
oes - 2o grau.
9. Fundamentos de Matematica Elementar, vol. 6
Complexos, Polin
omios e Equac
oes
Gelson Iezzi

16


POT 2012 - Algebra
- Nvel 3 - Aula 5 - Prof. Marcelo Mendes/ Prof. Ccero
Thiago

10. Mathematical Olympiad Treasures


Titu Andreescu e Bogdan Enescu
Birkhauser
11. 360 Problems for Mathematical Contests
Titu Andreescu e Dorin Andrica
GIL
12. The Mandelbrot Problem Book
Sam Vandervelde
13. The First Five Years
Sam Vandervelde
14. Winning Solutions
Cecil Rousseau e Edward Lozansky

15. Curso de Algebra,


Vol. 1.
Abramo Hefez
IMPA
16. Plane Trigonometry and Complex Numbers
Dusan Jevtic
17. A problem book in algebra
V. A. Krechmar
18. Matematica em nvel IME - ITA
N
umeros Complexos e Polin
omios
Caio dos Santos Guimaraes
19. Precalculus
Richard Rusczyk
the Art of Problem Solving
20. A Matematica do ensino medio, vol. 4
Enunciados e soluc
oes dos exerccios
Elon Lages Lima, Paulo Cezar Pinto Carvalho, Eduardo Wagner e Augusto Cesar Morgado
21. A Decade of Berkeley Math Circle
The American Experience, vol. 1
Zvezdelina Stankova e Tom Rike

17

Polos Olmpicos de Treinamento


Curso de lgebra - Nvel 3

Aula

Prof. Ccero Thiago / Prof. Marcelo Mendes

Miscel
anea sobre razes de polin
omios I
Defini
c
ao 1
Um polin
omio na vari
avel x e uma express
ao que pode ser escrita na forma
P (x) = an xn + an1 xn1 + . . . + a1 x + a0 ,
onde n N e ai (i = 0, 1, . . . , n), chamados coeficientes, s
ao n
umeros em algum dos
conjuntos (Z, Q, R, C). O n
umero n ser
a chamado de grau do polin
omio. Chamamos de
coeficiente lder o coeficiente do termo de maior grau, nesse caso an , e chamamos de termo
omio com todos os coeficientes iguais a zero e
independente o coeficiente a0 . Um polin
chamado de polin
omio nulo. Um polin
omio com coeficiente lder igual a 1 e chamado de
polin
omio m
onico.
Defini
c
ao 2
Seja c um n
umero, o n
umero P (c) = an cn + an1 cn1 + . . . + a1 c + a0 e chamado de valor
do polin
omio aplicado ao n
umero c. Se P (c) = 0, dizemos que c e um zero ou raiz do
polin
omio P (x).
Defini
c
ao 3
Dados dois polin
omios P (x) e M (x) 6= 0 , dividir P (x) por M (x) e determinar dois outros
polin
omios Q(x) e R(x) de modo que se verifiquem as duas condicoes seguintes:
(a) P (x) = M (x) Q(x) + R(x).
(b) O grau de R(x) e menor que o grau de M (x) ou R(x) = 0, caso em que a divisao e exata.
Teorema 1. Seja P (x) um polin
omio tal que x a e um fator de P (x), entao P (a) = 0.
Demonstrac
ao. Se x a e um fator de P (x), entao P (x) = (x a) Q(x) para algum
polin
omio Q(x). Fazendo x = a temos que P (a) = (a a) Q(a) = 0 Q(a) = 0, entao a e
uma raiz de P (x).
Teorema 2. Seja P (x) um polin
omio tal que P (a) = 0, entao x a e um fator de P (x).
Demonstrac
ao. Se a e uma raiz de P (x), entao P (a) = 0. Pelo algoritmo da divisao,


POT 2012 - Algebra
- Nvel 3 - Aula 6 - Prof. Ccero Thiago/ Prof. Marcelo
Mendes
temos que o resto quando P (x) e dividido por x a e P (a). Como P (a) = 0 entao o resto
e zero. Isto mostra que x a e um fator de P (x).
Dispositivo de Briot - Ruffini
Dados os polin
omios P (x) = an xn + an1 xn1 + . . . + a1 x + a0 , an 6= 0 e M (x) = x a.
Nosso desejo e determinar o quociente Q(x) e o resto R(x) da divisao de P (x) por M (x).
Seja Q(x) = qn1 xn1 + qn2 xn2 + . . . + q0 , entao:
(qn1 xn1 + qn2 xn2 + . . . + q0 ) (x a) =
qn1 xn + qn2 xn1 + . . . + q0 x aqn1 xn1 aqn2 xn2 . . . aq1 x aq0 =
qn1 xn + (qn2 aqn1 )xn1 + . . . + (q0 aq1 )x aq0

Fazendo P (x) = Q(x) (x a) + R(x), temos:


qn1 = an
qn2 aqn1 = an1 qn2 = aqn1 + an1
..
.
q0 aq1 = a1 q0 = aq1 + a1
R(x) aq0 = a0 R(x) = aq0 + a0
Exemplo
Vamos achar o quociente e o resto da divisao de P (x) = 2x4 7x2 +3x1 por M (x) = x3.
Para isso usaremos o dispositivo de Briot - Ruffini:
0
7
3
1
3 2
2 2 3 + 0 6 3 7 11 3 + 3 36 3 1
3 2 0 7 3 1
2 6 11 36 107
Portanto, Q(x) = 2x3 + 6x2 + 11x + 36 e R(x) = 107.

Teorema 3. (Teorema Fundamental da Algebra)


Todo polin
omio P (x) de grau n 1
possui ao menos uma raiz complexa.
Uma demonstrac
ao desse teorema pode ser encontrada em [1].
Teorema 4. Seja P (x) = an xn + an1 xn1 + . . . + a1 x+ a0 um polin
omio de grau n (n 1)
e an 6= 0, ent
ao
P (x) = an (x x1 )(x x2 ) . . . (x xn ),
em que x1 , x2 , . . . , xn s
ao as razes de P (x).
Para a demonstrac
ao desse teorema use os teoremas 2 e 3.


POT 2012 - Algebra
- Nvel 3 - Aula 6 - Prof. Ccero Thiago/ Prof. Marcelo
Mendes

Teorema 5. Se o polin
omio
P (x) = an xn + an1 xn1 + . . . + a1 x + a0 ,
de grau n, possui n + 1 razes, ent
ao este polin
omio e identicamente igual a 0, ou seja,
an = an1 = . . . = a0 = 0.
Demonstrac
ao. Vamos demonstrar usando inducao sobre n. Para n = 1, a prova e
imediata. Vamos provar que se a afirmacao e verdadeira para n 1, entao tambem ser
a
verdadeira para n. Seja x0 , x1 , . . . , xn s
ao razes de P entao
P (x) = (x xn )Q(x),
com o polin
omio Q(x) tendo grau n1 e n razes distintas x0 , x1 , . . . , xn1 . Pela inducao,
Q(x) e identicamente nulo. Segue que P (x) e tambem nulo.

Teorema 6. (Razes Irracionais)


Seja
t
=
b
+
c
d
e
t
=
b

c
d , em que bi , ci e d
i
i
i
i
i
i

s
ao n
umeros racionais e d e irracional. Entao,
(a) t1 + t2 = t1 + t2 .
(b) t1 t2 = t1 t2 .
(c) se P (x) e um polin
omio com coeficientes racionais tais que t1 e uma raiz de P (x), entao
t1 e tambem uma raiz.

Demonstrac
ao. (a) Temos que t1 + t
2 = (b1 + c1 d) + (b2 + c2 d) = (b
1 + b2 ) + (c1 + c2 )d,
entao t1 + t2 = (b1 + b2 ) (c1 + c2 ) d. Por outro lado, t1 = b1 c1 d e t2 = b2 c2 d,
portanto t1 + t2 = t1 + t2 .
(b) Temos que

t1 t2 = (b1 + c1 d)(b2 + c2 d) =

(b1 b2 + c1 c2 d) + (b1 c2 + b2 c1 ) d =

(b1 b2 + c1 c2 d) (b1 c2 + b2 c1 ) d.

Alem disso, t1 t2 = (b1 c1 d)(b2 c2 d) = (b1 b2 + c1 c2 d) (b1 c2 + b2 c1 ) d. Portanto,


t1 t2 = t1 t2 .
(c) Seja P (x) = an xn + an1 xn1 + . . . + a1 x + a0 . Aplicando (a) e (b), temos que:
P (t1 ) = an (t1 )n + an1 (t1 )n1 + . . . + a1 (t1 ) + a0
= an (t1 )n + an1 (t1 )n1 + . . . + a1 (t1 ) + a0
= an tn1 + an1 t1n1 + . . . + a1 t1 + a0
= P (t1 ) = 0 = 0,

pois t1 e uma raiz de P (x).


POT 2012 - Algebra
- Nvel 3 - Aula 6 - Prof. Ccero Thiago/ Prof. Marcelo
Mendes

Teorema 7. (Razes Racionais) Se P (x) = an xn + an1 xn1 + . . . + a1 x + a0 e um


p
e uma raiz, com p e q inteiros, q 6= 0 e
polin
omio com coeficientes inteiros, tal que
q
mdc(p, q) = 1, ent
ao p|a0 e q|an .
p
e uma raiz de P (x), entao
q
 n
 n1
p
p
p
an
+ an1
+ . . . + a1 + a0 = 0
q
q
q

Demonstrac
ao. Se

an

pn1
p
pn
+
a

+ . . . + a1 + a0 = 0
n1
n
n1
q
q
q

an pn + an1 pn1 q + an2 pn2 q 2 + . . . + a1 pq n1 + a0 q n = 0

an pn = q[an1 pn1 + an2 pn2 q + . . . + a1 pq n2 + a0 q n1 ]


p|a0
pois mdc(p, q) = 1.
De maneira an
aloga, e facil provar que q|an .
Teorema 8. (Razes Complexas) Se P (x) = an xn + an1 xn1 + . . . + a1 x + a0 e um
polin
omio com coeficientes reais tal que z = a + bi e uma raiz, com a e b reais, b 6= 0, entao
z = a bi e tambem uma raiz.
Demonstrac
ao. Se z e uma raiz de P (x) = an xn + an1 xn1 + . . . + a1 x + a0 , entao
P (z) = 0. Assim,
P (z) = an (z)n + an1 (z)n1 + . . . + a1 (z) + a0 =
an z n + an1 z n1 + . . . + a1 z + a0 =
an z n + an1 z n1 + . . . + a1 z + a0 =
an z n + an1 z n1 + . . . + a1 z + a0 =
an z n + an1 z n1 + . . . + a1 z + a0 =
P (z) = 0 = 0.
Teorema 9. (Rela
co
es de Girard) Seja P (x) = an xn + an1 xn1 + . . . + a1 x + a0 um
polin
omio e x1 , x2 , . . . , xn suas razes (reais ou complexas). Entao:
an1
,
an
an2
x1 x2 + x1 x3 + . . . + xn1 xn =
an
x1 + x2 + . . . + xn =


POT 2012 - Algebra
- Nvel 3 - Aula 6 - Prof. Ccero Thiago/ Prof. Marcelo
Mendes

x1 x2 x3 + x1 x2 x4 + . . . + xn2 xn1 xn =

an3
an

..
.
x1 x2 . . . xn = (1)n

a0
an

Exerccios Resolvidos
a b c
1. (Torneio as Cidades) Prove que se a, b e c s
ao n
umeros inteiros e as somas + +
b c a
a c b
e + + s
ao tambem inteiros, entao |a| = |b| = |c|.
c b a
Soluc
ao. Seja
p=

a b c
+ +
b c a

q=

a c b
+ + .
c b a

a b c
, e . Como os coeficientes s
ao inteiros e as
b c a
razes racionais, os u
nicos pssveis valores para as razes s
ao 1. Entao |a| = |b| = |c|.

As razes de x3 px2 + qx 1 = 0 s
ao

2. (OCM) Sejam a, b, c e d as razes (nos complexos) do polin


omio x4 + 6x2 + 4x + 2.
Encontre um polin
omio p(x), do quarto grau, que tenha como razes a2 , b2 , c2 e d2 .
Soluc
ao. Seja Q(x) = x4 + 6x2 + 4x + 2 = (x a)(x b)(x c)(x d). Queremos
encontrar
P (x) = (x a2 )(x b2 )(x c2 )(x d2 ).
Fazendo x = y 2 ,
P (y 2 ) = (y 2 a2 )(y 2 b2 )(y 2 c2 )(y 2 d2 )
= (y + a)(y a)(y + b)(y b)(y + c)(y c)(y + d)(y d)

[(y a)(y b)(y c)(y d)][(y + a)(y + b)(y + c)(y + d)]


Q(y)Q(y).

Assim,
P (y 2 ) = (y 4 + 6y 2 + 4y + 2)(y 4 + 6y 2 4y + 2)
= (y 4 + 6y 2 + 2)2 (4y 2 )2

y 8 + 12y 6 + 40y 4 + 8y 2 + 4.
Voltando para a vari
avel x pela substituicao y 2 = x, temos:
P (x) = x4 + 12x3 + 40x2 + 8x + 4.
5


POT 2012 - Algebra
- Nvel 3 - Aula 6 - Prof. Ccero Thiago/ Prof. Marcelo
Mendes

3. (Torneio das Cidades) Sabendo que a equacao


x4 + ax3 + 2x2 + bx + 1 = 0
possui uma raiz real, prove que
a2 + b2 8.
Soluc
ao. Temos que
x4 + ax3 + 2x2 + bx + 1 = (x2 + px + q)(x2 + sx + t) (1)
em que p, q, s, t s
ao reais. Como, pelo menos uma das razes s
ao reais, iremos
2
assumir que ela e raiz de x + sx + t, entao:
s2 4t.
Igualando os coeficientes em (1), temos:
a = p + s;
2 = q + t + ps;
b = pt + qs;
1 = qt.
Portanto,
a2 + b2 = p2 + q 2 + 2ps + p2 t2 + q 2 s2 + 2ptqs
= p2 (1 + t2 ) + s2 (1 + q 2 ) + 4ps
p2 (1 + t2 ) + 4(t + q + ps)
8.
4. (Bulgaria) Os comprimentos das alturas do ABC s
ao solucoes da equacao c
ubica
x3 + kx2 + lx + m = 0.
Determine o raio do crculo inscrito no ABC.
k
l
l
(b)
(c)
m
k
m
Soluc
ao. (C) Temos que

(a)

(d)

m
k

(e)

m
l

p
a
b
c
1
1
1
1
= =
+
+
=
+
+ .
r
S
2S 2S
2S
ha hb hc
Usando as Relac
oes de Girard, temos:
1
1
1
hb hc + ha hc + ha hb
l
l
+
+
=
=
= .
ha hb hc
ha hb hc
m
m
6


POT 2012 - Algebra
- Nvel 3 - Aula 6 - Prof. Ccero Thiago/ Prof. Marcelo
Mendes

5. (Bulgaria) Determine o n
umero de razes reais da equacao
x1994 x2 + 1 = 0.
(a) 0 (b) 2

(c) 4 (d) 1994

Soluc
ao. (A) Se |x| < 1 ent
ao 1 x2 > 0 e x1994 0 o que implica
x1994 x2 + 1 > 0.
Se |x| 1 ent
ao x1994 x2 = x2 (x1992 1) e, com isso,
x1994 x2 + 1 > 0,
portanto a equac
ao n
ao possui razes reais.
6. (IMTS) Seja f (x) = x4 + 17x3 + 80x2 + 203x + 125.
omio,
Determine
o polin
g(x), de
menor grau possvel, tal que f (3 3) = g(3 3) e f (5 5) = g(5 5).
Soluc
ao. Seja g(x) o polin
omio que desejamos
encontrare h(x) um polin
omio tal

que h(x) = f (x) g(x). Com isso h(3 3) = 0 e h(5 5) = 0. Portanto,

f (x) g(x) = h(x) = a(x)(x 3 3)(x 3 + 3)(x 5 5)(x 5 + 5)


f (x) g(x) = h(x) = a(x)(x4 16x3 + 86x2 180x + 120)
g(x) = f (x) a(x)(x4 16x3 + 86x2 180x + 120)

g(x) = x4 + 17x3 + 80x2 + 203x + 125 a(x)(x4 16x3 + 86x2 180x + 120).
Finalmente, g(x) ter
a grau menor que 4 se, e somente se, a(x) 1. Nesse caso
3
2
g(x) = 33x 6x + 383x + 5.
7. (Austr
alia) Seja P (x) um polin
omio c
ubico com razes r1 , r2 e r3 . Suponha que
 
 
1
1
P
+P
2
2
= 1000.
P (0)
Determine o valor de

1
1
1
+
+
.
r1 r2 r2 r3 r1 r3

Soluc
ao. Seja P (x) = a3 x3 + a2 x2 + a1 x + a0 . Pela relacoes de Girard, temos que
a0
a2
e r1 r2 r3 = . Assim,
r1 + r2 + r3 =
a3
a3
7


POT 2012 - Algebra
- Nvel 3 - Aula 6 - Prof. Ccero Thiago/ Prof. Marcelo
Mendes

1
1
r1 + r2 + r3
a2
1
+
+
=
= .
r1 r2 r2 r3 r1 r3
r1 r2 r3
a0
Mas,
 
a3 a2 a1
1
+
+
+ a0 ,
=
P
2
8
4
2
e


a3 a2 a1
1

+ a0 .
= +
P
2
8
4
2
 
 
1
1
a2
P
+P
+ 2a0
a2
2
2
Portanto, 1000 =
= 2
=
+ 2.
P (0)
a0
2a0


Finalmente,

1
1
a2
1
+
+
=
= 2(1000 2) = 1996.
r1 r2 r2 r3 r1 r3
a0

8. (Austr
alia) Determine todos os polin
omios f com coeficientes reais tais que
(x 27)f (3x) = 27(x 1)f (x)
para todo n
umero real x.
Soluc
ao. Considere a equac
ao
(x 27)f (3x) = 27(x 1)f (x) (1)
Se x = 27 temos que 0 = 0 f (81) = 27 26 f (27), entao f (27) = 0. De maneira
an
aloga, se x = 1 temos que 0 = 27 0 f (1) = 26 f (3) entao f (3) = 0.
Com isso, f (x) = (x 3)(x 27)q(x). Substituindo esse resultado encontrado na
equacao inicial temos que
(x 27)(3x 3)(3x 27)q(3x) = 27(x 1)(x 3)(x 27)q(x).
Para x 6= 1, 27 temos que
(x 9)q(3x) = 3(x 3)q(x). (2)
Agora, se x = 3 temos que 0 = 3 0 q(3) = 6q(9), ou seja, q(9) = 0, assim
q(x) = (x 9)g(x) que substituiremos na equacao (2) obtendo
(x 9)(3x 9)g(3x) = 3(x 3)(x 9)g(x),
8


POT 2012 - Algebra
- Nvel 3 - Aula 6 - Prof. Ccero Thiago/ Prof. Marcelo
Mendes

que para x 6= 1, 3, 9, 27 resulta g(3x) = g(x).


Em particular, se x = 2 ent
ao g(2) = g(6) = g(18) = . . . = g(2 3k ), k. Assim,
g(x) = g(2) possui infinitas razes, o que e impossvel, ou g(x) e uma constante, digamos g(x) = a.
Finalmente, q(x) = a(x 9) e f (x) = a(x 3)(x 9)(x 27), a R.
9. (Austr
alia) Prove que o polin
omio 4x8 2x7 + x6 3x4 + x2 x + 1 n
ao possui razes
reais.
Soluc
ao. Temos que
P (x) = 4x8 2x7 + x6 3x4 + x2 x + 1




2
1 2  3
1 2
P (x) = 3 x4
+ x (x 1) + x
.
2
2

Portanto, P (x) e uma soma de quadrados. Para que P (x) = 0 todos os


rquadrados
1
1
ex=
precisam ser iguais a zero, assim teremos que as duas igualdades x = 4
2
2
devem acontecer simultaneamente, absurdo. Finalmente, P (x) n
ao possui razes reais.
10. (AIME) Sejam a, b, c e d n
umeros reais tais que a equacao x4 + ax3 + bx2 + cx + d = 0
possui quatro razes n
ao reais. O produto
de duas dessas quatro razes e 13 + i e a
soma das outras duas e 3 + 4i em que i = 1. Determine b.
Soluc
ao. Sejam r1 , r2 , r3 e r4 as razes. Se r1 r2 = 13 + i e r3 + r4 = 3 + 4i. Como
o polin
omio possui coeficientes reais e nenhuma das razes s
ao reais, entao r3 = r1 e
r4 = r2 . Segue que r3 r4 = r1 r2 = 13 + i e r1 + r2 = r3 + r4 = 3 4i. Com isso, o
polin
omio ser
a
[x2 (3 4i)x + (13 + i)][x2 (3 + 4i)x + (13 + i)]
= x4 6x3 + 51x2 70x + 170.
Em particular, b = 51 = (13 + i) + (3 4i)(3 + 4i) + (13 i).
Exerccios propostos
1. (Espanha) Sejam a, b, c n
umeros reais. Prove que se x3 + ax2 + bx + c possui tres
2
razes reais, ent
ao 3b a .
9


POT 2012 - Algebra
- Nvel 3 - Aula 6 - Prof. Ccero Thiago/ Prof. Marcelo
Mendes

2. (Espanha) Dado o polin


omio p(x) = x3 + Bx2 + Cx + D, prove que se o quadrado de
uma de suas razes e igual ao produto das outras duas, entao B 3 D = C 3 .
3. Seja f (x) um polin
omio de grau n, n > 1, com coeficientes inteiros e n razes reais,
nem todas iguais, no intervalo (0, 1). Prove que se a e o coeficiente lder de f (x),
entao
|a| 2n + 1.
4. (Czech and Slovak) Seja a e b n
umeros reais. Prove que se a equacao
x4 4x3 + 4x2 + ax + b = 0
possui duas razes reais distintas tais que a soma e igual ao produto, entao a equacao
n
ao possui outras razes reais e, alem disso, a + b > 0.
5. (Canada) O polin
omio
P (x) = xn + a1 xn1 + a2 xn2 + . . . + an1 x + an
com coeficientes inteiros a1 , a2 , . . . , an , e tal que existem quatro inteiros distintos
a, b, c, d tais que
P (a) = P (b) = P (c) = P (d) = 5,
mostre que n
ao existe um inteiro k tal que P (k) = 8.
6. Sejam a e b duas razes do polin
omio x4 + x3 1. Prove que ab e uma raiz do po6
4
3
2
lin
omio x + x + x x 1.
7. (Romenia) Sejam a, b, c, a 6= 0, tais que a e 4a + 3b + 2c tem o mesmo sinal. Mostre
que a equac
ao ax2 + bx + c = 0 n
ao pode ter duas razes no intervalo (1, 2).
8. (OBM) a, b, c, d s
ao n
umeros reais distintos tais que a e b s
ao as razes da equacao
x2 3cx 8d = 0, e c e d s
ao as razes da equacao x2 3ax 8b = 0. Calcule a soma
a + b + c + d.
9. (TST Romenia) Sejam a, n n
umeros inteiros, e p um n
umero primo tal que p > |a|+1.
Prove que o polin
omio f (x) = xn + ax + p n
ao pode ser representado como o produto
de dois polin
omios com coeficientes inteiros.

10


POT 2012 - Algebra
- Nvel 3 - Aula 6 - Prof. Ccero Thiago/ Prof. Marcelo
Mendes
4

10. (Ibero) Ache as razes r1 , r2 , r3 e r4 da equacao 4x ax3 + bx2 cx + 5 = 0, sabendo


que as razes s
ao reais positivas, a, b, c s
ao reais e que
r1 r2 r3 r4
+
+
+
= 1.
2
4
5
8
11. (Espanha) Sejam x1 , x2 as razes do polin
omio P (x) = 3x2 + 3mx + m2 1, sendo
3
m um n
umero real. Prove que P (x1 ) = P (x32 ).
12. (Espanha) Prove que n
ao existem inteiros a, b, c e d tais que o polin
omio P (x) =
3
2
ax + bx + cx + d, a 6= 0 satisfaz P (4) = 1 e P (7) = 2.
13. (OBM) Seja f (x) = x2 +2007x+1. Prove que, para todo n inteiro positivo, a equacao
f (f (. . . (f (x)) . . .)) tem pelo menos uma solucao real.
| {z }
n vezes

14. (TST Brasil) Sejam a, b, c, d n


umeros reais distintos tais que
q

a = 4 + 5 + a,
b=
c=
d=
Determine abcd.

q
q

4
4+
4

5 + b,

5 c,

5 d.

15. Sejam a, b e c lados, com medidas inteiras, de um tri


angulo.
(a) Prove que se a equac
ao
x2 + (a + 1)x + b c = 0
possui razes inteiras, ent
ao o tri
angulo e is
osceles.
(b) Prove que se a equac
ao
x2 + (2ab + 1)x + a2 + b2 c2 = 0
possui razes inteiras, ent
ao o tri
angulo e ret
angulo.

11


POT 2012 - Algebra
- Nvel 3 - Aula 6 - Prof. Ccero Thiago/ Prof. Marcelo
Mendes

(c) Prove que se a equac


ao
x2 + (a2 + b2 + c2 + 1)x + ab + bc + ac = 0
possui razes inteiras, ent
ao o tri
angulo e equilatero.
16. (AIME) Sejam a, b e c as razes de x3 + 3x2 + 4x 11 = 0 e a + b, b + c e c + a as
razes de x3 + rx2 + sx + t = 0. Determine t.
17. (AIME) Considere os polin
omios P (x) = x6 x5 x3 x2 x e Q(x) = x4 x3 x2 1.
Dado que z1 , z2 , z3 e z4 s
ao as razes de Q(x) = 0, ache P (z1 )+P (z2 )+P (z3 )+P (z4 ).
18. (IMO Short List) Sejam a, b, c, d, e e f n
umeros inteiros positivos. Se S =
a + b + c + d + e + f divide abc + def e ab + bc + ca de ef f d. Prove que
S e composto.
19. (Iugosl
avia) Ache todos os racionais positivos a b c tais que os n
umeros
a + b + c,

1 1 1
+ + , abc
a b
c

sejam todos inteiros.


20. (Austr
alia) Os polin
omios x2 + x e x2 + 2 s
ao escritos em um quadro. Beatriz deve
escrever no mesmo quadro a soma, a diferenca ou o produto de quaisquer dois polin
omios que estiverem escritos no quadro. Se Beatriz repetir o processo quantas
vezes quiser, em algum momento ela conseguira chegar ao polin
omio x?
21. (Czech and Slovak) Determine todos os n
umeros reais s tais que
4x4 20x3 + sx2 + 22x 2 = 0
possui quatro razes reais e distintas tais que o produto de duas dessas razes seja 2.
22. (Romenia) Sejam a, b, c e d n
umeros reais tais que f : R R, f (x) = ax3 +bx2 +cx+d
e f (2) + f (5) < 7 < f (3) + f (4). Prove que existem u, v R tais que u + v = 7 e
f (u) + f (v) = 7.

12


POT 2012 - Algebra
- Nvel 3 - Aula 6 - Prof. Ccero Thiago/ Prof. Marcelo
Mendes

23. (Romenia) Seja P (x) = a1998 x1998 + a1997 x1997 + . . . + a1 x + a0 um polin


omio com
coeficientes reais tal que P (0) 6= P (1), e sejam a, b n
umeros reais. Seja Q(x) =
b1998 x1998 + b1997 x1997 + . . . + b1 x + b0 um polin
omio com coeficientes reais tal que
bk = aak + b, k = 0, 1, . . . , 1998. Prove que se Q(0) = Q(1) 6= 0, entao o
polin
omio Q n
ao possui razes reais.
24. Determine todos os polin
omios satisfazendo a equacao polinomial (x + 1)P (x) =
(x 10)P (x + 1).
25. Determine todos os polin
omios P (x) com coeficientes reais para os quais existe um
inteiro positivo n tal que para todo x,




1
1
P x+
+P x
= 2P (x).
n
n
Solu
co
es

1. Sem perda de generalidade sejam as razes, entao:


x3 +ax2 +bx+c = (x)(x)(x) = x3 +( )x2 +( + +)x.
Temos que a = e b = + + . Portanto,
a2 3b = ( )2 3( + + ) = 2 + 2 + 2
=

1
[( )2 + ( )2 ) + ( )2 0.
2

2. Sejam r, s, t as razes, ent


ao 0 polin
omio pode ser escrito da seguinte forma
p(x) = (x r)(x s)(x t) = x3 (r + s + t)x2 + (rs + st + tr)x rst.
Igualando os coeficientes, temos:

r + s +
rs + st +

t = B
tr = C
rst = D

Como r 2 = st, temos que



C = rs
+ r 2 + tr = r(r + s + t) = rB
D = rst = r 3
Finalmente,
C 3 = (rB)3 = r 3 B 3 = B 3 D.
13


POT 2012 - Algebra
- Nvel 3 - Aula 6 - Prof. Ccero Thiago/ Prof. Marcelo
Mendes

3. Seja f (x) = a(x x1 )(x x2 ) . . . (x xn ), em que x1 , x2 , . . . , xn s


ao as razes reais,
nem todas iguais, pertencentes ao intervalo (0, 1). Como os coeficientes s
ao inteiros
entao f (0) e f (1) s
ao n
umeros inteiros. Assim,
f (0) = a(0 x1 )(0 x2 ) . . . (0 xn ) = a(1)n x1 x2 . . . xn
e
f (1) = a(1 x1 )(1 x2 ) . . . (1 xn ).
Por outro lado, |f (0) f (1)| = |a2 (1)n x1 (1 x1 )x2 (1 x2 ) . . . xn (1 xn )| = a2 x1 (1
x1 )x2 (1 x2 ) . . . xn (1 xn ) 1, pois f (0) e f (1) s
ao n
umeros inteiros. Mas, se
1
1
0 < x < 1, ent
ao x(1 x) , com igualdade acontecendo se, e somente se, x =
4
4
(MA MG).
Como as razes n
ao s
ao todas iguais,
ao teremos igualdade acontecendo em x1 (1
 n
n
1
x1 )x2 (1 x2 ) . . . xn (1 xn ) a2
, portanto:
4
 n
1

1 a2 x1 (1 x1 )x2 (1 x2 ) . . . xn (1 xn ) < a2
4
22n < a2 |a| 2n + 1,
pois a e um n
umero inteiro.
4. Sejam x1 e x2 razes reais e distintas de x4 4x3 + 4x2 + ax + b = 0 tais que
x1 + x2 = x1 x2 = p, ent
ao:
x4 4x3 + 4x2 + ax + b = (x2 px + p)(x2 + rx + s), (1)
em que r e s s
ao n
umeros reais. A igualdade (1) garante que:
4 = p + r, (2)
4 = p + s pr, (3)
a = ps + pr, (4)
b = ps. (5)
Da equac
ao (2) temos que r = p 4, (6). Substituindo (6) em (3) temos
s = 4 p + p(p 4) = (p 4)(p 1). (7)

14


POT 2012 - Algebra
- Nvel 3 - Aula 6 - Prof. Ccero Thiago/ Prof. Marcelo
Mendes

Mas a equac
ao quadr
atica x2 px + p = 0 possui razes reais e distintas x1 e x2 , com
isso seu discriminante e positivo, ou seja
p2 4p > 0. (8)
Adicionando as equac
oes (4) e (5) e substituindo r em (6), temos
a + b = pr = p(p 4) = p2 4p > 0.
Seja D o discriminante da equac
ao
x2 + rx + s = 0.
Das equac
oes (6), (7) e (8) temos
D = r 2 4s = (p 4)2 4(p 4)(p 1) = 3p(p 4) = 3(p2 4p) < 0.
Portanto, a equac
ao possui apenas duas razes reais.
5. Seja M (x) = P (x) 5 temos que:
M (x) = (x a)(x b)(x c)(x d)Q(x)
e o polin
omio Q(x) com coeficientes inteiros. Vamos admitir a existencia de um
inteiro k tal que P (k) = 8 M (k) = 3. Dessa forma,
3 = (k a)(k b)(k c)(k d)Q(k).
Mas essa u
ltima igualdade n
ao pode existir pois 3 n
ao pode ser escrito como o produto de pelo menos 4 inteiros distintos. Portanto, n
ao existe tal k.
6. Sejam c e d as outras duas razes de x4 + x3 1. Pelas relacoes de Girard, temos:
a + b + c + d = 1,
ab + ac + ad + bc + bd + cd = 0,
abc + abd + acd + bcd = 0,
abcd = 1.
Fazendo m = a + b, n = c + d, r = ab e s = cd, entao
m + n = 1, (1)
r + s + mn = 0, (2)
15


POT 2012 - Algebra
- Nvel 3 - Aula 6 - Prof. Ccero Thiago/ Prof. Marcelo
Mendes

rn + sm = 0, (3)
rs = 1. (4)
Fazendo s =

1
e n = 1 m em (2) e (3), respectivamente, temos
r
r

1
m2 m = 0, (5)
r

e
r(1 m)
De (6) encontramos m =

m
= 0. (6)
r

r2
. Substituindo em (5), temos
r2 + 1

r4
r2
1
2
=0
+
r (r + 1)2 r 2 + 1
r 6 + r 4 + r 3 r 2 1 = 0,

ou seja, r = ab e uma raiz de x6 + x4 + x3 x2 1.


7. Temos que
0

b
c
4a + 3b + c
= 4+3 +2 = 2x1 x2 3(x1 +x2 )+4 = (x1 1)(x2 2)+(x1 2)(x2 1).
a
a
a

Se x1 e x2 pertencerem ao intervalo (1, 2), entao cada termo da soma acima ser
a
estritamente negativo, o que e uma contradicao.
facil perceber que a + b = 3c e que c + d = 3a. Somando e subtraindo membro a
8. E
membro as duas igualdades obteremos b + d = 2 (a + c) e b d = 4 (c a). Como a
e raiz de x2 3cx 8d = 0, segue que
a2 3ac 8d = 0 (1).
Do mesmo modo, como c e raiz de x2 3ax 8b = 0, temos que
c2 3ac 8d = 0 (2).
Subtraindo as igualdades (1) e (2) e utilizando as relacoes anteriormente obtidas,
vem:
a2 c2 = 8(d b) (a c)(a + c) = 8 4(a c). Como a c 6= 0, conclumos que
a + c = 32.
Portanto, a + c = 32 e b + d = 2(a + c) = 64, donde a + b + c + d = 96.

16


POT 2012 - Algebra
- Nvel 3 - Aula 6 - Prof. Ccero Thiago/ Prof. Marcelo
Mendes

9. Seja z uma raiz complexa do polin


omio. Vamos provar que |z| > 1. Suponha que
|z| 1, ent
ao z n + az = p, ent
ao:
p = |z n + az| = |z||z n1 + a| |z n1 | + |a| 1 + |a|,
contrariando o fato que p > |a| + 1. Agora, seja f (x) = g(x)h(x) uma decomposicao
de f (x) em polin
omios com coeficientes inteiros entao p = g(0)h(0), entao |g(0)| = 1
ou |h(0)| = 1. Suponha que |g(0)| = 1. Se z1 , z2 , . . . , zk s
ao razes de g(x) entao
s
ao tambem razes de f (x), assim:
1 = |g(0)| = |z1 z2 . . . zk | = |z1 ||z2 | . . . |zk | >,
que e uma contradic
ao.
10. Sejam r1 , r2 , r3 e r4 as razes da equacao, entao:
5
r1 r2 r3 r4 = .
4
Usando a desigualdade entre as medias aritmetica e geometrica, temos que:
r1 r2 r3 r4
r
+
+
+
2
4
5
8 4 r1 r2 r3 r4
4
2 4 5 8
1
1
,
4
4
ou seja, aconteceu a igualdade entre as medias, portanto:
r1
r2
r3
r4
1
=
=
=
=
2
4
5
8
4
5
1
r1 = , r2 = 1, r3 = , r4 = 2.
2
4
11. Temos que x1 + x2 = m e x1 x2 =

m2 1
e
3

P (x31 ) P (x32 ) = 3x61 + 3mx31 + m2 1 (3x62 + 3mx32 + m2 1)


= 3(x61 x62 ) + 3m(x31 x32 )

= 3(x31 + x32 )(x31 x32 ) + 3m(x31 x32 )


3(x31 x32 )(x31 + x32 + m).

Mas x31 + x32 = (x1 + x2 )3 3x1 x2 (x1 + x2 ) = (m)3 3


x31 + x32 + m = 0 P (x31 ) = P (x32 ).

17

m2 1
(m) = m. Entao,
3


POT 2012 - Algebra
- Nvel 3 - Aula 6 - Prof. Ccero Thiago/ Prof. Marcelo
Mendes

12. Vamos admitir a existencia do polin


omio. Pelo teorema do resto P (x) = (x
4)Q(x) + 1, sendo Q(x) um polin
omio de grau 2 com coeficientes inteiros. Entao
1
P (7) = 2 = (7 4)Q(7) + 1 Q(7) = , que n
ao e inteiro, contrariando o fato que
3
Q(x) possui coeficientes inteiros.
13. Sejam f 1 (x) =
f (x) e para cada n 1, f n+1 (x) = f (f n (x)). Sejam 1 = 20072 4,
2007 + 1
. Temos f (x1 = 0. Vamos mostrar por inducao que existe uma
x1 =
2

2007 + n
, temos
sequencia de reais positivos (n ) tal que, definindo xn =
2
n+1
n
f (xn+1 ) = xn , para todo n, donde f
(xn+1
) = f (xn ) = 0. Para isso, note que a
2007 + n+1
maior raiz de x2 + 2007x+ 1 = xn e
, onde n+1 = 20072 4+ 4xn =
2

20072 4018 + 2 n > 0.


14.

4+ 5+a

a2 = 4 + 5 + a

a2 4 = 5 + a

a=

a4 8a2 a + 11 = 0.
Analogamente,
b4 8b2 b + 11 = 0,

c4 8c2 + c + 11 = 0,

d4 8d2 + d + 11 = 0.
Seja f (x) = x4 8x2 x + 11 e g(x) = x4 8x2 + x + 11. Entao a e b s
ao razes de
4
2
f (x) = 0, e c e d s
ao razes de g(x) = 0. Mas f (x) = x 8x + x + 11 = g(x).
Entao, as razes de f (x) = 0 s
ao a, b, c e d. Portanto, pelas relacoes de Girard
temos que ab(c)(d) = abcd = 11.
15. (a) Se b > c ent
ao (a + 1)2 4(b c) e um quadrado perfeito menor e de mesma
paridade que (a + 1)2 . Portanto,
(a + 1)2 4(b c) (a 1)2
assim a + c b, contrariando a desigualdade triangular. O caso b < c e an
alogo.

18


POT 2012 - Algebra
- Nvel 3 - Aula 6 - Prof. Ccero Thiago/ Prof. Marcelo
Mendes

(b) Se a2 + b2 > c2 ent


ao (2ab + 1)2 4(a2 + b2 c2 ) e o quadrado de um n
umero
mpar menor que 2ab + 1. Portanto,
(2ab + 1)2 4(a2 + b2 c2 ) (2ab 1)2 ,
assim c2 (a b)2 , contradic
ao. O caso c2 > a2 + b2 e an
alogo.
(c) Se a equac
ao tem razes inteiras, entao (a2 + b2 + c2 + 1)2 4(ab + bc + ac) e um
quadrado perfeito menor e de mesma paridade que (a2 + b2 + c2 + 1)2 . Portanto,
(a2 + b2 + c2 + 1)2 4(ab + bc + ac) (a2 + b2 + c2 + 1)2
(a b)2 + (b c)2 + (c a)2 0
a = b = c.

16. A primeira equac


ao garante que a + b + c = 3. A segunda equacao garante
que t = (a + b)(b + c)(c + a). Segue que t = (3 c)(3 a)(3 b), ou seja,
t = 27 + 9(a + b + c) + 3(ab + ac + bc) + abc. A primeira equacao garante que
ab + bc + ca = 4 e abc = 11, ent
ao t = 27 27 + 12 + 11 = 23.
17. Usando o algoritmo da divis
ao temos que P (x) = Q(x)(x2 + 1) + x2 x + 1. Portanto,
!2
4
4
4
4
4
X
X
X
X
X
X
2
zi + 4.
zi zj
zi 2
zi + 4 =
zi
P (zi ) =
i=1

i=1

i=1

i=1

Usando as relac
oes de Girard, temos que

4
X
i=1

i<j

i=1

P (zi ) = 1 + 2 1 + 4 = 6.

18. Todos os coeficientes do polin


omio
f (x) = (x + a)(x + b)(x + c) (x d)(x e)(x f )
= Sx2 + (ab + bc + ca de ef f d)x + (abc + def )

s
ao m
ultiplos de S. Ent
ao, f (d) = (a + d)(b + d)(c + d) e um m
ultiplo de S. Isto
implica que S e composto pois a + d, b + d e c + d s
ao menores que S.
19. Seja P (x) = (x a)(x b)(x c) = x3 (a + b + c)x2 + (ab+ bc + ca)x abc
1 1 1
um polin
omio cujas razes s
ao a, b e c. Mas ab + bc + ca = abc
+ +
Z,
a b
c
portanto P (x) possui coeficientes inteiros. Pelo teorema das razes racionais temos
que a, b e c s
ao inteiros. Sabemos que a, b e c s
ao positivos entao nosso problema
1
1
1
ao:
e achar todos os valores de a, b e c tais que + + e natural. As solucoes s
a
b
c
(a, b, c) = (1, 1, 1); (1, 2, 2); (3, 3, 3); (2, 4, 4); (2, 3, 6).

19


POT 2012 - Algebra
- Nvel 3 - Aula 6 - Prof. Ccero Thiago/ Prof. Marcelo
Mendes
facil ver que P (2) = Q(2) = 6. Suponha
20. Sejam P (x) = x2 + x e Q(x) = x2 + 2. E
que A(x) e B(x) s
ao dois polin
omios quaisquer tais que A(2) e B(2) s
ao divisveis
por 6. Segue que A(2) + B(2), A(2) B(2) e A(2) B(2) s
ao todos divisveis por 6.
Portanto, qualquer polin
omio R(x) gerado tem R(2) divisvel por 6. Como T (x) =
tem T (2) = 2 ent
ao T (x) = x nunca aparecera no quadro.
21. Sejam x1 , x2 , x3 e x4 as razes tais que x1 x2 = 2, (0). Pelas relacoes de Girard
temos que:
x1 + x2 + x3 + x4 = 5, (1)
s
x1 x2 + x1 x3 + x1 x4 + x2 x3 + x2 x4 + x3 x4 = , (2)
4
11
x1 x2 x3 + x1 x2 x4 + x1 x3 x4 + x2 x3 x4 = , (3)
2
1
x1 x2 x3 x4 = . (4)
2
Das equac
oes (0) e (4) temos que
1
x3 x4 = .
4
Fatorando a equac
ao (3) e, em seguida, substituindo os valores de x1 x2 e x3 x4 temos:
(x1 + x2 )x3 x4 + (x3 + x4 )x1 x2 =

11

11
1
(x1 + x2 ) 2(x3 + x4 ) = .
4
2
Au
ltima equac
ao e (1) garantem que x1 + x2 = 2 e x3 + x4 = 3. Fatorando a equacao
(2) e substituindo os valores ja encontrados temos que
x1 x2 + (x1 + x2 )(x3 + x4 ) + x3 x4 =

As razes do polin
omio s
ao x1,2

s = 17.

3
= 1 3 e x3,4 = 2.
2

22. Precisamos provar que f (x) + f (7 x) = 7 possui solucao. Seja g(x) = f (x) + f (7
facil ver que g e um polin
x) 7. E
omio com grau no m
aximo 2. Temos que g(2) < 0
e g(3) > 0. Segue que g(x) = 0 possui pelo menos uma raiz real.

20


POT 2012 - Algebra
- Nvel 3 - Aula 6 - Prof. Ccero Thiago/ Prof. Marcelo
Mendes
facil ver que Q(x) = aP (x) + b(x1998 + x1997 + . . . + 1). De Q(0) = Q(1) temos
23. E
que a(P (0) P (1)) = 0, ent
ao a = 0. Com isso, Q(x) = b(x1998 + x1997 + . . . + 1,
facil ver que Q(x) n
com b 6= 0 pois Q(0) 6= 0. E
ao possui razes positivas. Para x 1
temos
x1998 + x1997 + . . . + 1 = x1997 (x + 1) + x1995 (x + 1) + . . . + x(x + 1) + 1 1,
e para x (1, 0),
x1998 + x1997 + . . . + 1 = x1998 + x1996 (x + 1) + . . . + x2 (x + 1) + x + 1 > 0.
Conclusao: Q(x) n
ao possui razes reais.
24. Temos que (x + 1)P (x) = (x 10)P (x + 1) mostra que P (x) e divisvel por (x 10).
Trocando x + 1 por x na igualdade inicial temos que xP (x 1) = (x 11)P (x), ou
seja, P (x) e divisvel por x. Ent
ao P (x) = x(x 10)P1 (x). Substituindo na equacao
original e cancelando os termos temos que
xP1 (x) = (x 9)P1 (x + 1).
Repetindo o argumento encontramos P1 (x) = (x 1)(x 9)P2 (x) e P (x) = x(x
1)(x 2) . . . (x 10)Q(x), em que Q(x) = Q(x + 1). Segue que Q(x) e constante, e
a soluc
ao do problema e
P (x) = ax(x 1)(x 2) . . . (x 10),
em que a e uma constante arbitraria.
25. Seja m o grau do polin
omio P (x), entao
P (x) = am xm + am1 xm1 + . . . + a0 .
Usando bin
omio de Newton para

1
x
n

m

1
x
n

2am xm + 2am1 xm1 + 2am2 xm2 + am

m1

temos que

m(m 1) m2
x
+ Q(x)
n2

= 2am xm + 2am1 xm1 + 2am2 xm2 + R(x),


em que Q e R s
ao polin
omios de grau no m
aximo m 3. Comparando os coeficientes
m(m 1)
encontramos am
= 0. Mas am 6= 0, pois e o coeficiente lder do polin
omio,
n2
entao m(m 1) = 0, ou seja, m = 0 ou m = 1. Uma simples verificacao garante que
todos os polin
omios de grau 0 ou 1 satisfazem a condicao inicial.

21


POT 2012 - Algebra
- Nvel 3 - Aula 6 - Prof. Ccero Thiago/ Prof. Marcelo
Mendes

Bibliografia
1. T
opicos de Matematica Elementar, vol. 6
Polin
omios
Antonio Caminha Muniz Neto
SBM
2. Intermediate Algebra
the Art of Problem Solving
Richard Rusczyk e Mathew Crawford
3. Equations and Inequalities
Elementary Problems and Theorems in Algebra and Number Theory
Jiri Herman, Radan Kucera e Jaromir Simsa
4. Fundamentos de Matematica Elementar, vol. 6
Complexos, Polin
omios e Equac
oes
Gelson Iezzi
5. Equations and Inequalities
MIR Publishers Moscow
V. V. Vavilov, I. I. Melnikov, S. N. Olekhnik e P. I. Pasichenko
6. The Ussr Olympiad Problem Book
Selected Problems and Theorems of Elementary Mathematics
D. O. Shklarsky, N. N. Chentzov e I. M. Yaglom
7. Mathematical Olympiad Treasures
Titu Andreescu e Bogdan Enescu
8. Lecture Notes on Mathematical Olympiad Courses
For Junior Section, vol. 1
Xu Jiagu
9. Lecture Notes on Mathematical Olympiad Courses
For Junior Section, vol. 2
Xu Jiagu
10. Problem - Solving Through Problems
Loren C. Larson
11. Kvant Selecta: Algebra and Analysis, I
Serge Tabachnikov
12. Kvant Selecta: Algebra and Analysis, II
22


POT 2012 - Algebra
- Nvel 3 - Aula 6 - Prof. Ccero Thiago/ Prof. Marcelo
Mendes

Serge Tabachnikov
13. Bulgarian Mathematics Competition, 1999 - 2001
BJ Lazarov, JB Tabov, PJ Taylor e AM Storozhev
AMT
14. Australian Mathematical Olympiads, 1996 - 2011
H Lausch, A Di Pasquale, DC Hunt e PJ Taylor
AMT
15. Mathematical Competitions - Baltic Way - 1990 - 2005
16. Olimpadas Cearenses de Matematica, Ensino Medio, 1981 - 2005
Emanuel Carneiro, Francisco Ant
onio M. de Paiva e Onofre Campos
17. 101 Problems in Algebra: From the training of the USA IMO team.
Titu Andreescu
AMT publishing
18. Winning Solutions
Cecil Rousseau e Edward Lozansky
19. Mathematical Olympiad Treasures
Titu Andreescu e Bogdan Enescu
Birkhauser
20. Tournament of Towns - 1993 - 1997 (Book 4)
PJ Taylor e AM Storozhev
AMT
21. Olimpiada Matematica Espa
nola
15000 Problemas de diferentes Olimpiadas de Matematica en el Mundo
22. 360 Problems for Mathematical Contests
Titu Andreescu e Dorin Andrica
GIL
23. International Mathematical Talent Search
Part 1
G Berzsenyi
AMT
24. International Mathematical Talent Search
Part 1
23


POT 2012 - Algebra
- Nvel 3 - Aula 6 - Prof. Ccero Thiago/ Prof. Marcelo
Mendes

G Berzsenyi
AMT
25. Putnam and Beyond
Razvan Gelca e Titu Andreescu
Springer
26. A problem book in algebra
V. A. Krechmar
27. Matematica em nvel IME - ITA
N
umeros Complexos e Polin
omios
Caio dos Santos Guimaraes
28. A Matematica do ensino medio, vol. 4
Enunciados e soluc
oes dos exerccios
Elon Lages Lima, Paulo Cezar Pinto Carvalho, Eduardo Wagner e Augusto Cesar Morgado

29. Curso de Algebra,


Vol. 1.
Abramo Hefez
IMPA
30. Problem - Solving Strategies
Arthur Engel
Springer
31. Problems from the book
Titu Andreescu e Gabriel Dospinescu
XYZ Press

24

Polos Olmpicos de Treinamento


Curso de lgebra - Nvel 3

Aula

Prof. Ccero Thiago / Prof. Marcelo Mendes

Miscel
anea sobre razes de polin
omios II
Defini
c
ao 1: Seja P (x) = an xn + an1 xn1 + . . . + a1 x + a0 um polin
omio com an 6= 0 e

n1
n2
n > 0. Definiremos P (x) = nan x
+ (n 1)an1 x
+ . . . + a1 como sendo o polin
omio
que e a derivada do polin
omio P (x). As derivadas dos polin
omios P (x) e Q(x) satisfazem
(1) P (x) = k, k constante P (x) = 0.
(2) (P + Q) (x) = P (x) + Q (x).
(3) (P Q) (x) = P (x) Q (x).
(4) (P Q) (x) = P (x)Q(x) + P (x)Q (x).
Como consequencia de (3) temos que se x1 , x2 , . . ., xn s
ao razes de, um polin
omio de grau
n, P (x), entao
1
1
1
P (x)
=
+
+ ... +
.
P (x)
x x1 x x2
x xn
Em seguida, um teorema bem interessante sobre razes m
ultiplas de um polin
omio.

Teorema 1. Se r e raiz de multiplicidade m do polin


omio P (x), entao r e raiz de multi
plicidade m 1 do polin
omio P (x).
Demonstrac
ao. Temos que P (x) = (x r)m Q(x) P (x) = m(x r)m1 Q(x) + (x
m

r) Q (x) = (xr)m1 [mQ(x)+(xr)Q (x)] e, como mQ(r)+(rr)Q(r) = mQ(r) 6= 0,


ou seja, r e uma raiz de multiplicidade m 1 de P (x).
1. Determine um polin
omio P (x), de grau 5, tal que P (x) + 1 e divisvel por (x 1)3 e
P (x) 1 e divisvel por (x + 1)3 .
Soluc
ao. Se 1 e uma raiz de multiplicidade 3 de P (x) entao 1 e raiz de multiplicidade
2 do polin
omio P (x). Da mesma forma 1 e uma raiz de multiplicidade 2 de P (x).
Segue que P (x) e divisvel pelo polin
omio (x1)2 (x+1)2 . Mas, P (x) e um polin
omio
de grau 4. Ent
ao,
P (x) = c(x 1)2 (x + 1)2 = c(x4 2x2 + 1),


POT 2012 - Algebra
- Nvel 3 - Aula 7 - Prof. Ccero Thiago/ Prof. Marcelo
Mendes


1 5 2 3
x x + x + d, para c e d reais.
para alguma constante c. Agora, P (x) = c
5
3
3
5
15
15
e d = 0 e P (x) = x5 + x3 x.
Como P (1) = 1 e P (1) = 1, entao c =
8
8
4
8
Vamos ver uma outra soluc
ao.
3
Note que (x 1) divide P (x) + 1 e P (x) 1, entao (x 1)3 divide P (x) + P (x).
Alem disso, (x + 1)3 divide P (x) 1 e P (x) + 1, entao (x 1)3 divide P (x) + P (x).
Dessa forma, (x 1)3 (x + 1)3 divide P (x) + P (x), que e um polin
omio de grau 5, assim P (x) + P (x) = 0, x. Portanto, os coeficientes dos termos de grau par de P (x)
s
ao iguais a zero. Agora, P (x) + 1 = (x 1)3 (Ax2 + Bx 1). Com isso, B 3A = 0 e
3
9
3
5
15
3 + 3B A = 0 , ou seja, A = e B = . Finalmente, P (x) = x5 + x3 x.
8
8
8
4
8
2. Sejam x1 , x2 , . . . , xn1 , as razes diferentes de 1 do polin
omio P (x) = xn 1, n 2.
Prove que
1
1
n1
1
.
+
+ ... +
=
x x1 x x2
x xn1
2
Soluc
ao. Seja R(x) um polin
omio de grau n 1, cujas razes s
ao x1 , x2 , . . . , xn1 .
Segue que
1
1
1
R (x)
=
+
+ ... +
.
R(x)
x x1 x x2
x xn1
xn 1
= xn1 + xn2 + . . . + x + 1, entao R(1) = n e
x1
n(n 1)
. Dessa forma,
R (1) = (n 1) + (n 2) + . . . + 1 =
2

Por outro lado, R(x) =

1
1
R (1)
n1
1
+
+ ... +
=
=
.
1 x1 1 x2
1 xn1
R(1)
2
3. Prove que o polin
omio P (x) = 1 +

x x2
xn
+
+ ... +
n
ao possui razes m
ultiplas.
1
2!
n!

Soluc
ao. O polin
omio P possui uma raiz m
ultipla r se P (r) = P (r) = 0. Mas
n
rn
x
r = 0.
P (x) = P (x) + . Dessa forma, se r for uma raiz entao P (r) = P (r) +
n!
n!
Por outro lado, P (0) = 1. Assim, P n
ao possui razes m
ultiplas.
4. Determine a para que 1 seja uma raiz m
ultipla de P (x) = x5 ax2 ax + 1.
Soluc
ao. Temos que P (1) = 1 a + a + 1 = 0. Mas, P (1) = 0 5 + 2a a =
0 a = 5.


POT 2012 - Algebra
- Nvel 3 - Aula 7 - Prof. Ccero Thiago/ Prof. Marcelo
Mendes

5. Prove que (x 1)2 |nxn+1 (n + 1)xn + 1.


Soluc
ao. Temos que P (1) = n (n + 1) + 1 = 0 e P (1) = n(n + 1) (n + 1)n = 0.
Portanto, 1 e raiz com multiplicidade 2.
Exerccios propostos
1. Sejam x1 , x2 , . . ., xn as razes do polin
omio xn + xn1 + . . . + x + 1. Prove que
1
1
1
n
+
+ ... +
= .
1 x1 1 x2
1 xn
2
2. Demonstre que, se a equac
ao x3 ax + b = 0 (ab 6= 0), com a, b reais, tiver uma raiz
dupla, ent
ao a ser
a sempre positivo.
3. (ITA) Seja k R tal que a equac
ao 2x3 + 7x2 + 4x + k = 0 possua uma raiz dupla e
inteira x1 e uma raiz x2 , distinta de x1 . Entao, (k + x1 )x2 e igual a
(a) 6. (b) 3. (c) 1. (d) 2. (e) 8.
4. Prove que (x + 1)2 |x4n+2 + 2x2n+1 + 1.
5. Determine todos os polin
omios P (x), com coeficientes inteiros, que satisfazem P (P (x)) =

P (P (x)), x R.
6. Se a equac
ao x3 + ax2 + 3x + 1 = 0 tem raiz tripla, qual o valor de a?
7. Sejam P (z) e Q(z) polin
omios com coeficientes complexos, de grau maior ou igual a
1, tais que P (z) = 0 se, e somente se, Q(z) = 0 e P (z) = 1 se, e somente se, Q(z) = 1.
Prove que os polin
omios s
ao iguais.
Solu
co
es/Sugest
oes
1. Vamos fazer uma soluc
ao com uma ideia diferente das que foram trabalhadas nessa
aula. Observe o polin
omio com razes
yk =

1
, k = 1, 2, . . . , n.
1 xk

Da igualdade acima temos que


xk =
3

yk 1
,
yk


POT 2012 - Algebra
- Nvel 3 - Aula 7 - Prof. Ccero Thiago/ Prof. Marcelo
Mendes

em que xk e uma raiz de xn + xn1 + . . . + x + 1, dessa forma




yk 1
yk

n

yk 1
yk

n1

+ ... +

yk 1
+ 1 = 0.
yk

Au
ltima igualdade e equivalente a
(yk 1)n + yk (yk 1)n1 + . . . + ykn1 (yk 1) + ykn = 0.
Segue que yk e uma raiz do polin
omio
P (x) = (x 1)n + x(x 1)n1 + . . . + xn1 (x 1) + xn .
Queremos calcular
y1 + y2 + . . . + yn .
Observe que
n

P (x) = (n + 1)x x

n1

  

 
n
n1
1
+
+ ... +
+ ....
1
1
1

Usando relac
oes de Girard, temos que
y1 + y2 + . . . + yn =

n
1

n1
1

+ ... +
n+1

1
1

n(n + 1)
n
= .
2(n + 1)
2

4. Seja P (x) = x4n+2 + 2x2n+1 + 1. Entao, P (1) = 1 2 + 1 = 0 e P (1) =


(4n + 2) + 2(2n + 1) = 0.
5. Vamos primeiro considerar o caso em que n 2. Seja P (x) = an xn + an1 xn1 +
. . . + a0 , an 6= 0. Ent
ao
P (x) = nan xn1 + (n 1)an1 xn2 + . . . + a1 .
Fazendo a identidade dos coeficientes de xn(n1) na igualdade P (P (x)) = P (P (x)),
obtemos
an+1
nn = ann n.
n
Isto implica que an nn1 = 1, ou seja, an =

1
nn1

. Como an deve ser inteiro, entao

n = 1, o que e uma contradic


ao. Se n = 1, entao P (x) = ax + b. Dessa forma, temos
que a2 + b = a b = a a2 . A resposta do problema s
ao todos os polin
omios da
2
2
forma P (x) = ax + a a .


POT 2012 - Algebra
- Nvel 3 - Aula 7 - Prof. Ccero Thiago/ Prof. Marcelo
Mendes

Bibliografia
1. Problem - Solving Strategies
Arthur Engel
2. Putnam and Beyond
Razvan Gelca e Titu Andreescu
3. Fundamentos de Matematica Elementar, vol.6
Gelson Iezzi
4. 101 Problems in Algebra: Form the training of the USA IMO team.
Titu Andreescu e Zuming Feng
5. Mathematical Olympiad Treasures
Titu Andreescu e Bogdan Enescu
6. T
opicos de Matematica Elementar, vol. 6
Antonio Caminha Muniz Neto

Polos Olmpicos de Treinamento


Aula

Curso de lgebra - Nvel 3

Prof. Ccero Thiago / Prof. Marcelo Mendes

Aplica
c
oes de razes da unidade
Nesta aula vamos nos concentrar nas razes do polin
omio
P (x) = xn 1,
em que n e um inteiro positivo. As razes desse polin
omio s
ao chamadas de razes n facil ver que 1 e uma raiz desse polin

esimas da unidade. E
omio. Mas quais s
ao as
outras? Quais as aplicac
oes legais dessas razes?
Teorema 1. As razes de P (x) = xn 1 s
ao




2k
2k
+ i sin
, 0 k < n, k Z.
k = cos
n
n
Demonstrac
ao. Use o Teorema 2 da aula 5.
Dessa forma
0 = cos 0 + i s sin 0 = 1;

2
2
+ i sin
= ;
n
n
4
4
+ i sin
= 2;
2 = cos
n
n
...
2(n 1)
2(n 1)
n1 = cos
+ i sin
= n1 .
n
n
As razes n - esimas da unidade determinam um polgono regular inscrito em um crculo
unit
ario tal que um dos vertices e o ponto (1, 0).
1 = cos

Para n = 3, as razes c
ubicas da unidade s
ao
k = cos

2k
2k
+ i sin
, k {0, 1, 2},
3
3


POT 2012 - Algebra
- Nvel 3 - Aula 8 - Prof. Ccero Thiago/ Prof. Marcelo
Mendes
ou seja,

2
1
3
2
+ i sin
= +i
=
0 = 1, 1 = cos
3
3
2
2
e

4
4
1
3
2 = cos
+ i sin
= i
= 2.
3
3
2
2
As razes c
ubicas da unidade determinam um tri
angulo equilatero inscrito em um crculo
com centro em (0, 0) e raio 1, como podemos ver na figura a seguir.
y

O
b

1 x

Exerccios resolvidos
1. Prove que 1 + + 2 + 3 + . . . + n1 = 0 em que 1, , 2 , 3 , . . . , n1 s
ao as
razes n - esimas da unidade.
Soluc
ao.
Temos que
1 + x + x2 + x3 + . . . + xn1 =

xn 1
.
x1

Assim, fazendo x = , temos 1 + + 2 + 3 + . . . + n1 =


2. Calcule
Soluc
ao.

2000
2

2000
5

2000
8

2000
2000 .

+ ... +

Seja
2000

f (x) = (1 + x)

2000
X
k=0


2000 k
x .
k

n 1
= 0.
1


POT 2012 - Algebra
- Nvel 3 - Aula 8 - Prof. Ccero Thiago/ Prof. Marcelo
Mendes

3
1
. Ent
ao, 3 = 1 e 2 + + 1 = 0. Entao
Seja = + i
2
2

 
 



2000
2000
2000
2000
3
+
+
+ ... +
2
5
8
2000
= f (1) + f () + 2 f ( 2 )
= 22000 + (1 + )2000 + 2 (1 + 2 )2000
= 22000 + ( 2 )2000 + 2 ()2000
22000 + 2 + = 22000 1.
Dessa forma o valor da soma e

22000 1
.
3

3. (Leningrado) Dizemos que uma sequencia a0 , a1 , . . . , an de n


umeros reais e p balanceada para algum inteiro positivo p se
a0 + ap + a2p + . . . = a1 + ap+1 + a2p+1 + . . . = . . . =
= ap1 + a2p1 + a3p1 . . . .
Se a sequencia a0 , a1 , . . . , a49 e p - balanceada para p = 3, 5, 7, 11, 13, 17, prove
que a0 = a1 = . . . = a49 = 0.
Soluc
ao. Suponha que a sequencia a0 , a1 , . . . , an e p - balanceada para algum p e
seja S o valor das somas
a0 + ap + a2p + . . . , a1 + ap+1 + a2p+1 + . . . , . . . , ap1 + a2p1 + a3p1 + . . . .
O pr
oximo passo e introduzir o polin
omio P (x) = an xn + an1 xn1 + . . . + a1 x + a0 .
Seja uma raiz p - esima da unidade ( 6= 1), ou seja, p = 1. Assim, r = p+r =
2p+r = 3p+r = . . ., para cada r = 0, 1, 2, . . . , p 1. Dessa forma, k e igual a
alguma das potencias b
asicas 1, , 2 , . . . , p1 para todo inteiro positivo k. Entao
P () = an n + an1 n1 + . . . + a1 + a0
= (a0 +ap +a2p +. . .)+(a1 +ap+1 +a2p+1 +. . .) +. . .+(ap1 +a2p1 +a3p1 +. . .) p1
= S(1 + + 2 + . . . + p1 ) = 0.
Portanto, e uma raiz de P . Voltando ao problema, o polin
omio P possui grau 49.
Dessa forma, para p = 3 o polin
omio possuira duas razes, para p = 5 o polin
omio possuira quatro razes e, assim sucessivamente, ate que para p = 17 o polin
omio possuira
16 razes. Finalmente, o polin
omio P ter
a 2 + 4 + 6 + 10 + 12 + 16 = 50 razes. Como
P possui grau 49 ent
ao ele ser
a identicamente nulo, ou seja, a0 = a1 = . . . = a49 = 0.


POT 2012 - Algebra
- Nvel 3 - Aula 8 - Prof. Ccero Thiago/ Prof. Marcelo
Mendes

4. (USAMO) Se P (x), Q(x), R(x) e S(x) s


ao polin
omios tais que P (x5 ) + xQ(x5 ) +
2
5
4
3
2
x R(x ) = (x + x + x + x + 1)S(x), prove que (x 1) e um fator de P (x).
Soluc
ao.
2
2
+ i sin , ou seja, 5 = 1. Substituindo x por , 2 , 3 e 4 na
5
5
igualdade P (x5 ) + xQ(x5 ) + x2 R(x5 ) = (x4 + x3 + x2 + x + 1)S(x), temos

Seja = cos

P (1) + Q(1) + 2 R(1) = 0,


P (1) + 2 Q(1) + 4 R(1) = 0,
P (1) + 3 Q(1) + R(1) = 0,
P (1) + 4 Q(1) + 3 R(1) = 0.
Multiplicando cada uma das equacoes acima por , 2 , 3 e 4 , temos
P (1) 2 Q(1) 3 R(1) = 0,
2 P (1) 4 Q(1) R(1) = 0,

3 P (1) Q(1) 4 R(1) = 0,

4 P (1) 3 Q(1) 2 R(1) = 0.


Somando as 8 igualdades e usando o fato que 1 + + 2 + 3 + 4 = 0 encontramos
5P (1) = 0 P (1) = 0, ou seja, x 1|P (x).
5. Mostre que para cada n N maior que 1 vale
sin

2
3
(n 1)
n
sin
sin
. . . sin
= n1 .
n
n
n
n
2

Soluc
ao.
2
2
Sejam 1, , 2 , . . ., n1 , = cos
+ i sin , as razes do polin
omio xn 1.
n
n
Assim,
xn 1 = (x 1)(x )(x 2 ) . . . (x n1 ).
Se dividirmos os dois lados da igualdade acima por x 1, encontraremos
xn1 + xn1 + . . . + x + 1 = (x )(x 2 ) . . . (x n1 ).
Fazendo x = 1 na u
ltima equac
ao temos
n = (1 )(1 2 ) . . . (1 n1 ).
4


POT 2012 - Algebra
- Nvel 3 - Aula 8 - Prof. Ccero Thiago/ Prof. Marcelo
Mendes

Au
ltima igualdade implica que n = |1 ||1 2 | . . . |1 n1 |. Usando a definicao
de m
odulo de um n
umero complexo temos

|1 | =

s

2k
1 cos
n

2

2k
+ sin
=
n
2

2 2 cos

2k
k
= 2 sin
n
n

para k = 0, 1, . . . , n 1. Multiplicando tudo encontramos a igualdade desejada.


Exerccios propostos
1. (ARML) Seja z uma raiz de x5 1 = 0, com z 6= 1. Determine o valor de z 15 + z 16 +
z 17 + . . . + z 50 .
1
2. (AIME) Seja z um n
umero complexo tal que z + = 2 cos 3 . Determine o menor
z
1
inteiro maior que z 2000 + 2000 .
z
3. (Harvard - MIT) O polin
omio f (x) = x2007 +17x2006 +1 tem razesdistintasr1 , r2 , . . . ,
1
= 0 para
omio P de grau 2007 tem a propriedade que P rj +
r2007 . Um polin
rj
P (1)
j = 1, . . . , 2007. Determine o valor de
.
P (1)
4. (OCM) Seja A1 , A2 , . . ., An os vertices de um polgono regular de n lados inscrito
na circunferencia unit
aria S e A um ponto dessa circunferencia. Encontre o valor
m
aximo do produto P dos n segmentos A1 A, A2 A, . . . An A e a posicao de A para o
qual esse m
aximo ocorre.
5. Prove que para todo natural n e real satisfazendo n > 1 e sin 6= 0, o polin
omio
P (x) = xn sin x sin n + sin(n 1)
e divisvel pelo polin
omio Q(x) = x2 2x cos + 1.
6. (AIME) A equac
ao
x10 + (13x 1)10 = 0
possui 10 razes complexas r1 , r1 , r2 , r2 , r3 , r3 , r4 , r4 , r5 , r5 . Determine o valor
de
1
1
1
1
1
+
+
+
+
.
r1 r1 r2 r2 r3 r3 r4 r4 r5 r5
5


POT 2012 - Algebra
- Nvel 3 - Aula 8 - Prof. Ccero Thiago/ Prof. Marcelo
Mendes
 
 
+isin
.
7. Seja f (x) = x2004 +2x2003 +3x2002 +. . .+2004x+2005 e z = cos
1003
1003
Expresse o produto
f (z)f (z 2 ) . . . f (z 2005 )
na forma ab , em que a e b s
ao inteiros.
8. (AIME) Seja v e w n
umeros distintos escolhidos arbitrariamente
entre as razes de
p

z 1997 1 = 0. Determine a probabilidade de acontecer 2 + 3 |v + w|.


9. (Romenia) Seja Un o conjunto das razes n - esimas da unidade. Prove que as
afirmac
oes a seguir s
ao equivalentes:
a) existe um Un tal que 1 + Un ;
b) existe Un tal que 1 Un .
Bibliografia
1. Complex Numbers form A to Z
Titu Andreescu e Dorin Andrica
2. Equations and Inequalities
Jiri Herman, Radan Kucera e Jaromir Simsa
3. Precalculus
Richard Rusczyk
4. Problem - Solving Strategies
Arthur Engel
5. 101 Problems in Algebra - From the traninig of the USA IMO team
Titu Andreescu e Zuming Feng
6. Mathematical Miniatures
Svetoslav Savchev e Titu Andreescu

Polos Olmpicos de Treinamento


Curso de lgebra - Nvel 3

Aula

Prof. Ccero Thiago / Prof. Marcelo Mendes

Somas de Newton
Chamaremos de somas de Newton as somas das k - esimas potencias das razes de um
polin
omio. Iniciaremos este material com alguns problemas que servem de motivacao para
a sequencia da teoria.
!10
!10
1 5
1+ 5
+
.
Exemplo 1. Calcule
2
2

1+ 5
1 5
Soluc
ao. Seja x =
ey=
. Defina 1 = x + y = 1, 2 = x y = 1 e Sk =
2
2
xk + y k , k natural. Temos que x e y s
ao razes do polin
omio quadr
atico P (z) = z 2 z 1.
Dessa forma,
x2 x 1 = 0 (1)
e
y 2 y 1 = 0. (2)

Multiplique (1) por xk2 e (2) por y k2 , k 2, assim

xk xk1 xk2 = 0 (3)


e
y k y k1 y k2 = 0. (4)
Adicionando (3) e (4) temos
xk + y k (xk1 + y k1 ) (xk2 + y k2 ) = 0.
Sk Sk1 Sk2 = 0
Sk = Sk1 + Sk2 .
Portanto,
S0 = x0 + y 0 = 1 + 1 = 2.
S 1 = x1 + y 1 = x + y = 1


POT 2012 - Algebra
- Nvel 3 - Aula 9 - Prof. Ccero Thiago/ Prof. Marcelo
Mendes
S2 = S1 + S0 = 3
S3 = S2 + S1 = 4
S4 = S3 + S2 = 7
S5 = S4 + S3 = 11
S6 = S5 + S4 = 18
S6 = S5 + S4 = 18
S7 = S6 + S5 = 29
S8 = S7 + S6 = 47
S9 = S8 + S7 = 76
S10 = S9 + S8 = 123
Exemplo 2. Escreva Sk = xk + y k em funcao de Sk1 = xk1 + y k1 , Sk2 = xk2 + y k2 ,
1 = x + y e 2 = x y, k 2.
Soluc
ao.
Seja P (z) = z 2 1 z + 2 um polin
omio cujas razes s
ao x e y. Entao,
x2 1 x + 2 = 0 (1)
e
y 2 1 y + 2 = 0. (2)

Multiplicando (1) por xk2 e (2) por y k2 , k 2, temos

xk 1 xk1 + 2 xk2 = 0 (3)


e
y k 1 y k1 + 2 y k2 = 0. (4)
Somando (3) e (4) temos
xk + y k 1 (xk1 + y k1 ) + 2 (xk2 + y k2 ) = 0
Sk 1 Sk1 + 2 Sk2 = 0
Sk = 1 Sk1 2 Sk2 .

Exemplo 3. Escreva Sk = xk + y k + z k em funcao de Sk1 = xk1 + y k1 + z k1 ,


Sk2 = xk2 + y k2 + z k2 , Sk3 = xk3 + y k3 + z k3 , 1 = x + y + z, 2 = xy + yz + zx
e 3 = xyz, k 3.
Soluc
ao.
2


POT 2012 - Algebra
- Nvel 3 - Aula 9 - Prof. Ccero Thiago/ Prof. Marcelo
Mendes

Seja P (z) = z 3 1 z 2 + 2 z 3 um polin


omio cujas razes s
ao x, y e z. Entao,
x3 1 x2 + 2 x 3 = 0 (1)
y 3 1 y 2 + 2 y 3 = 0 (2)

z 3 1 z 2 + 2 z 3 = 0. (3)
Multiplicando (1) por xk3 , (2) por y k3 e (3) por z k3 , k 3, encontramos
xk 1 xk1 + 2 xk2 3 xk3 = 0 (4)
y k 1 y k1 + 2 y k2 3 y k3 = 0 (5)
z k 1 z k1 + 2 z k2 3 z k3 = 0. (6)
Somando (4), (5) e (6), temos
xk +y k +z k 1 (xk1 +y k1 +z k1 )+2 (xk2 +y k2 +z k2 )3 (xk3 +y k3 +z k3 ) = 0
Sk 1 Sk1 + 2 Sk2 3 Sk3 = 0
Sk = 1 Sk1 2 Sk2 + 3 Sk3 .
Teorema 1. (Newton) Seja P (x) = an xn + an1 xn1 + . . . + a1 x + a0 um polin
omio
e sejam r1 , r2 , . . ., rn as razes do polin
omio. Seja Sk = r1k + r2k + . . . + rnk , k n.
Entao, an Sk + an1 Sk1 + . . . + a0 Skn = 0. Em particular, quando k = n, temos
an Sn + an1 Sn1 + . . . + na0 = 0.
Demonstrac
ao. Como r1 , r2 , . . ., rn s
ao as razes de P (x) entao
P (ri ) = an rin + an1 rin1 + . . . + a1 ri + a0 = 0, i = 1, 2, . . . , n.
Multiplicando cada uma das equac
oes por rikn encontramos
an r1k + an1 r1k1 + . . . + a0 r1kn = 0
an r2k + an1 r2k1 + . . . + a0 r2kn = 0
..
.
an rnk + an1 rnk1 + . . . + a0 rnkn = 0
Somando todas as equac
oes encontramos
an (r1k + . . . + rnk ) + an1 (r1k1 + . . . + rnk1 ) + . . . + a0 (r1kn + . . . + rnkn ) = 0
3


POT 2012 - Algebra
- Nvel 3 - Aula 9 - Prof. Ccero Thiago/ Prof. Marcelo
Mendes

an Sk + an1 Sk1 + . . . + a0 Skn = 0.


Em particular, quando k = n, Skn = S0 = r10 + r20 + . . . + rn0 = n, assim an Sn + an1 Sn1 +
. . . + na0 = 0.
Exerccios Resolvidos
1. Sejam r1 , r2 , . . . , r1000 as razes de x1000 10x + 10 = 0. Determine o valor de
1000 .
r11000 + r21000 + . . . + r1000
Soluc
ao.
Temos que a1000 , a1 e a0 s
ao os u
nicos coeficientes diferentes de 0. Entao, pelo teorema de Newton, S1000 10S1 + 1000 10 = 0. Como o coeficiente de x999 e zero
temos que S1 = 0 e S1000 = 10000.
2. (Bulgaria) Sejam x e y n
umeros reais que satisfazem as equacoes
x2 + y 2 = 2

x3 + y 3 = 2 2.
Ache o valor de x4 + y 4 .
(a) 2 (b) 3 (c) 4 (d) 4 2

(e) n
ao pode ser determinado.

Soluc
ao.
Seja 1 = x + y e 2 = xy e usando as ideias do exemplo 2 temos que
Sk = 1 Sk1 2 Sk2 .
Assim
S 2 = 1 S 1 2 S 0
S2 = 1 1 22
S2 = 12 22 .

Da mesma forma
S 3 = 1 S 2 2 S 1

S3 = 1 (12 22 ) 2 1
S3 = 13 31 2 ,
4


POT 2012 - Algebra
- Nvel 3 - Aula 9 - Prof. Ccero Thiago/ Prof. Marcelo
Mendes

e
S 4 = 1 S 3 2 S 2

S4 = 1 (13 31 2 ) 2 (12 22 )

S4 = 14 412 2 + 222 .

O enunciado diz que S2 = 2 e S3 = 2 2, assim


12 22 = 2
e

13 31 2 = 2 2.

Queremos S4 = x4 + y 4 = 14 412 2 + 222 , com x, y reais. Por outro lado,


(12 22 )2 = 22 4 412 2 + 422 = 4 S4 + 222 = 4 S4 = 4 222 . Alem
disso,

13 31 2 = 2 2

1 (12 32 ) = 2 2

12 (12 32 )2 = (2 2)2
(2 + 22 )(2 2 )2 = 8

(1 + 2 )(4 42 + 22 ) = 4

4 42 + 22 + 42 422 + 23 = 4
23 322 = 0

2 = 0 ou 2 = 3.
Finalmente, S4 = 4 ou S4 = 14. Como x, y s
ao reais temos que S4 0, ou seja,
S4 = 4.
3. Determine todas as soluc
oes reais da equacao

1x+

15 + x = 2.

Soluc
ao.

Faca 4 1 x = a e 4 15 + x = b, assim a4 = 1 x e b4 = 15 + x. Dessa forma,


 4
a + b4 = 16
a + b = 2
Se 1 = a + b = 2 e 2 = a b. Assim, S4 = a4 + b4 = 14 412 2 + 222 =
24 162 + 222 = 16. Assim, 2 = 0 ou 2 = 8.
5


POT 2012 - Algebra
- Nvel 3 - Aula 9 - Prof. Ccero Thiago/ Prof. Marcelo
Mendes

1 caso: 2 = 0

a + b = 2
ab = 0

Assim, a = 0 e b = 2 ou a = 2 e b = 0. Se a = 0 e b = 2 entao x = 1. Se a = 2 e
b = 0 ent
ao x = 15.
2 caso: 2 = 8

a + b = 2
ab = 8

Nesse caso, a e b n
ao s
ao reais e, alem disso, x n
ao e real.
4. (OBM) Sejam a, b e c n
umeros reais n
ao nulos tais que a + b + c = 0. Calcule os
possveis valores de
(a3 + b3 + c3 )2 (a4 + b4 + c4 )
.
(a5 + b5 + c5 )2
Soluc
ao. Usando as ideias do exemplo 3, ou seja, Sk = 1 Sk1 2 Sk2 + 3 Sk3 ,
com 1 = a + b + c, 2 = ab + bc + ca e 3 = abc.
Alem disso, S1 = 1 = 0 ent
ao
S2 = a2 + b2 + c2 = (a + b + c)2 2(ab + bc + ca) = 12 22 = 22 ,
S3 = 1 S2 2 S1 + 3 S0 = 33 ,

S4 = 1 S3 2 S2 + 3 S1 = 222 ,
S5 = 1 S4 2 S3 + 3 S2 = 52 3 .
Portanto,

(a3 + b3 + c3 )2 (a4 + b4 + c4 )
(33 )2 222
18
=
= .
5
5
5
2
2
(a + b + c )
(52 3 )
25

5. Sejam x, y n
umeros reais n
ao nulos satisfazendo x2 + xy + y 2 = 0.
2001 
2001

y
x
+
.
x+y
x+y
Soluc
ao.

Determine


POT 2012 - Algebra
- Nvel 3 - Aula 9 - Prof. Ccero Thiago/ Prof. Marcelo
Mendes
x
y
x
y
xy
xy
facil ver
+
=1e

= 2
= 1. E
=
2
x+y x+y
x+y x+y
x + 2xy + y
xy

k 
k
x
y
y
x
2
que
e
s
ao as razes de t t + 1 = 0. Assim, Sk =
+
x+y x+y
x+y
x+y
satisfaz
Observe que

Sk+2 = Sk+1 Sk , k 0
S0 =
2,
S1 =
1

A sequencia Sk , k 0, e 2, 1, 1, 2, 1, 1, 2, 1, . . . e Sk = Sl para k l (mod 6).


Portanto, S2001 = S3 = 2.
Exerccios propostos
20 .
1. Sejam r1 , r2 , . . . , r20 as razes de x20 19x + 2. Determine r120 + r220 + . . . + r20
20 .
2. Sejam r1 , r2 , . . . , r20 as razes de x20 19x2 + 2. Determine r120 + r220 + . . . + r20

3. Fatore x3 + y 3 + z 3 3xyz.
4. Sejam x1 e x2 as razes do polin
omio P (x) = x2 6x + 1. Prove que xn1 + xn2 e um
inteiro n
ao divisvel por 5 para todo inteiro n
ao negativo n.
5. Determine todos os valores de a R tais que as razes x1 , x2 e x3 de x3 6x2 +ax+a =
0 satisfazem
(x1 3)3 + (x2 3)3 + (x3 3)3 = 0.
6. Mostre que se a, b e c R e a + b + c = 0, entao
a4 + b4 + c4 = 2(ab + ac + bc)2 .
7. Resolva o sistema de equac
oes
x + y + z = 2,
2

x + y 2 + z 2 = 6,
x3 + y 3 + z 3 = 8.
8. Sejam a, b, c n
umeros reais n
ao nulos tais que a+b+c = 0 e a3 +b3 +c3 = a5 +b5 +c5 .
6
Prove que a2 + b2 + c2 = .
5


POT 2012 - Algebra
- Nvel 3 - Aula 9 - Prof. Ccero Thiago/ Prof. Marcelo
Mendes

9. Se a3 + b3 + c3 = a2 + b2 + c2 = a + b + c = 1, prove que abc = 0.


10. Determine todas as soluc
oes reais do sistema
x+y+z =1
x3 + y 3 + z 3 + xyz = x4 + y 4 + z 4 + 1.
11. Prove que se a + b + c = 0, ent
ao
a5 + b5 + c5 a2 + b2 + c2
a7 + b7 + c7
=

.
7
5
2
12. Prove que se a + b + c = 0, ent
ao
a5 + b5 + c5
a3 + b3 + c3 a2 + b2 + c2
=

.
5
3
2
13. Sejam x1 e x2 as razes do polin
omio P (x) = x2 + x + c. Determine o valor de c se
2x32
2x31
+
= 1.
2 + x2 2 + x1
14. Prove que o n
umero
c=
e uma raiz de F (x) = x3 +

r
3

1
+
9

r
3

2
+
9

r
3

4
9

6x2 1.

Bibliografia
1. Equations and inequalities - Elementary Problems and Theorems in Algebra and Number Theory
Jiri Herman, Radan Kucera e Jaromir Simsa
2. the Art of Problem Solving, vol. 2: and Beyond
Richard Rusczyk e Sandor Lehoczky
3. Problem - Solving Strategies
Arthur Engel
4. T
opicos de Matematica Elementar, vol. 6 - Polin
omios
Antonio Caminha Muniz Neto
5. Polin
omios Simetricos - Revista Eureka 25
Carlos A. Gomes

Polos Olmpicos de Treinamento


Curso de lgebra - Nvel 3

Aula

10

Prof. Ccero Thiago / Prof. Marcelo Mendes

Diferencas finitas e o polin


omio interpolador de Lagrange.
1. Diferencas Finitas
Seja P (x) um polin
omio de grau m. Defina k+1 P (n) = k P (n + 1) k P (n), k 1,
1
com P (n) = P (n + 1) P (n).
Teorema 1. Seja P (x) um polin
omio de grau m, em que m > 0. Entao m P (n) e uma
constante diferente de zero.
Demonstrac
ao.
Seja P (x) = am xm + am1 xm1 + . . . + a1 x + a0 um polin
omio qualquer. Entao
P (n) = P (n + 1) P (n) =
am (n + 1)m + am1 (n + 1)m1 + . . . + a1 (n + 1) + a0 (am nm + am1 nm1 + . . . + a1 n + a0 ).

facil ver que o grau de P (n) e m 1 e seu termo de maior grau e am m nm1 , pois
E
1
am 6= 0.
Dessa forma, para k m, o grau de k P (n) e m k. Portanto, quando k = m o grau de
m P (n) e 0, assim m P (n) e uma constante diferente de zero.
Problema 1. Determine todos os polin
omios P (x) tais que P (x + 1) P (x) = 2x + 1, x.
Soluc
ao. Temos que P (n) = 2n + 1, assim 2 P (n) = P (n + 1) P (n) = 2(n + 1) +
1 (2n + 1) = 2. Como 2 P (n) e constante e diferente de zero entao P (x) tem grau 2, ou
seja, P (x) = ax2 + bx + c, assim
a(x + 1)2 + b(x + 1) + c ax2 bx c = 2x + 1
2ax + a + b = 2x + 1, x.

Dessa forma, 2a = 2 e a + b = 1. Portanto, a = 1 e b = 0 e P (x) = x2 + c, para alguma


constante c.


POT 2012 - Algebra
- Nvel 3 - Aula 10 - Prof. Ccero Thiago/ Prof. Marcelo
Mendes
Problema 2. (AIME) Sejam x1 , x2 , . . . , x7 n
umeros reais tais que
x1 + 4x2 + 9x3 + 16x4 + 25x5 + 36x6 + 49x7 = 1,
4x1 + 9x2 + 16x3 + 25x4 + 36x5 + 49x6 + 64x7 = 12,
9x1 + 16x2 + 25x3 + 36x4 + 49x5 + 64x6 + 81x7 = 123.
Determine o valor de 16x1 + 25x2 + 36x3 + 49x4 + 64x5 + 81x6 + 100x7 .
Soluc
ao.
Defina P (n) = (n + 1)2 x1 + (n + 2)2 x2 + (n + 3)2 x3 + . . . + (n + 7)2 x7 . Temos que P (0) = 1,
P (1) = 12, P (2) = 123 e que P (n) e um polin
omio quadr
atico. Desejamos calcular o valor
de P (3). Assim
P (0) = P (1) P (0) = 11,
P (1) = P (2) P (1) = 111,

2 P (0) = P (1) P (0) = 100.

Dessa forma, 2 P (n) = 100, n. Assim, 2 P (1) = P (2) P (1) 100 = P (2)
111 P (2) = 211. Mas, P (2) = P (3) P (2) 211 = P (3) 123 P (3) = 334.

2. Polin
omio interpolador de Lagrange
Vamos resolver um problema que serve de motivacao para a construcao do polin
omio interpolador de Lagrange.
Problema 3. Determine um polin
omio quadr
atico tal que P (1) = 4, P (1) = 2 e
P (2) = 1.
Soluc
ao. Seja P (x) = ax2 + bx + c, com a 6= 0, entao
P (1) = a(1)2 +b(1)+c = 4, P (1) = a12 +b1+c = 2 e P (2) = a22 +b2+c = 1.
Resolvendo o sistema linear encontramos P (x) = 2x2 + 3x + 1.
Resolver um sistema linear pode ser muito trabalhoso e difcil se o grau do polin
omio aumentar muito. Para resolver problemas deste tipo e outros problemas vamos estudar o
polin
omio interpolador de Lagrange.
Teorema 2. Dados n N, a0 , a1 , . . . , an e b0 , b1 , . . . , bn n
umeros complexos com
a0 , a1 , . . . , an distintos, existe um u
nico polin
omio P (x) tal que
P (i ) = i , 0 i n.
Demonstra
c
ao:


POT 2012 - Algebra
- Nvel 3 - Aula 10 - Prof. Ccero Thiago/ Prof. Marcelo
Mendes

Vamos inicialmente determinar o polin


omio. Para isto, observe os polin
omios
Dk (x) =
e facil ver que

(x 0 )(x 1 ) . . . (x k1 )(x k+1 ) . . . (x n )


(k 0 )(k 1 ) . . . (k k1 )(k k+1 ) . . . (k n )

Dk (i ) =

1, i = k
0, i =
6 k

Agora multiplique Dk (x) pelo n


umero k e, entao, adicione todos esses polin
omios resultando no polin
omio
P (x) =

n
X

k Dk (x)

k=0

que satisfaz as condic


oes do enunciado. Para demonstrar a unicidade sejam P1 e P2 dois
polin
omios, que satisfazem as condic
oes impostas. O Polin
omio H(x) = P1 (x) P2 (x) tem
grau no m
aximo n e possui n + 1 razes 0 , 1 , . . . , n , portanto, e identicamente nulo.
Com isso, P1 (x) P2 (x).
O polin
omio Dk (x) n
ao caiu do ceu entao vamos ver a sua construcao. Temos que

1, i = k
Dk (i ) =
0, i 6= k
Como Dk (i ) = 0 para todo i 6= k, ent
ao
Dk (x) = C(x 0 ) . . . (x k1 )(x k+1 ) . . . (x n ).
Para determinar o valor de C vamos substituir x = k e usar a condicao Dk (k ) = 1.
Entao,
1 = C(k 0 ) . . . (k k1 )(k k+1 ) . . . (k n ).
Portanto,
Dk (x) =

(x 0 )(x 1 ) . . . (x k1 )(x k+1 ) . . . (x n )


.
(k 0 )(k 1 ) . . . (k k1 )(k k+1 ) . . . (k n )

Problema 4. (Mandelbrot) Seja P (x) um polin


omio de grau 2 tal que P (0) = cos3 10 ,
P (1) = cos 10 sin2 10 e P (2) = 0. Determine P (3).
Soluc
ao. Usando o polin
omio interpolador de Lagrange temos
P (x) = P (0)

(x 1)(x 2)
(x 0)(x 2)
(x 0)(x 1)
+ P (1)
+ P (2)
.
(0 1)(0 2)
(1 0)(1 2)
(2 0)(2 1)

Como P (0) = cos3 10 , P (1) = cos 10 sin2 10 e P (2) = 0 entao


POT 2012 - Algebra
- Nvel 3 - Aula 10 - Prof. Ccero Thiago/ Prof. Marcelo
Mendes

P (x) = cos3 10

(x 1)(x 2)
(x 0)(x 2)
(x 0)(x 1)
+ cos 10 sin2 10
+0
.
(0 1)(0 2)
(1 0)(1 2)
(2 0)(2 1)

Queremos P (3) assim

P (3) = cos3 10

(3 1)(3 2)
(3 0)(3 2)
(3 0)(3 1)
+ cos 10 sin2 10
+0

(0 1)(0 2)
(1 0)(1 2)
(2 0)(2 1)
P (3) = cos3 10 1 + cos 10 sin2 10 (3)
P (3) = cos3 10 3 cos 10 sin2 10
P (3) = cos3 10 3 cos 10 (1 cos2 10 )
P (3) = 4 cos3 10 3 cos 10

P (3) = cos 3 10 = cos 30 =

3
.
2

Problema 5. (IMO Short List)Seja F1 = F2 = 1, Fn+2 = Fn+1 + Fn e seja f um polin


omio
de grau 990 tal que f (k) = Fk , k {992, 993, . . . , 1982}. Mostre que f (1983) = F1983 1.
Soluc
ao. Temos que f (k + 992) = Fk+992 , para k = 0, 1, . . . , 990 e precisamos provar que
f (992 + 991) = F1983 1. Seja g(x) = f (x + 992), que tambem possui grau 990. Nosso
novo problema e tal que se g(k) = Fk+992 , para k = 0, 1, . . . , 990, entao g(991) = F1983 1.
Usando o polin
omio interpolador de Lagrange temos que
g(x) =

990
X
k=0

g(k)

(x 0)(x 1) . . . (x k + 1)(x k 1) . . . (x 990)


.
(k 1)(k 2) . . . 1 (1) . . . (k 990)

Entao
990
X




990 
X
991
991
k
g(991) =
g(k)
(1) =
Fk+992 (1)k .
k
k
k=0
k=0

an bn
1+ 5
1 5
Sabemos que Fn =
, em que a =
eb=
. Assim,
2
2
5

990 
X
991
k=0

" 990  
#

990 
X 991
X
991
1
ak+992 (1)k
bk+992 (1)k .
Fk+992 (1)k =
k
5 k=0 k
k=0
4


POT 2012 - Algebra
- Nvel 3 - Aula 10 - Prof. Ccero Thiago/ Prof. Marcelo
Mendes

Usando bin
omio de Newton temos que

990 
X
991
k=0

k+992

(1) = a

992


990 
X
991
k=0



(a)k = a992 (1 a)991 + a991 .

Mas a2 = a + 1, ent
ao


a992 (1 a)991 + a991 = a(a a2 )991 + a1983 = a + a1983 .

Temos que b2 = b + 1 ent


ao


990 
X
991
k=0

1
Fk+992 (1)k = (a1983 b1983 a + b)
5

a1983 b1983 a b

= F1983 1.
5
5

1. Determine o polin
omio P , de menor grau possvel, que satisfaz P (1) = 3, P (2) = 7,
P (3) = 13, P (4) = 21 e P (5) = 31. (AHSME)
2. Um polin
omio c
ubico f (n) satisfaz f (0) = 5, f (1) = 4, f (2) = 17 e f (3) = 56. Determine f (4).
3. (AIME) A partir de uma sequencia de n
umeros reais A = a1 , a2 , a3 , . . ., defina A
como a sequencia de n
umeros reais a2 a1 , a3 a2 , a4 a3 , . . ., em que o n - esimo
termo e an+1 an . Se todos os termos da sequencia (A) s
ao iguais a 1, e que
a19 = a92 = 0, determine a1 .
4. Determine um polin
omio de grau 3 tal que:
(a) P (1) = 2, P (2) = 1, P (3) = 4 e P (4) = 3.
(b) P (1) = 1, P (i) = 2, P (1) = 3 e P (i) = 4.
5. Se f e um polin
omio de grau n tal que f (i) = 2i para i = 0, 1, . . . , n, determine
f (n + 1).
6. (Mandelbrot) Se P (x) e um polin
omio de grau n tal que P (0) = 1, P (1) = 1,
P (2) = 1, P (3) = 1, . . ., P (n) = (1)n . Determine P (n + 1).
7. (Mandelbrot) Se P (x) e um polin
omio de grau n com P (1) = 1, P (2) = 3, P (4) = 9,
. . . e P (2n ) = 3n . Determine P (2n+1 ).


POT 2012 - Algebra
- Nvel 3 - Aula 10 - Prof. Ccero Thiago/ Prof. Marcelo
Mendes

8. (USAMO)Se P (x) e um polin


omio de grau n tal que P (k) =
determine P (n + 1).

k
, k = 0, 1, . . . , n,
k+1

9. (IMO Short List)Se P (x) e um polin


omio de grau n tal que P (i) =
i = 0, 1, 2, . . . , n. Determine P (n + 1).

n+1 ,
k

para

10. Resolva o sistema

ax1

bx1
cx

dx1

+
+
+
+

a2 x2
b2 x2
c2 x2
d2 x2

+
+
+
+

a3 x3
b3 x3
c3 x3
d3 x3

+
+
+
+

a4 x4
b4 x4
c4 x4
d4 x4

=
=
=
=

1
1
1
1

se a, b, c, d s
ao reais n
ao - nulos e distintos.
11. (Reino Unido) Sejam a1 , a2 , . . . , an n
umeros inteiros positivos distintos. Prove que
n
X
ak
Y i
para qualquer inteiro positivo k o n
umero
e um inteiro.
(ai aj )
i=1
j6=i

12. Sejam x1 , x2 , . . . , xn , n 2, n n
umeros reais distintos no intervalo [1, 1]. Prove
que
1
1
1
+ + ... +
2n2 ,
t1 t2
tn
Y
em que
|xi xj |.
j6=i

13. Prove que se m e n s


ao inteiros, 1 < m < n, entao
n
X
k=1

14. Prove a identidade

n
X

 
n
(1) k
= 0.
k
k m

(1)nk kn+1

k=0

 
n
n(n + 1)!
.
=
2
k

15. Seja f R [X] um polin


omio de grau n com coeficiente lder igual a 1, e sejam
x0 < x1 < . . . < xn n
umeros inteiros. Prove que existe k {0, 1, . . . , n} tal que
|f (xk )|
6

n!
.
2n


POT 2012 - Algebra
- Nvel 3 - Aula 10 - Prof. Ccero Thiago/ Prof. Marcelo
Mendes

16. (IMO Short List - 1997) Seja f um polin


omio com coeficientes inteiros, e seja p um
n
umero primo tal que f (0) = 0, f (1) = 1 e f (k) e congruente a 0 ou 1 m
odulo p,
para todo inteiro positivo k. Mostre que o grau de f e no mnimo p 1.
17. (USAMO) Prove que qualquer polin
omio m
onico de grau n, com coeficientes reais,
pode ser escrito como media aritmetica de dois polin
omios m
onicos de grau n com n
razes reais cada.
umeros reais tais que bi aj 6= 0 para i, j =
18. Sejam a1 , a2 , a3 , a4 , b1 , b2 , b3 , b4 n
1, 2, 3, 4. Suponha que exista um u
nico conjunto de n
umeros X1 , X2 , X3 , X4 tais
que
X2
X3
X4
X1
+
+
+
= 1.
b1 a1 b1 a2 b1 a3 b1 a4
X1
X2
X3
X4
+
+
+
= 1.
b2 a1 b2 a2 b2 a3 b2 a4
X1
X2
X3
X4
+
+
+
= 1.
b3 a1 b3 a2 b3 a3 b3 a4
X2
X3
X4
X1
+
+
+
= 1.
b4 a1 b4 a2 b4 a3 b4 a4

Determine X1 + X2 + X3 + X4 em funcao de a1 , a2 , a3 , a4 , b1 , b2 , b3 , b4 .

Bibliografia
1. Problems from the book
Titu Andreescu e Gabriel Dospinescu
2. Winning Solutions
Edward Lozansky e Cecil Rousseau
3. The Mandelbrot Problem Book
Sam Vandervelde
4. Kvant Selecta: Algebra and Analysis, II
Serge Tabachnikov
5. Lagrange Interpolation Formula
Kin Y. Li
Mathematical Excalibur - July - September, 2010

Polos Olmpicos de Treinamento


Aula

Curso de lgebra - Nvel 3

11

Prof. Ccero Thiago / Prof. Marcelo Mendes

Polin
omios em Z[x] (1)
Dizemos que P (x) Z[x] se P (x) = an xn + an1 xn1 + . . . + a1 x + a0 , com an , an1 , . . .,
a1 e a0 inteiros.
Teorema 1. Sejam a e b n
umeros inteiros distintos e P (x) um polin
omio com coeficientes
inteiros, entao a b|P (a) P (b).
Demonstrac
ao.
Seja P (x) = an xn + an1 xn1 + . . . + a1 x + a0 com an , an1 , . . ., a1 e a0 inteiros. Entao,
P (a) P (b) = (an an + an1 an1 + . . . + a1 a + a0 ) (an bn + an1 bn1 + . . . + a1 b + a0 )
=

n
X
i=0

n
X
i=0

= (a b)

"

n
X
i=0

ai (ai bi )

ai (a b)(ai1 + . . . + bi1 )
#

ai (ai1 + . . . + bi1 ) = (a b) k, k Z.

Portanto, a b|P (a) P (b).

Problema 1. Seja P um polin


omio m
onico com coeficientes inteiros. Seja x um real tal
que:
(i) P (x) = 0
(ii) P (x + x) = 2P (1) + P (0) + 1.
Prove que x e irracional.
Soluc
ao.


POT 2012 - Algebra
- Nvel 3 - Aula 11 - Prof. Ccero Thiago/ Prof. Marcelo
Mendes
Lema: O n
umero x + x e par se, e somente se, x e inteiro.
Demonstrac
ao. Se x e inteiro, ent
ao x = x = x, entao x + x = 2x, que e um
n
umero par. Se x n
ao e inteiro, ent
ao k < x < k + 1, para algum inteiro k. Logo x = k
e x = k + 1, onde x + x = 2k + 1, que e um n
umero mpar. Por outro lado, x e uma
raiz do polin
omio pela condic
ao i. Pelo teorema das razes racionais e como P e m
onico
entao toda raiz racional e inteira. Portanto, basta mostrar que x n
ao e inteiro.//
Se x e inteiro ent
ao x + x e par. Dessa forma,
x + x|P (x + x) P (0) = 2P (1) + 1,
ou seja, x + x deveria ser mpar. Portanto, x e irracional.
Problema 2. Seja P (x) um polin
omio com coeficientes inteiros tal que P (21) = 17, P (32) =
247 e P (37) = 33. Prove que se P (N ) = N + 51 para algum inteiro N , entao N = 26.
Soluc
ao. Usando o teorema 1 temos
N 21|N + 34 N 21|N + 34 (N 21) N 21|55 (1)
N 32|N + 298 N 32|N + 298 (N 32) N 32|330 (2)
N 37|N + 18 N 37|N + 18 (N 37) N 37|55 (3)
De (1) temos que N 21 = 1, 5, 11 ou 55. Os possveis valores de N s
ao:
22, 20, 26, 16, 32, 10, 76 e 34. O u
nico destes valores de N que n
ao contradiz 2 e
3 e N = 26.
Problema 3. (Peru TST) Seja k um inteiro positivo e seja P (x) um polin
omio com coeficientes inteiros. Prove que existe um inteiro positivo n tal que
P (1) + P (2) + . . . + P (n)
e divisvel por k.
Soluc
ao. Pelo teorema 1 temos que P (kj + i) P (i) (m
od kj), em particular, P (kj + i)
P (i) (m
od k). Logo,
P (1) + P (2) + . . . + P (k 2 ) =

k
k1 X
X
j=0 i=1

k
k1 X
X

P (i) (m
od k)

j=0 i=1

P (kj + i)


POT 2012 - Algebra
- Nvel 3 - Aula 11 - Prof. Ccero Thiago/ Prof. Marcelo
Mendes

k
X

P (i)

i=1

(m
od k)

0 (m
od k)

Logo, considerando n = k2 , temos que P (1) + P (2) + . . . + P (n) e divisvel por k.


Exerccios propostos

1. (USAMO) Sejam a, b e c n
umeros inteiros distintos. Prove que n
ao e possvel encontrar um polin
omio P (x) de coeficientes inteiros tal que P (a) = b, P (b) = c e P (c) = a.
2. (AIME) Seja P (x) um polin
omio com coeficientes inteiros tal que P (17) = 10 e
P (24) = 17. Alem disso, a equacao P (n) = n + 3 possui duas solucoes inteiras distintas n1 e n2 . Calcule n1 n2 .
3. (IMO) Seja P (x) um polin
omio de grau n > 1 com coeficientes inteiros e seja k um
inteiro positivo. Considere o polin
omio Q(x) = P (P (. . . P (P (x)) . . .)), onde P ocorre
k vezes. Prove que existem no m
aximo n inteiros t tais que Q(t) = t.
4. (Romenia TST) Seja n um inteiro positivo e
f (x) = am xm + am1 xm1 + . . . + a1 x + a0 ,
com m 2, um polin
omio com coeficientes inteiros, tal que:
i. a2 , a3 , . . . , am s
ao divisveis por todos os fatores primos de n,
ii. a1 e n s
ao primos entre si.
Prove que para qualquer inteiro positivo k, existe um inteiro positivo c, tal que f (c)
e divisvel por nk .
5. (Baltic Way) Seja P um polin
omio com coeficientes inteiros tal que P (n) < P (n) <
n para algum inteiro n. Prove que P (n) < n.
Bibliografia
1. III Olimpiada Nacional Escolar de Matematica - 2006
Jorge Tipe, John Cuya, Claudio Espinoza e Sergio Vera

Polos Olmpicos de Treinamento


Aula

Curso de lgebra - Nvel 3

12

Prof. Ccero Thiago / Prof. Marcelo Mendes

Irredutibilidade de polin
omios.
Continuaremos, neste artigo, trabalhando com polin
omios em Z[x]. Alem disso,vamos dizer que um polin
omio com coeficientes inteiros P (x) e irredutvel sobre Z se, e somente
se, n
ao for possvel escrever P (x) como produto de dois polin
omios (n
ao constantes) com
coeficientes inteiros. Vamos comecar com um problema de motivacao!!
Problema 1. Prove que o polin
omio
(x a1 )(x a2 ) . . . (x an ) 1,
em que a1 , a2 , . . ., an s
ao inteiros distintos, n
ao pode ser escrito como produto de dois
polin
omios n
ao constantes com coeficientes inteiros, ou seja, e irredutvel.
Soluc
ao.
Suponha, por contradic
ao, que
f (x) = (x a1 )(x a2 ) . . . (x an ) 1 = p(x)q(x),
em que p(x) e q(x) s
ao polin
omios com coeficientes inteiros com grau menor que n. Entao
g(x) = p(x) + q(x)
e um polin
omio com coeficientes inteiros com grau menor que n. Entao
p(ai )q(ai ) = f (ai ) = 1
e ambos p(ai ) e q(ai ) s
ao inteiros,
|p(ai )| = |q(ai )| = 1
e
p(ai ) + q(ai ) = 0.


POT 2012 - Algebra
- Nvel 3 - Aula 12 - Prof. Ccero Thiago/ Prof. Marcelo
Mendes
Assim, g(x) possui pelo menos n razes. Mas o grau g < n, entao g(x) 0. Entao
p(x) = q(x) e f (x) = p(x)2 ,
implicando que o coeficiente lder de f (x) e um n
umero negativo, o que e impossvel, pois
o polin
omio e m
onico.
Teorema 1. (Criterio de Einsenstein) Seja P (x) = an xn + an1 xn1 + . . . + a1 x + a0 um
polin
omio com coeficientes inteiros ai e seja p um n
umero primo que satisfaz as seguintes
condicoes
(i) p n
ao divide an ;
(ii) p divide a0 , a1 , . . . , an1 ;
(iii) p2 n
ao divide a0 .
Entao P (x) e irredutvel sobre Z.
Demonstrac
ao. Suponha, por absurdo, que existem polin
omios n
ao constantes F (x) e
G(x) com coeficientes inteiros tais que P (x) = F (x)G(x). Logo,
P (x) = an xn + . . . + a1 x + a0
F (x) = bs xs + . . . + b1 x + b0
G(x) = ct xt + . . . + c1 x + c0 .
Sem perda de generalidade podemos considerar que s t, como P (x) = F (x)G(x), ao
realizarmos o produto poderemos comparar os coeficientes dos termos semelhantes, obtendo
as seguintes igualdades
a0 = b0 c0
a1 = b0 c1 + b1 c0
a2 = b0 c2 + b1 c1 + b2 c0
..
.
as = b0 cs + b1 cs1 + . . . + bs1 c1 + bs c0
..
.
at = b0 ct + b1 ct1 + . . . + bs cts
..
.
an = bs ct .
Por (ii) e (iii) temos que a0 = b0 c0 e divisvel por p e n
ao e divisvel por p2 , assim, exatamente um dos n
umeros b0 e c0 e divisvel por p. Sem perda de generalidade suponha que
p divide b0 e que p n
ao divide c0 .
Como p divide a1 = b0 c1 + b1 c0 e b0 , ent
ao p divide b1 c0 , porem como p n
ao divide c0 , entao
2


POT 2012 - Algebra
- Nvel 3 - Aula 12 - Prof. Ccero Thiago/ Prof. Marcelo
Mendes

p divide b1 .
Portanto, raciocionando da mesma maneira, podemos demonstrar que p divide b2 , b3 , . . .,
bs1 e bs . Logo, p divide bs ct = an , contrariando (i).
Problema 2. Prove que o polin
omio P (x) = x2 + x + 1 e irredutvel sobre Z.
Soluc
ao. Nao e difcil provar que P (x) e irredutvel sobre Z se, e somente se, Q(x) =
P (x + 1) e irredutvel sobre Z, Logo
Q(x) = (x + 1)2 + (x + 1) + 1
= x2 + 3x + 3.
O primo p = 3 satisfaz o criterio de Eisenstein, portanto Q(x) e irredutvel sobre Z e, com
isso, P (x) tambem ser
a.
Problema 3. (TST Romenia) Sejam a, n n
umeros inteiros, e p um n
umero primo tal que
p > |a| + 1. Prove que o polin
omio f (x) = xn + ax + p n
ao pode ser representado como o
produto de dois polin
omios com coeficientes inteiros.
Soluc
ao. Seja z uma raiz complexa do polin
omio. Vamos provar que |z| > 1. Suponha que
n
|z| 1, entao z + az = p, ent
ao:
p = |z n + az| = |z||z n1 + a| |z n1 | + |a| 1 + |a|,
contrariando o fato que p > |a|+1. Agora, seja f (x) = g(x)h(x) uma decomposicao de f (x)
em polin
omios com coeficientes inteiros entao p = g(0)h(0), entao |g(0)| = 1 ou |h(0)| = 1.
Suponha que |g(0)| = 1. Se z1 , z2 , . . . , zk s
ao razes de g(x) entao s
ao tambem razes de
f (x), assim:
1 = |g(0)| = |z1 z2 . . . zk | = |z1 ||z2 | . . . |zk | > 1,
que e uma contradic
ao.

Exerccios Propostos
1. Prove que o polin
omio
P (x) = x101 + 101x100 + 102
e irredutvel em Z [x].
2. Prove que para todo n
umero primo p, o polin
omio
P (x) = xp1 + xp2 + . . . + x + 1
e irredutvel em Z [x].


POT 2012 - Algebra
- Nvel 3 - Aula 12 - Prof. Ccero Thiago/ Prof. Marcelo
Mendes
n

3. Prove que para todo inteiro positivo n, o polin


omio P (x) = x2 + 1 e irredutvel em
Z [x].
4. Prove que para quaisquer inteiros distintos a1 , a2 , . . ., an o polin
omio
P (x) = (x a1 )2 (x a2 )2 . . . (x an )2 + 1
n
ao pode ser escrito com um produto de dois polin
omios n
ao constantes com coeficientes inteiros.
5. Seja p um primo da forma 4k + 3, k inteiro. Prove que para qualquer inteiro positivo
n, o polin
omio (x2 + 1)n + p e irredutvel em Z[x].
6. Seja p um n
umero primo. Prove que o polin
omio
P (x) = xp1 + 2xp2 + 3xp3 + . . . + (p 1)x + p
e irredutvel sobre Z[x].
7. (IMO) Seja n > 1 um inteiro e f (x) = xn + 5xn1 + 3. Mostre que f (x) e irredutvel
sobre Z.
Bibliografia
1. III Olimpiada Nacional Escolar de Matematica - 2006
Jorge Tipe, John Cuya, Claudio Espinoza e Sergio Vera
2. Putnam and Beyond
Razvan Gelca e Titu Andreescu

Polos Olmpicos de Treinamento


Curso de lgebra - Nvel 3

Aula

13

Prof. Ccero Thiago / Prof. Marcelo Mendes

Substitui
c
ao trigonom
etrica.
Um metodo de resoluc
ao para equac
oes e inequacoes algebricas bastante eficiente e o da
substituicao trigonometrica. Vamos ver alguns exemplos.
Problema 1. Sejam a, b, c e d n
umeros reais tais que a2 + b2 = 1, c2 + d2 = 1 e ac + bd = 0.
Determine ab + cd.
Soluc
ao. Se a2 + b2 = 1 e c2 + d2 = 1 entao a = sin , b = cos , c = sin e d = cos , em
que 0 2 e 0 2. Neste caso temos
ac + bd = sin sin + cos cos = cos( ).
Pelas condicoes do problema ac + bd = 0 = cos( ). Alem disso,
ab + cd = sin cos + sin cos =
1
(sin 2 + sin 2) = sin( + ) cos( ) = 0.
2

Problema 2. Resolva a equac


ao x3 3x = x + 2.
facil ver que x 2. Considere os seguintes casos:
Soluc
ao. E
1. 2 x 2. Faca x = 2 cos a, 0 a , a equacao resultante e
p
8 cos3 a 6 cos a = 2(cos a + 1)
r
a
2 cos 3a = 4 cos2
2
a
cos 3a = cos .
2
a
a
Entao, 3a = 2m, m Z ou 3a + = 2n, n Z. Como 0 a , as solucoes neste
2
2
caso s
ao
4
4
e x = 2 cos
.
x = 2 cos 0 = 2, x = 2 cos
5
7


POT 2012 - Algebra
- Nvel 3 - Aula 13 - Prof. Ccero Thiago/ Prof. Marcelo
Mendes
2. x > 2. Ent
ao x3 4x = x(x2 4) > 0 e
x2 x 2 = (x 2)(x + 1) > 0

x > x + 2.
Portanto,
x3 3x > x >

x + 2,

e, com isso, n
ao temos soluc
oes.
Problema 3. Sejam a, b e c n
umeros reais. Prove que
(ab + bc + ca 1)2 (a2 + 1)(b2 + 1)(c2 + 1).

< x, y, z < . Entao a2 + 1 =


2
2
sec2 x, b2 + 1 = sec2 y e c2 + 1 = sec2 z. Multiplicando os dois lados da desigualdade por
cos2 x cos2 y cos2 z encontramos

Soluc
ao. Seja a = tan x, b = tan y e c = tan z com

[(ab + bc + ca 1) cos x cos y cos z]2 1.


facil ver que
E
(ab + bc) cos x cos y cos z = sin x sin y cos z + sin y sin z cos x = sin y sin(x + z)
e
(ca 1) cos x cos y cos z = sin z sin x cos y cos x cos y cos z
= cos y cos(x + z).
Consequentemente,

[(ab + bc + ca 1) cos x cos y cos z]2

= [sin y sin(x + z) cos y cos(x + z)]2


= cos2 (x + y + z) 1.

Problemas propostos

1. (OBMU) Resolva a equac


ao em R: x =

2+

2 + x.

2. (ITA)
Para quais valores do par
ametro real a existe um n
umero real x satisfazendo

2
1 x a x.


POT 2012 - Algebra
- Nvel 3 - Aula 13 - Prof. Ccero Thiago/ Prof. Marcelo
Mendes

3. Dados quatro n
umeros reais distintos no intervalo (0, 1), mostre que existem dois
deles, x e y, tais que
p
p
1
0 < x 1 y 2 y 1 x2 < .
2

p
a a + a2 x 2 +
4. (IME)
Seja
a
uma
constante
real
positiva.
Resolva
a
equa
c
a

o
p

3a a a2 x2 = 2 2x, para x R e 0 x a.
5. Determine todas as soluc
oes reais da equacao x +

1 x2 =

2(2x2 1).

6. Seja xn uma sequencia satisfazendo


xn+1 =

3xn 1
, n 1.
xn + 3

Prove que a sequencia e peri


odica.

7. A sequencia xn satisfaz xn+2 + 2 xn 2 para todo n 1. Determine todos os


possveis valores de x1986 .
8. Sejam a, b e c n
umeros reais tais que ab + ac + bc = 1. Prove que
b
c
4abc
a
.
+
+
=
2
2
2
2
1a
1b
1c
(1 a )(1 b2 )(1 c2 )
9. Determine o maior valor de
S = (1 x1 )(1 y1 ) + (1 x2 )(1 y2 )
se x21 + x22 = y12 + y22 = c2 .
Bibliografia
1. Metodos no est
andares para la resolucion de ecuaciones y desigualdades.
V. P. Supr
un
2. Mathematical Olympiad Challenges
Titu Andreescu e Razvan Gelca
3. 103 trigonometry problems - From the training of the USA IMO team
Titu Andreescu e Zuming Feng

Polos Olmpicos de Treinamento


Aula

Curso de lgebra - Nvel 3

14

Prof. Ccero Thiago / Prof. Marcelo Mendes

Revis
ao I.
Problema 1. Sejam x, y e z n
umeros reais tais que

1 1 1
+ + = 1. Prove que
x y z

(x 1)(y 1)(z 1) 8.
Soluc
ao. A desigualdade e equivalente a




y1
z1
8
x1

x
y
z
xyz




1
1
8
1
.
1
1

1
x
y
z
xyz
Usando a condic
ao inicial e MA MG temos que
r
1
1 1
2
11
1 = + 2
= .
x
y z
yz
yz
2
2
1
1
e 1 . Multiplicando as tres desigualdades verificay
z
xy
zx
mos a validade da desigualdade inicial. A igualdade ocorre se, e somente se, x = y = z = 3.
Analogamente, 1

Problema 2. Sejam a, b, c R+ . Prove a desigualdade


abc
1

.
(1 + a)(a + b)(b + c)(c + 16)
81
Soluc
ao. Temos que
(1 + a)(a + b)(b + c)(c + 16)


a a
c
b
b 
c
(c + 8 + 8)
= 1+ +
b+ +
a+ +
2 2
2 2
2 2
r
r
r
r
2
2
2
3 a
3 ab
3 bc
3 64c
3
3
3
3
81abc.
4
4
4
4


Entao,

1
abc

.
(1 + a)(a + b)(b + c)(c + 16)
81


POT 2012 - Algebra
- Nvel 3 - Aula 14 - Prof. Ccero Thiago/ Prof. Marcelo
Mendes
Problema 3. (Col
ombia) Seja f : R R uma funcao. Prove que se a funcao
F (x) = f (x) +

1
1
(f (x))2 + (f (x))3
2
3

e peri
odica, ent
ao a func
ao f tambem ser
a.
1
1
Soluc
ao. Note que F (x) = g(f (x)), em que g(x) = x + x2 + x3 e uma funcao estrita2
3
mente crescente. Seja T o perodo de F , ou seja, F (x + T ) = F (x) para todo x. Como g e
estritamente crescente ent
ao ela e tambem injetora. Dessa forma, g(f (x + T )) = g(f (x))
f (x + T ) = f (x) para todo x, ou seja, f e peri
odica.
Exerccios propostos
1. Sejam a, b, c e d n
umeros reais positivos. Prove que (a2 + a + 1)(b2 + b + 1)(c2 + c +
2
1)(d + d + 1) 81abcd.
2. Prove que para quaisquer x, y reais positivos entao

x4

x
y
1
.
+ 2

2
4
+y
x +y
xy

3. Sejam x, y e z n
umeros reais positivos. Prove que
x
x+

(x + y)(x + z)
z
z+

y
y+

(y + x)(y + z)

(z + x)(z + y)

1.

4. Sejam x, y, z n
umeros reais positivos tais que x4 + y 4 + z 4 = 1. Determine o valor
mnimo de
y3
z3
x3
+
+
.
1 x8 1 y 8 1 z 8

5. Determine todas as func


oes f : N R tais que, para quaisquer k, m e n, vale
f (km) + f (kn) f (k)f (mn) 1.
6. (Rioplatense) A cada n
umero inteiro positivo n associamos um inteiro n
ao - negativo
f (n), de modo que sejam satisfeitas as seguintes condicoes:
(i) f (ab) = f (a) + f (b),
(ii) f (n) = 0, se n e primo maior que 10,
(iii) f (1) < f (243) < f (2) < 10.
Ache f (1998) sabendo que e menor que 10.


POT 2012 - Algebra
- Nvel 3 - Aula 14 - Prof. Ccero Thiago/ Prof. Marcelo
Mendes

7. (Coreia) Seja f : N N N uma funcao tal que f (1, 1) = 2,


f (m + 1, n) = f (m, n) + m e
f (m, n + 1) = f (m, n) n
para todos m, n N. Ache todos os pares (p, q) tais que f (p, q) = 2001.
8. Seja f : R R uma func
ao tal que f (x + 2) = f (x 1)f (x + 5), para todo x R.
Prove que f e peri
odica.
9. (Ira) Seja f : R+ R+ uma funcao estritamente decrescente tal que para todo
x, y R+ ,
f (x + y) + f (f (x) + f (y)) = f (f (x + f (y)) + f (y + f (x))).
Prove que f (f (x)) = x.
10. Seja n um inteiro positivo mpar e sejam x1 , x2 , . . . , xn n
umeros reais distintos.
Encontre todas as func
oes bijetivas
f : {x1 , x2 , . . . , xn } {x1 , x2 , . . . , xn }
tais que |f (x1 ) x1 | = |f (x2 ) x2 | = . . . = |f (xn ) xn |.

Polos Olmpicos de Treinamento


Aula

Curso de lgebra - Nvel 3

15

Prof. Ccero Thiago / Prof. Marcelo Mendes

Fun
c
oes definidas implicitamente III

Problema 1. A func
ao f satisfaz a equacao f (x + 1) + f (x 1) =
Prove que esta func
ao e peri
odica.

2f (x) para todo x real.

Soluc
ao. Usando a equac
ao que foi dada temos que

f (x + 2) + f (x) = 2f (x + 1)
i
h
= 2 2f (x) f (x 1)

= 2f (x) 2f (x 1)

f (x + 2) = f (x) 2f (x 1).
Segue que

f (x + 4) = f (x + 2) 2f (x + 1)

= f (x) 2 [f (x + 1) + f (x 1)]
= f (x).

Portanto,
f (x + 8) = f (x + 4) = f (x).
Problema 2. (Ira) Seja f : R+ R+ uma funcao estritamente decrescente tal que para
quaisquer x, y R+ ,
f (x + y) + f (f (x) + f (y)) = f (f (x + f (y)) + f (y + f (x))).
Prove que f (f (x)) = x.
Soluc
ao. Fazendo y = x temos
f (2x) + f (2f (x)) = f (2f (x + f (x))).


POT 2012 - Algebra
- Nvel 3 - Aula 15 - Prof. Ccero Thiago/ Prof. Marcelo
Mendes
Na u
ltima igualdade substitua x por f (x) entao
f (2f (x)) + f (2f (f (x))) = f (2f (f (x) + f (f (x)))).
Subtraindo as duas igualdades temos que
f (2f (f (x))) f (2x) = f (2f (f (x) + f (f (x)))) f (2f (x + f (x))).
Se f (f (x) > x, o lado esquerdo da u
ltima igualdade e um n
umero negativo, entao
f (f (x) + f (f (x)) > f (x + f (x))
f (x) + f (f (x)) < x + f (x),
o que e uma contradic
ao. Uma contradicao semelhante ocorre se f (f (x)) < x. Portanto,
f (f (x)) = x.
Problema 3. (Tchecoslovaquia) Seja f : R+ R+ uma funcao tal que
f (xf (y)) + f (yf (x)) = 2xy,
para quaisquer x, y R+ . Prove que f (x) = x para todo x.
Soluc
ao. Fazendo x = y temos que f (xf (x)) = x2 e, em particular para x = 1, f (f (1)) = 1.
Assim,
f (1)2 = f (f (1) f (f (1))) = f (f (1)) = 1
f (1) = 1.
Fazendo y = 1 na equac
ao original temos que f (x) + f (f (x)) = 2x. Esta condicao implica
que se f (a) = f (b) ent
ao a = b, ou seja, f e injetora. Agora vamos fazer a substituicao
1
x = zf (z) e y = , com z > 0, e lembrando que f (zf (z)) = z 2 temos:
z

 


1
1
1
f zf (z)f
f (zf (z)) = 2zf (z)
+f
z
z
z

 
1
= f (z).
f zf (z)f
z
 
1
1
Como f e injetora segue que f (z) f
= 1. Fazendo x = z e y = na equacao original
z
z
temos que
  


1
f (z)
f zf
+f
= 2.
z
z
 
1
1
Como f
ent
ao
=
z
f (z)




f (z)
z
+f
= 2.
f
f (z)
z
2


POT 2012 - Algebra
- Nvel 3 - Aula 15 - Prof. Ccero Thiago/ Prof. Marcelo
Mendes

Alem disso,
f

z
f (z)

f (z)
z

= 1.

Portanto,
f

z
f (z)

=f

f (z)
z

= 1 = f (1)

f (z) = z.

Exerccios propostos
1. (Austr
alia) Seja f : R R uma funcao tal que f (x + 2) = f (x 1)f (x + 5) para
todo x R. Prove que f e peri
odica.
2. (Austr
alia) Prove que existe uma u
nica funcao f : R R satisfazendo f (x) =
 
1
xf
e f (x) + f (y) = 1 + f (x + y) para quaisquer x, y R tais que x + y 6= 0.
x
1
e
3. (Austr
alia) Determine todas as funcoes reais f : R+ R+ tais que f (1) =
2
 
 
3
3
f (xy) = f (x)f
+ f (y)f
.
y
x
4. (IMO) Seja f : R R tal que, para alguma constante positiva a, f satisfaz
f (x + a) =

1 p
+ f (x) f (x)2 , x R.
2

Prove que f e peri


odica.
5. (IMO) Determine todas as func
oes f : R R tais que
f (x2 + f (y)) = f (x)2 + y,
para quaisquer x, y R.

Polos Olmpicos de Treinamento


Aula

Curso de lgebra - Nvel 3

16

Prof. Ccero Thiago / Prof. Marcelo Mendes

Sequ
encias I
1. Progress
ao Aritm
etica
Defini
c
ao 1: Uma progress
ao aritmetica e uma sequencia a1 , a2 , . . . ou somente (an )
(finita ou infinita) satisfazendo a2 a1 = a3 a2 = . . . = r; sendo r chamado de raz
ao da
progressao.
Teorema 1. Se (an ) e uma progress
ao aritimetica de raz
ao r, entao
an = a1 + (n 1)r,
para todo n inteiro e positivo.
Demonstrac
ao. Pela definic
ao de progressao aritmetica, temos
a2 a1 = r
a3 a2 = r
a4 a3 = r
..
.
an an1 = r.
Somando essas n 1 igualdades, obtemos an a1 = (n 1)r, isto e, an = a1 + (n 1)r.
Problema 1. Prove que n
ao existem inteiros positivos a e r tais que os n
umeros
a, a + r, a + 2r, a + 3r, . . . , a + nr, . . .
sejam todos quadrados perfeitos.


POT 2012 - Algebra
- Nvel 3 - Aula 16 - Prof. Ccero Thiago/ Prof. Marcelo
Mendes
Soluc
ao. Suponha que existem a e r como desejado. Para k suficientemente grande temos


r1
r1 2
e a + kr = q 2 , para algum q N. Mas neste caso temos q >
,
a + kr >
2
2
donde


r1
(q + 1)2 = q 2 + 2q + 1 > q 2 + 2
+ 1 = a + kr + r = a + (k + 1)r.
2
Logo, a + (k + 1)r n
ao ser
a quadrado perfeito. Um absurdo!
Problema 2. Prove que
aritmetica.

2,

3e

5 n
ao podem ser termos de uma mesma progressao

Problema 3. Prove que os termos de uma P.A. qualquer em que 0 n


ao participa verficam
a relacao:
1
1
1
n1
+
+ ... +
=
.
a1 a2 a2 a3
an1 an
a1 an
Problema 4. Prove que se a, b e c s
ao n
umeros positivos entao a2 , b2 e c2 est
ao em P.A.
1
1
1
,
e
tambem est
ao em P.A.
se, e somente se,
b+c a+c a+b
Problema 5. Prove que se uma progressao aritmetica de inteiros positivos contem um quadrado, entao ir
a conter infinitos quadrados.
Problema 6. O conjunto dos inteiros positivos e particionado em varias progressoes aritmeticas.
Prove que pelo menos um dos termos iniciais e divisvel pela raz
ao de sua progressao.
Problema 7. Prove que os termos de uma P.A. qualquer em que 0 n
ao participa verficam
a relacao:
1
1
n1
+ ... +
=
.

a1 + a2
an1 + an
a1 + an
Problema 8. (Putnan) Prove que n
ao existem quatro coeficientes binomiais consecutivos
  
 
 

n
n
n
n
,
,
,
k
k+1
k+2
k+3
(n, k inteiros positivos e 4 k + 3 n) que est
ao em progressao aritmetica.
Problema 9. (OCM) Os lados de um tri
angulo medem 3, 7 e 8, respectivamente. Mostre
que os angulos deste tri
angulo, medidos em graus, entao em progressao aritmetica.


POT 2012 - Algebra
- Nvel 3 - Aula 16 - Prof. Ccero Thiago/ Prof. Marcelo
Mendes

Teorema 2. A soma dos n primeiros termos da progressao aritmetica (an ) = (a1 , a2 , . . . ,


an , . . .) e igual a
(a1 + an )n
Sn =
.
2
Demonstrac
ao.
Sn = a1 + a2 + a3 + . . . + an2 + an1 + an
Sn = an + an1 + an2 + . . . + a3 + a2 + a1 .
Sa, 2Sn = (a1 + an ) + (a2 + an1 ) + . . . + (an + a1 ).
Observe que, ao passar de um parenteses para o seguinte, a primeira parcela aumenta de r
e a segunda parcela diminui de r, o que n
ao altera a soma. Portanto, todos os parenteses
s
ao iguais ao primeiro, (a1 + an ).
Logo, 2Sn = (a1 + an )n e Sn =

(a1 + an )n
.
2

Problema 10. (OCM) Seja S = 12 22 + 32 42 + . . . + 19982 + 19992 . Expresse S como


a soma de 1000 n
umeros mpares, todos eles termos de uma progressao aritmetica.
Soluc
ao. Usaremos apenas a fatorac
ao da diferenca de dois quadrados:
a2 b2 = (a + b)(a b).
Assim, teremos:
S = 12 + (32 22 ) + (52 42 ) + . . . + (19992 19982 )
= 1 + (3 + 2)(3 2) + (5 + 4)(5 4) + . . . + (1999 + 1998)(1999 1998)
= 1 + 5 + 9 + . . . + 3997.
Esta u
ltima express
ao contem extamente 1000 n
umeros mpares em P.A.
Problema 11. (China) Seja Sn a soma dos n primeiros termos de uma progressao aritmetica
S15
S1 S2
,
, . . .,
.
(an ). Se S15 > 0 e S16 < 0, determine o maior entre os n
umeros
a1 a2
a15
Problema 12. Demonstre que em toda P.A., com n
umero mpar de termos, o termo medio e
igual `a diferenca entre a soma dos termos de ordem mpar e a soma dos termos de ordem par.
Problema 13. (Espanha) Calcule a soma dos quadrados dos 100 primeiros termos de uma
progressao aritmetica, dado que a soma dos 100 primeiros termos e 1 e a soma dos termos
de ordem par (a2 , a4 , . . . , a100 ) e 1.


POT 2012 - Algebra
- Nvel 3 - Aula 16 - Prof. Ccero Thiago/ Prof. Marcelo
Mendes

Problema 14. Suponha que a1 , a2 , . . . , an est


ao em progressao aritmetica. Ache a formula
para a21 + a22 + . . . + a2n em termos de n, a1 e an .
Problema 15. (ITA) Numa progress
ao aritmetica com n termos, n > 1, sabemos que o
1+n
1 + 3n
primeiro termo e igual a
e a soma deles vale
. Entao o produto da raz
ao desta
n
2
progressao pelo u
ltimo termo e igual a:
2
3
(a) 2n (b)
(c) 3n (d)
(e) 5n
n
n
Problema 16. (ITA) Seja a1 , a2 , . . . uma progressao aritmetica infinita tal que
n
X
k=1

a3k = n 2 + n2 , n N .

Determine o primeiro termo e a raz


ao da progressao.
Problema 17. (IME) Determine as possveis progressoes aritmeticas para as quais o resultado da divis
ao da soma dos seus n primeiro termos pela soma dos seus 2n primeiros
termos seja independente do valor de n.
Problema 18. (AIME) Seja a1 , a2 , a3 , . . . uma progressao geometrica com raz
ao 1 tal que
S1 = a1 + a2 + a3 + . . . + a98 ,
S2 = a2 + a4 + a6 + . . . + a9 8.
(a) Ache uma equac
ao relacionando S1 e S2 .
(b) Determine o valor de S2 sabendo que S1 = 137.

2. Progress
ao Geom
etrica
Defini
c
ao 2: Uma sequencia a1 , a2 , . . . , an , . . . e uma progressao geometrica se existe
um n
umero q tal que para cada k = 1, 2, 3, . . . ,
ak+1 = qak
Teorema 3. Se (an ) e uma progress
ao geometrica de raz
ao q,entao
an = a1 q n1 ,
para todo inteiro positivo n.


POT 2012 - Algebra
- Nvel 3 - Aula 16 - Prof. Ccero Thiago/ Prof. Marcelo
Mendes

Demonstrac
ao. Pela definic
ao de progressao geometrica e admitindo conhecidos o primeiro
termo (a1 6= 0), a raz
ao (q 6= 0) e o ndice (n) de um termo desejado, temos:
a2 = a1 q
a3 = a2 q
...
an = an1 q
Multiplicando essas n 1 igualdades, temos:
a2 a3 a4 . . . an = a1 a2 a3 . . . an1 q n1
an = a1 q n1
Teorema 4. A soma dos n primeiros termos de uma progressao geometrica (an ) = (a1 , a2 , . . . , an , . . .)
e igual a
a1 q n a1
Sn =
.
q1
Demonstrac
ao. Temos:
a1 + a1 q + a1 q 2 + . . . + a1 q n2 + a1 q n1 . (1)
Multiplicando ambos os membros por q, obtemos:
qSn = a1 q + a1 q 2 + . . . + a1 q n1 + a1 q n . (2)
(2) (1) qSn Sn = a1 q n a1 Sn (q 1) = a1 q n a1 .

Supondo q 6= 1, resulta:

Sn =

a1 q n a1
.
q1

Teorema 5. Se (a1 , a2 , . . . , an , . . .) e uma P.G. com raz


ao q tal que 1 < q < 1,entao
a1
.
S = a1 + a2 + a3 + . . . + an + . . . =
1q

A demonstrac
ao do teorema 5 ficar
a como exerccio.

Problema 19. Determine a raz


ao de uma P.G. de termos n
ao nulos tal que
an+2 = an+1 + an .
Soluc
ao. Sabemos que an = a1 q n1 . Entao
a1 q

n+1

= a1 q + a1 q

n1

1 5
q q1=0q =
.
2
2


POT 2012 - Algebra
- Nvel 3 - Aula 16 - Prof. Ccero Thiago/ Prof. Marcelo
Mendes

Problema 20. Prove que podemos eliminar alguns termos de um progressao aritmetica de
inteiros positivos de tal maneira, que sempre podemos rearranjar os termos formando uma
progressao geometrica.
Problema 21. (OCM) Determine a soma dos n primeiros termos da sequencia:
1, 1 + 2, 1 + 2 + 22 , 1 + 2 + 22 + 23 , . . . , 1 + 2 + 22 + 23 + . . . + 2n1 , . . .
Problema 22. Tres n
umeros reais n
ao nulos x, y, z, nessa ordem, est
ao em P.A. Seus
quadrados, na mesma ordem, tambem est
ao em P.A. Nessas condicoes, prove que x, y, z,
nessa ordem, tambem est
ao em P.G.
Problema 23. Uma progress
ao aritmetica e uma progressao geometrica tem, cada uma,
200 termos e a1 = b1 = 3 e a2 = b2 = 12. Determine os valores de i para os quais ai e um
divisor de bi .
Problema 24. (OCM) (a) Sabendo - se que os tres lados de um tri
angulo ret
angulo, de hipotenusa a, ent
ao em progress
ao geometrica. Determine os catetos do tri
angulo em funcao
apenas de a.
(b) Mostre que a altura relativa `
a hipotenusa tambem faz parte da progressao.
Problema 25. Suponha que a1 = 2 e ak+1 = 3ak + 1 para todo k 1. Ache uma formula
geral para a1 + a2 + . . . + an .
Problema 26. (AIME) A soma dos infinitos termos de uma progressao geometrica e 2005.
Uma nova sequencia obtida elevando ao quadrado cada termo da sequencia original, tem
soma dez vezes maior que a soma original. Determine a raz
ao da sequencia original.
Bibliografia
1. Fundamentos de matem
atica elementar 4
Gelson Iezzi e Samuel Hazzan
2. Intermediate Algebra
Richard Rusczyk e Mathew Crawford

Polos Olmpicos de Treinamento


Curso de lgebra - Nvel 3

Aula

17

Prof. Ccero Thiago / Prof. Marcelo Mendes

Sequ
encias II
1. Recorr
encias lineares
Uma recorrencia linear de ordem k com coeficientes constantes em uma vari
avel e
fn = cn1 fn1 + cn2 fn2 + . . . + cnk fnk + g(n),
em que c1 , c2 , c3 , . . ., cnk s
ao constantes e g(n) e uma funcao de n. A recorrencia linear
e chamada homogenea se g(n) 0 e, n
ao homogenea, caso contr
ario.

2. Recorr
encias lineares de ordem 2 homog
eneas
Teorema 1. Seja (fn )n1 uma sequencia de n
umeros reais tal que, para todo k 1 inteiro,
tenhamos
fk+2 + rfk+1 + sfk = 0,
onde r, s s
ao constantes reais dadas, sendo r 6= 0. Se a equacao x2 + rx + s = 0, chamada
de equacao caracterstica, tiver razes reais e , entao existem constantes reais A e B,
determinadas pelos valores de f1 e f2 , tais que:
(a) Se 6= , ent
ao fn = An1 + B n1 para todo n 1.
(b) Se = , ent
ao fn = An1 + B(n 1)n1 para todo n 1.
Problema 1. Determine o termo geral da sequencia de Fibonacci definida por Fn =
Fn1 + Fn2 , n 2, F1 = F2 = 1.
Soluc
ao.
A equacao caracterstica associada `
a equacao em questao (Fn = Fn1 + Fn2 ) e
x2 x 1 = 0,

1 5
1+ 5
e=
. As condicoes F1 = F2 = 1 implicam no sistema:
cujas razes s
ao =
2
2

A
+
B
= 1

!
!
1 5
1+ 5
A +
B = 1,

2
2


POT 2012 - Algebra
- Nvel 3 - Aula 17 - Prof. Ccero Thiago/ Prof. Marcelo
Mendes

1+ 5
1 5

cuja solucao e A =
e B = . Obtemos, portanto, a conhecida formula para
2 5
2 5
Fn :
!n
!n
1+ 5
1 5
1
1

, para n 1.
Fn =
2
2
5
5
Problema 2. Seja (an )n1 a sequencia dada por a1 = 1, a2 = 4 e, para todo inteiro positivo
k, ak+2 = 5ak+1 6ak . Calcule an em funcao de n.
Problema 3. (Romenia TST) Considere a sequencia (an )n0 definida por a0 = a1 = 1 e
an+1 = 14an an1 , n 1. Prove que para todo n 0, 2an 1 e um quadrado perfeito.
Problema 4. (Ibero) Seja (an ) e (bn ) duas sequencias de n
umeros inteiros que verificam as
seguinte condic
oes:
(i) a0 = 0; b0 = 8
(ii) an+2 = 2an+1 an + 2; bn+2 = 2bn+1 bn
(iii) a2n + b2n e um quadrado perfeito para todo n.
Determinar pelo menos dois valores do par (a1992 , b1992 ).

3. Recorr
encias n
ao - lineares
Problema 5. A sequencia (xn )n1 e tal que x1 = 0 e
p
xn+1 = 5xn + 24x2n + 1

para todo n 1. Prove que todos os termos da sequencia s


ao inteiros positivos.
facil ver que a sequencia e crescente e todos os termos s
Soluc
ao. E
ao positivos. Temos
tambem que a recorrencia original e equivalente a
x2n+1 10xn xn+1 + x2n 1 = 0.
Substituindo n por n 1 temos

x2n 10xn xn1 + x2n1 1 = 0.

Entao, para n 2, os n
umeros xn+1 e xn1 s
ao razes positivas e distintas da equacao
x2 10xxn + x2n 1 = 0.

Usando as relac
oes de Girard temos que
xn+1 + xn1 = 10xn
xn+1 = 10xn xn1 , n 2.

Como x1 = 1 e x2 = 10, segue indutivamente que os termos da sequencia s


ao inteiros e
positivos.


POT 2012 - Algebra
- Nvel 3 - Aula 17 - Prof. Ccero Thiago/ Prof. Marcelo
Mendes

Problema 6. Considere a sequencia (an )n1 tal que a1 = a2 = 1, a3 = 199 e


an+1 =

1989 + an an1
, n 3.
an2

Prove que todos os termos da sequencia s


ao inteiros e positivos.
a definida pelas seguintes condicoes:
Problema 7. (Torneio das cidades) A sequencia xn est
x1 = 19, x2 = 97, xn+2 = xn

1
.
xn+1

Demonstrar que existe um termo desta sequencia que e igual a 0. Determinar o ndice desse
termo.

Problema 8. (China) Seja (an ) uma sequencia tal que a1 = 1, a2 = 2 e

a2 + 1
an+2
= n+1
an
a2n + 1

para todo n 1.
(a) Determine an em func
ao de n.
(b) Prove que 63 < a2008 < 78.
Bibliografia
1. Lecture notes on mathematical olympiad courses - For senior section vol.2
Xu Jiagu
2. Mathematical Olympiad Treasures
Titu Andreescu e Bogdan Enescu
3. T
opicos de matem
atica elementar vol.4
Antonio Caminha Muniz Neto
4. Introducao `
a an
alise combinat
oria
Jose Plnio O. Santos, Margarida P. Mello e Idani T. C. Murari

Você também pode gostar